Vous êtes sur la page 1sur 307

1B Matematicas 1 - cub 29/2/08 11:10 P gina 1

C M Y CM MY CY CMY K

BACHILLERATO
1 Matemáticas
Matemáticas

Matemáticas
María José Ruiz Jiménez

1
Jesús Llorente Medrano
Carlos González García

BACHILLERATO
BACHILLERATO

ISBN 978-84-9771-345-0

9 788497 713450

Composici n
Índice

UNIDAD 1 – NÚMEROS REALES ....................................................................................................5


CUESTIONES INICIALES.............................................................................................................5
RESOLUCIÓN DE PROBLEMAS .................................................................................................6
Actividades finales p. 28................................................................................................................7
Actividades finales p. 29..............................................................................................................10
Actividades finales p. 30..............................................................................................................13
Actividades finales p. 31..............................................................................................................16
Actividades finales p. 32..............................................................................................................18

UNIDAD 2 – NÚMEROS REALES ..................................................................................................21


CUESTIONES INICIALES...........................................................................................................21
RESOLUCIÓN DE PROBLEMAS ...............................................................................................22
Actividades finales p. 52..............................................................................................................23
Actividades finales p. 53..............................................................................................................27
Actividades finales p. 54..............................................................................................................31

UNIDAD 3 – ECUACIONES Y SISTEMAS DE ECUACIONES. INECUACIONES.........................35


CUESTIONES INICIALES...........................................................................................................35
RESOLUCIÓN DE PROBLEMAS ...............................................................................................37
Actividades finales p. 80..............................................................................................................38
Actividades finales p. 81..............................................................................................................43
Actividades finales p. 82..............................................................................................................48
Actividades finales p. 83..............................................................................................................53
Actividades finales p. 84..............................................................................................................56

UNIDAD 4 – TRIGONOMETRÍA I ...................................................................................................60


CUESTIONES INICIALES...........................................................................................................60
RESOLUCIÓN DE PROBLEMAS ...............................................................................................62
Actividades finales p. 106............................................................................................................64
Actividades finales p. 107............................................................................................................69
Actividades finales p. 108............................................................................................................74

UNIDAD 5 – TRIGONOMETRÍA II ..................................................................................................79


CUESTIONES INICIALES...........................................................................................................79
RESOLUCIÓN DE PROBLEMAS ...............................................................................................81
Actividades finales p. 126............................................................................................................82
Actividades finales p. 127............................................................................................................87
Actividades finales p. 128............................................................................................................92

2
UNIDAD 6 – NÚMEROS COMPLEJOS ..........................................................................................96
CUESTIONES INICIALES...........................................................................................................96
RESOLUCIÓN DE PROBLEMAS ...............................................................................................97
Actividades finales p. 148............................................................................................................98
Actividades finales p. 149..........................................................................................................102
Actividades finales p. 150..........................................................................................................109

UNIDAD 7 – GEOMETRÍA ANALÍTICA EN EL PLANO...............................................................113


CUESTIONES INICIALES.........................................................................................................113
RESOLUCIÓN DE PROBLEMAS .............................................................................................115
Actividades finales p. 170..........................................................................................................117
Actividades finales p. 171..........................................................................................................121
Actividades finales p. 172..........................................................................................................126

UNIDAD 8 – LUGARES GEOMÉTRICOS. CÓNICAS ..................................................................132


CUESTIONES INICIALES.........................................................................................................132
RESOLUCIÓN DE PROBLEMAS .............................................................................................134
Actividades finales p. 194..........................................................................................................136
Actividades finales p. 195..........................................................................................................141
Actividades finales p. 196..........................................................................................................147

UNIDAD 9 – PROPIEDADES GLOBALES DE LAS FUNCIONES...............................................151


CUESTIONES INICIALES.........................................................................................................151
RESOLUCIÓN DE PROBLEMAS .............................................................................................152
Actividades finales p. 216..........................................................................................................154
Actividades finales p. 217..........................................................................................................158
Actividades finales p. 218..........................................................................................................161

UNIDAD 10 – FUNCIONES ELEMENTALES ...............................................................................165


CUESTIONES INICIALES.........................................................................................................165
RESOLUCIÓN DE PROBLEMAS .............................................................................................168
Actividades finales p. 240..........................................................................................................170
Actividades finales p. 241..........................................................................................................173
Actividades finales p. 242..........................................................................................................179
Actividades finales p. 243..........................................................................................................181
Actividades finales p. 244..........................................................................................................185

UNIDAD 11 – LÍMITES DE FUNCIONES. CONTINUIDAD...........................................................189


CUESTIONES INICIALES.........................................................................................................189
RESOLUCIÓN DE PROBLEMAS .............................................................................................190
Actividades finales p. 266..........................................................................................................193
Actividades finales p. 267..........................................................................................................196
Actividades finales p. 268..........................................................................................................199
Actividades finales p. 269..........................................................................................................203
Actividades finales p. 270..........................................................................................................207

3
UNIDAD 12 – INTRODUCCIÓN A LAS DERIVADAS ..................................................................210
CUESTIONES INICIALES.........................................................................................................210
RESOLUCIÓN DE PROBLEMAS .............................................................................................212
Actividades finales p. 292..........................................................................................................214
Actividades finales p. 293..........................................................................................................219
Actividades finales p. 294..........................................................................................................222
Actividades finales p. 295..........................................................................................................227
Actividades finales p. 296..........................................................................................................231

UNIDAD 13 – APLICACIÓN DE LAS DERIVADAS .....................................................................235


CUESTIONES INICIALES.........................................................................................................235
RESOLUCIÓN DE PROBLEMAS .............................................................................................237
Actividades finales p. 316..........................................................................................................239
Actividades finales p. 317..........................................................................................................242
Actividades finales p. 318..........................................................................................................245

UNIDAD 14 – INTRODUCCIÓN A LAS INTEGRALES Y SUS APLICACIONES ........................252


CUESTIONES INICIALES.........................................................................................................252
RESOLUCIÓN DE PROBLEMAS .............................................................................................253
Actividades finales p. 338..........................................................................................................255
Actividades finales p. 339..........................................................................................................259
Actividades finales p. 340..........................................................................................................263

UNIDAD 15 – DISTRIBUCIONES BIDIMENSIONALES. CORRELACIÓN Y REGRESIÓN ........268


CUESTIONES INICIALES.........................................................................................................268
RESOLUCIÓN DE PROBLEMAS .............................................................................................270
Actividades finales p. 360..........................................................................................................272
Actividades finales p. 361..........................................................................................................275
Actividades finales p. 362..........................................................................................................278

UNIDAD 16 – PROBABILIDAD ....................................................................................................281


CUESTIONES INICIALES.........................................................................................................281
RESOLUCIÓN DE PROBLEMAS .............................................................................................282
Actividades finales p. 384..........................................................................................................284
Actividades finales p. 385..........................................................................................................287
Actividades finales p. 386..........................................................................................................290

UNIDAD 17 – DISTRIBUCIONES DE PROBABILIDAD.


DISTRIBUCIONES BINOMIAL Y NORMAL..........................................................293
CUESTIONES INICIALES.........................................................................................................293
RESOLUCIÓN DE PROBLEMAS .............................................................................................294
Actividades finales p. 410..........................................................................................................296
Actividades finales p. 411..........................................................................................................300
Actividades finales p. 412..........................................................................................................304

4
Unidad 1 – Números reales
PÁGINA 9

SOLUCIONES

1. Diremos que:
2 3
a) Los números comprendidos entre y son: 0,42; 0,46; 0,54; 0,57.
5 5
b) Los números comprendidos entre 2,1 y 2,2 son: 2,11; 2,14; 2,18; 2,195.
c) Los números comprendidos entre 2,01 y 2,1 son: 2,03; 2,045; 2,076; 2,098.

2. Utilizando la tecla del producto podemos conseguir aproximaciones sucesivas del valor. Así:
3
2 2 2 10 3 3 3
2, 1 2, 1 2, 1 3
10 2, 2 2, 2 2, 2
2, 15 2, 15 2, 15 3
10 2, 16 2, 16 2, 16
2, 154 2, 154 2, 154 3
10 2, 155 2, 155 2, 155
2, 1544 2, 1544 2, 1544 3
10 2, 1545 2, 1545 2, 1545

3. La ordenación queda: 4,2101 4,21 4,201 4,201 4,211 5,201 5,2101 5,31

4. Elevando al cuadrado ambos miembros obtenemos:


2 2 2 2
2 2 3 6 2 4 2 3 6 2 2 12

8 4 3 8 4 3 Se verifica la igualdad

5. n par y a R + ó n impar y a R

5
PÁGINA 25

SOLUCIONES

1. 2 4 6 8 ... 2m m m 1

2. Resolvemos en los siguientes pasos:


Supongamos que el camello lleva un bidón hasta la mitad del camino, vuelve a Kamal,
carga con otro bidón hasta el mismo punto y se bebe uno de los bidones transportados,
quedándole otro. Repitiendo el proceso conseguirá llevar 50 bidones hasta la mitad del
camino. De aquí repitiendo lo mismo hasta Wadi conseguirá que lleguen 25 bidones
según la expresión:
12
50 bidones 100 ×
22

Si mejoramos la solución conseguiremos que lleguen más bidones, haciendo el camino


en tres fases tras el 1.er tercio, el camello habrá bebido 33,333 bidones y quedan
66,666 En el 2.do tercio se bebe 22,222 y quedan 44,444 En Wadi se bebe
14,81 y quedan 29,629 bidones, es decir:

8 23
100 × 100 × 3
27 3

Avanzando por cuartos de camino se puede mejorar la solución, llegan:

4
81 34 3
31,640 100 × 100× 4 100 ×
256 4 4

Siguiendo así sucesivamente se puede decir que en el mejor de los casos llegan:

100
99 1
100 × 100 ×
100 e

6
PÁGINA 28

7
SOLUCIONES

1. Quedan del siguiente modo:


7
3 ; 4,23 ; 13 ; 0 ; ;
3
12
64 8 ; 1,03 ; 4 ;
3
3 1 1,3
8 2 ; ;
0,5

2. Las siguientes afirmaciones son:


a) Verdadero, todos los decimales son relacionales excepto los decimales infinitos y no
periódicos.
b) Falso, sólo son naturales los enteros positivos y el cero.
15
c) Falso, es racional y entero.
3
d) Verdadero, todos los decimales infinitos no periódicos son reales e irracionales.

3. Quedaría representado del siguiente modo:

4. Las representaciones quedarían:

8
5. Quedan del siguiente modo:
a) (1, + ). No acotado.
b) ( 2,2]. Acotado. Inf 2. Máximo 2.
c) [ 4, 1] U [1,4). Acotado. Mínimo 4. Sup 4.
d) {x 1 x 3 }. Acotado. Mínimo 1. Máximo 3.
e) ( 3,3) {5}. Acotado. Inf 3. Máximo 5.
f) ( ,5 ] [5,+ ) {7}. No acotado.

6. Quedan del siguiente modo:


a) Acotado. Supremo 11; Ínfimo 1.
b) Acotado. Sup 1; Inf 0 Mínimo.
c) Acotado. Sup 3; Inf 3; Máximo 3; Mínimo 3.
d) Acotado. Sup 8; Inf 4; Máximo 8; Mínimo 4.
e) Acotado. Sup -4; Inf 6.
f) No acotado inferiormente, por tanto no está acotado.
g) Acotado. Sup 7; Inf 1.
h) Acotado. Sup 1; Inf 0; Máximo 1.

7. Quedan del siguiente modo:


a) E 2,4 E 2,2 6,2 0,4 0,2 . Acotado.
b) 2, . No acotado. Mínimo en el 2.

9
PÁGINA 29

10
SOLUCIONES

8. La tabla completa se presenta del siguiente modo:

Aproximación Aproximación
Redondeo a Truncamiento a
decimal a milésimas decimal a centésimas
Valor exacto décimas y cota centésimas y cota de
por defecto y cota por exceso y cota de
de error error
de error error

2,236 2,24 2,2 2,23


2,236067
Cota 0,001 Cota 0,01 Cota 0,05 Cota 0,01
3,421 3,43 3,4 3,42
3,42173
Cota 0,001 Cota 0,01 Cota 0,05 Cota 0,01
0,764 0,77 0,8 0,76
0,7643
Cota 0,001 Cota 0,01 Cota 0,05 Cota 0,01
32,427 32,43 32,4 32,42
32,42751
Cota 0,001 Cota 0,01 Cota 0,05 Cota 0,01

9. Para cada uno de los números queda:

1 725 no es redondeo.
1 724,16 es un redondeo a centésimas. Cota de error 0,005.
1 724,2 es un redondeo a décimas. Cota de error 0,05.
1 724,1 no es redondeo.
1 724,158 no esredondeo.
1 724,1572 es un redondeo a diezmilésimas. Cota de error 0,00005.

10. Realizamos el siguiente cálculo:

Consideramos como valor real 3,141592.


Error absoluto :
221
3,141592 0,028916...
71
Error relativo :
Error absoluto 0,028916
0,0092...
Valor real 3,141592

11. El número de oro es :


1,61803398...
Redondeo a centésimas : 1,62
Error absoluto 0,00197...
Error relativo 0,00121506...

11
12. La notación científica queda:

a) 3,84 × 105. Orden de magnitud 105.


b) 1,5 × 108. Orden de magnitud 108.
c) 2,2 × 10 -9. Orden de magnitud 10-9.
d) 5 × 10 -11. Orden de magnitud 10 -10.
e) 5,7 × 1013. Orden de magnitud 1014.
f) 6,23 × 10-3. Orden de magnitud 10 -2.
g) 3,5 × 10 4. Orden de magnitud 10 4.
h) 1,2 × 10-2. Orden de magnitud 10-2.

13. El cálculo queda:

1 año - luz 9,4605 × 1012 km 1 segundo luz es :


9,4605 × 1012
299 990,4871 km
365 × 24 × 60 × 60
Distancia de la Tierra a la Luna es 384 403 km,es decir :
384 403 : 299 990, 4871 1,28138 segundos luz
redondeando a las centésimas es 1,28 segundos luz.
Distancia de la Tierra al Sol es 1,5 × 108 km,es decir :
1,5 × 108 : 17 999 429,22 8,333596 minutos luz
que redondeando a las centésimas es 8,33 minutos - luz.

14. Quedaría:

Primero redondea y luego suma :


1,33 0,48 25,01 122,55 82,57 7,63 239,57 euros.
Primero sumamos y después redondeamos a centésimas :
1,325 0,477 25,008 122,553 82,572 7,634 239,569 euros.
que redondeando a la centésima queda : 239,57 euros.

12
PÁGINA 30

13
SOLUCIONES

15. Las soluciones son:

a) 4,32657 × 10 -2 d) 167 040


b) 1,912307692 × 106 e) 75 466
c) 21 735 f) 1 500

16. Las soluciones son:

a) 5ab 2 c) 4a 2 b e) 3a 2
b) 2 x 2 d) 5 z 2 f) 2xy 2

17. Las expresiones quedan:

1 5 -3 -2
a) a 3
c) a 4
e) a 2
g) a 3

3
1 1
b) 4 d) 5 f) h)
125 3
a2

18. Las expresiones quedan:

a) 6
8 2 c) 8
37 e) 12
a

b) 3
a10 b 5 d) 4
a 24 b 4 a6b f) 12
a11

19. Las expresiones quedan:

a)10 10 c) 2a 3 a 2 e) 4a 2 b 3 ab g) 2 a 2 1

b) ab 3 a d) x 2 y 4 y f) yz 5 xz 2 h) a 1 b

20. Los radicales quedan:

a) 32 c) 4
36 e) 3
27a

b) 3
8a5 b 3 d) 2a5 b11 f) 3
64a5 b

14
21. La solución queda:

1 1
a) 3
23 2 c) 5
a a 5
e) 6
32 3 3

5 -2 1
b) a5 a 2
d) 3
a -2 a 3
f) a a 2

22. La solución queda:

98 37
a) 2 c) 2 e) 9 x
15 20
11 4
b) 3 d) 10 3 2 f) 2x 2 3 x
12

15
PÁGINA 31

16
SOLUCIONES

23. Quedan:

9
a) 2 2 d) 64 8 6 g) 120 j) 2
2
b) 9 e) 3 3 6 h) 4 k) 1
50
c) 4 4 2 f) 30 i) 6 12 6 l)
3

24. La solución queda:

a) 10
52 < 10 25 c) 18
3 2 < 18 26 < 18 59 e) 12
23 < 12 24 < 12 26

b) 12
62 < 12 43 d) 15
105 < 15 1003 f) 4
5-3 < 4 3-2

25. Tras operar obtenemos:

a) 12 8 640 000 e) 8
315

b) 3
a4 f) 24
16a11b17
b
c) 4 g) 6
35
2a
d) 1 h) 12 213 212 2

26. Tras racionalizar se obtiene:

5 5
162
a) 2 d) g)
10 3
6
73 3 2 6 3 087 6 3 2
b) e) h) 2 3
3 3 2
11 3 5 2 7
c) 9 4 5 f) i) 3 7
17

27. La solución queda:


2 15
a) 11 3 c)
15
34 23 2
b) d) 2 3
2

17
PÁGINA 32

18
SOLUCIONES

28. Queda:

4
8 38 4 3
a) c) 48 10 3 9
2
b) 92 d) 2

29. Tras simplificar obtenemos:

22 6 37 5 3 5
a) c)
5 2
49 20 7
b) 8 3 d)
21

30. Queda:

a) 6 d) 5
b) 4 e) 6
6 2
c) 516 53 f)
2

31. Elevamos los dos miembros al cuadrado:

2
4 2 3 4 2 3 22 4 2 3 2 4 2 3 4 2 3 4 2 3 4

8 2 4 4 8 4 4 Se verifica la igualdad.

32. Elevamos los dos miembros al cuadrado:

2
2
2 3 2 6 2 3 2 2 12 6
2 4 4 4

2 3 8 4 3
Se verifica la igualdad
4 16

19
33. Queda:

2 2 2 3 2 2 3
a) 2 2 3
2 3 2 3 2 3 4 3

5 3 2 5 3 2 5 3 2 1 2 3 2 4 3 15 6 5
b) 2
5 3 2 5 3 22 2 15 11

34. Queda:

2
5 2 9 4 5 8 5
1 1 160 72 5
5 2 9 4 5 9 4 5

35. Elevamos al cuadrado y agrupamos:

11 112 x y 2 xy 11 112 x y 4 xy
De donde se obtienen dos ecuaciones que forman el siguiente sistema :
x y 11 x 7 x 4
Las dos posibles soluciones serían y también
4 xy 112 y 4 y 7

36. Resolviendo:

2 2 2 2
1 6 1 6 7 2 6 7 2 6 7 2 6 7 2 6 56 6
2 2
= 2
1 6 1 6 7 2 6 7 2 6 72 2 6 25

20
Unidad 2 – Polinomios y fracciones algebraicas
PÁGINA 35

SOLUCIONES

1. Diremos que:

a) Resto 0; Cociente x 2 4 x 4
b) Resto 14; Cociente x 3 2 x 2 4 x 7

2. Utilizando el teorema del resto:


13
Resto A( 3) 5 ( 3) 3 a ( 3) 2 7( 3) 2 a
9

3. La descomposición queda:

a) x 3 5 x 2 6 x x ( x 2) ( x 3 )

b) x 4 16 ( x 2) ( x 2) ( x 2 4)

4. Operando obtenemos:

x2 1 x x 1 x
2
x 1 x 1 x 1 x 1

21
PÁGINA 49

SOLUCIONES

1. Sí puede ser cierto; se trata de dos padres que se han casado cada uno con la hija del otro.

2. Diremos que:

a1 7
a2 8

an 7 (n 1) 1 n 6
Además sabemos que an n 42 n 18 damas.
an 42 18 24 caballeros.
Había 18 damas y 24 caballeros.

3. Luis tarda 15 minutos en llegar a la sierra.


La perra, por lo tanto, ha estado moviéndose durante 15 minutos. Por tanto ha recorrido:
16 km :4 4 kilómetros.
h

4. Diremos que:

2 000 19 xy 9 x y
2 000 (1000 900 10 x y) 9 x y
11x 2y 91 x 7 y 7
Es decir, Astérix nació en el año 1 977 y en el año 2 000 tendrá 23 años.

22
PÁGINA 52

23
SOLUCIONES

1. Quedan del siguiente modo:

Mediante identidad de polinomios:


2
(x 3)(ax b ) x 3 2 x 2 3 x 6
ax 3 bx 2 3ax 3b x 3 2x 2 3 x 6
Identificando coeficientes obtenemos:
a 1, b 2 el polinomio A( x ) x 2
Mediante división:
x 3 2x 2 3 x 6
A( x ) x 2
x2 3

2. El cálculo queda:

a) ax 2 6 x b (2 x c )2

a 4

ax 2 6 x b 4 x 2 4cx c 2 b 9
4
c 3
2

b) ( x 2)(ax 2 bx c ) 2 x 3 9 x 2 14 x 8
a 2
a 2
3 2 2 3 2
b 2a 9
ax bx cx 2ax 2bx 2c 2 x 9x 14 x 8 b 5
c 2b 14
c 4
2c 8

3. Quedarían:

P(2) (3 2 2)2 16
Q( 2) (4 2 1)(4 2 1) 31
2
1 1
P 3 2 1
3 3
1 4 4
Q 1 1 3
2 2 2
R(0) 9

24
4. Operando y utilizando la identidad de polinomios:

a 1 b 2

5. Las descomposiciones pedidas son:

a) A(x) ( x 3)( x 3)( x 4)( x 4)


b) B(x) x ( x 1)2
c) C(x) ( x 1)2 ( x 1)
1 3
d)D(x) 8 ( x 1) x x
4 2
e)E(x) ( x 1)2 ( x 1)( x 3)

6. En cada uno de los casos descomponemos los polinomios en factores y calculamos el MCD y
el mcm.

a) A( x ) x (x 3) ( x 2) MCD [ A( x ), B( x )] (x 2)
B( x ) (x 3) ( x 2) mcm [ A( x ), B( x )] x (x 3) ( x 2) ( x 3)

b) C( x ) (x 3) ( x 2 x 2) MCD [C ( x ), D( x )] 1

D( x ) (x 2)2 ( x 1) mcm [C ( x ), D( x )] C ( x ) D( x )

c) E( x ) 2 (x2 x 1) ( x 2) MCD [E ( x ), F ( x )] 2 (x 2)
5
5 mcm [E ( x ), F ( x )] –2 ( x 2) ( x 2 x 1) x
F( x ) –2 ( x 2) x 2
2

25
7. Quedaría:
1
a) El resto de dividir P(x) por x debe ser cero.
2
3 2
1 1 1 1
Resto P 2 K 6 0 K 27
2 2 2 2

b) Ha de ser divisible por ( x 2) y por ( x 2) . Por tanto:

A(2) 0 8 24 2a b 0 a 4
A( 2) 0 8 24 2a b 0 b 24

c) Queda:

5 3
Resto 3 4 3 m 3 5 3 m 2

d) Queda:

2
1 1 1 1
Resto C 2 2
2 2 2 2

e) Queda:

Para que sea divisible por (x – 2 ) B(2) 0


8 4a 2b c 0 a 3
Para que sea divisible por (x+1) B( 1) 0 1 a–b c 0 b 0
c 4 c 4
Para que dé resto 4 al dividir por x B(0) 4

26
PÁGINA 53

27
SOLUCIONES

8. Queda en cada caso:

5x 2
15 x 5x ( x 3) x 3
a)
10 x 3
15 x 2
5 x (2 x 2
3) 2x 2
3x

2x 4 2 (x 2) 2
b)
x 2
4x 4 (x 2) 2
x 2

6 x x 2
(x 2) ( x 3) ( 1) x 3
c)
x 2
2x 8 (x 2) ( x 4) x 4

x 3
x 2
8 x 12 (x 2) ( x
2
3) (x 2) ( x 3) x 2
x 6
d)
x 3
6x 2
2 x 12 (x 2) ( x 2
4x 6) x 2
4x 6 x 2
4x 6

9. Queda en cada caso:

5x 3 5x 2
7x 9
a)
x 3 x 2 (x 3 ) (x 2)

2x 1 2 3
b)
x 2
4 x 2 x 2
4

7x 5 6x 7x 2
22 x 15
c)
x 3 x 3 x 2
9 x 2
9

2x 6 5x 5 2 (x 3) 5 ( x 1) 5
d)
x 2
1 4 x 12 ( x 1) (x 1) 4 ( x 3) 2x 2

x 1 x2 1 ( x 1) ( 3) ( x 3) 3
e) :
2x 6 3x 9 2 (x 3 ) (x 1) (x 1) 2x 2

x 2
6 x 5 2 x 2 8 2 x 10 ( x 1) (x 5 ) 2 (x 2 ) (x 2 ) x (x 3)
f) : x 2
x 2
5x 6 x x x 3x 2 2
(x 2 ) (x 3 ) x (x 1) 2 (x 5)

1 x 1 x 2
1 x 1
g) x : : x 1
x x x x

28
x x x 2
x 2
2x x ( x 1)
2
x
h) x : x :
x 1 x 1 x 1 x 1 ( x 1) x ( x 2) x 2

x 3 x 3
9x x 2
9 x 3
9x (x 3 ) (x 3) x ( x 2
9)
i)
3 x x 3 3x x 3 3 x (x 3)
(x 3 ) (x 2
9) x 3
3x 2
9x 27
3 3

x 1 1 1 x 1 2 2 (x 1) 1
j)
2x x 1 x 1 2x ( x 1) (x 1) 2 x ( x 1) (x 1) x 2
x

10. La descomposición en cada caso queda:

5x 2 5x 2 2 1
a)
3x 2
x x (3 x 1) x 3x 1

2 x 10 2 x 10 2 2x 2
b) 3
x 2x 2
3x 6 (x 2 ) (x 2
3) x 2 x 2
3

1 3
2x 1 2x 1 2 2
c)
x 2
2x 8 (x 2 ) (x 4) x 2 x 4

4x 2
5 4x 2
5 3 x 1
d)
x 3
x 2
2x 2 ( x 1) (x 2
2) x 1 x 2
2

2x 2
10 x 20 2x 2
10 x 20 3 2 1
e)
x 3
2x 2
4x 8 (x 2 ) (x 2) 2
x 2 (x 2) 2
x 2

2x 3
x 2
2x 2 (2 x 1) (x 2
x) x 2 x 2 2 1
f) (2 x 1) 2x 1
x 2
x x 2
x x ( x 1) x x 1

2x 2
2x 4 2x 2
2x 4 1 1 2
g) 3
x 4x x (x 2 ) (x 2) x x 2 x 2

x 2
2 ( x 1) (x 1) 3 3
h) x 1
x 1 x 1 x 1

2 x x2 (x 2 ) (x 1) x 2 3
i) 2
1
x 2
2x 1 (x 1) x 1 x 1

29
11. Los valores en cada caso son:

2 Ax B 2 ( x 1) 2
( Ax B ) (x 2)
a)
x 2 ( x 1) 2
(x 2 ) (x 1) 2

2 ( x 1) 2
( Ax B ) (x 2) 3x 2
12 x 6

Por el principio de identidad de polinomios obtenemos: A 5 y B 2

2 A B 2 ( x 1) 2
A (x 2) B ( x 2) ( x 1)
b)
x 2 ( x+1) 2
x 1 (x 2) ( x 1) 2

2 ( x 1) 2
A (x 2) B ( x 2) ( x 1) 3x 2
12 x 6

Por el principio de identidad de polinomios obtenemos: A 3 y B 5

12. Queda:

El polinomio es : P( x ) ax 2 bx c

Imponiendo las condiciones del enunciado obtenemos :


P (0) 5 c 5 a 2
P ( 1) 0 a b c 0 b 3
P ( 2) 9 4a 2b c 9 c 5

El polinomio buscado es : 2 x 2 3 x 5.

30
PÁGINA 54

31
SOLUCIONES

13. Utilizando el binomio de Newton y operando obtenemos:

5
a) 3 x 243 405 x 270 x 2 90 x 3 15 x 4 x5
6
b) 1 2 x 1 12 x 60 x 2 160 x 3 240 x 4 192 x 5 64 x 6
7
c) 4 x 16 384 28 672 x 21504 x 2 8 960 x 3 2 240 x 4 336 x 5 28 x 6 x7
4
d) 2 2 68 48 2
5
1 61484 3 297 676
e) 3 3
3 27 243
5
f) 3 2x 2 32 x 10 240 x 8 720 x 6 1080 x 4 810 x 2 243
4
1 3 1 3 27 27 81
g) x x x2 x3 x4
2 4 16 8 32 32 256
5
1 1 5 10
h) 3x x x 2 30 x 3 135 x 4 243 x 5
3 243 27 3

14. En cada uno de los casos quedaría:

4
7 3 1 945
a) T5º 3x
4 x x
5
7 2 1 189
b) T6º 3x Coeficiente 189
5 x x3
3
7 4 1
c) T4º 3x 2835 x Exponente del término en x es 1
3 x

15. Desarrollando cada una de las potencias mediante la fórmula del binomio de Newton
obtenemos:

( x 3)4 ( x 3)4 ( x 4 12 x 3 54 x 2 108 x 81) ( x 4 12 x 3 54 x 2 108 x 81)


2 x 4 108 x 2 162

32
16. Ambos resultados quedan:

1 x 1 2x
1 x 1 x 2
(1 x ) ( 1 x ) (1 2 x ) (1 x ) 1
a)
x 2x 2
x (1 2 x ) x (1 x ) ( 1 x ) ( 1 2 x ) x ( x 1)
1 x 1 x

1 2x 5
2
x 2 x 2 5 (x 2) 1
b)
2 4x 5x 5x ( x 2) x
1
x 2 x 2

17. Los polinomios pedidos son:

2
a) P(x) 1 x 1 x 0 x 1 x4 x3 x2 x
3
b) P(x) 2 x 1 x 2 2 x 4 10 x 3 18 x 2 14 x 4
c) P(x) a x 1 x 2 x 3 x 4 a ( x 4 10 x 3 35 x 2 50 x 24)

18. Operamos y aplicamos el principio de identidad de polinomios:

A 3
5x 2 x 12 A x2 4 Bx C x 2 2
5x x 12 A B x Cx 4 A B 2
x3 4x x x2 4
C 1

19. Queda lo siguiente:

10 5 5 2
a) T6º 1 x 252 x
5
12 6 6 6 6 6 6
b) T7º 3 4x 924 3 4 x 2 759 049 216 x
6
3n
12 2
12 n n 12 24
c) Tn + 1 x x x 2

n n
3n
Por tanto : 24 18 n 4 es el término quinto.
2

33
20. Desarrollamos:

6 n
6 1 n 6 n 6 n
Tn +1 2x xn 6
xn 2 x 2n 6
2
n x n n
Por tanto, el término independiente cumplirá : 2n 6 0 n 3
El valor del término independiente es :
3
6 1 3
T4º 2x 160
3 x

21. Ambos resultados quedan:

x a x 2
a 2
2ax
x a x 2
a 2
x 2
a 2
2ax ( x a) 2
x a
a)
x a
2
4ax 4ax ( x a) ( x a) 2x 2a
1
x a (x a) 2

m 2 1 1 2m 1
1
m 1 m m 2
( m 1) 2
m 2
(2m 1) ( m 1) m
b)
m 1 2m 1 m 1 ( m 1) (2m 1) m ( m 1)
2 2

1 1
m 1 m m 1 m
1 1
m (m 2
1) m 3
m

34
Unidad 3 – Ecuaciones y sistemas de ecuaciones.
Inecuaciones
PÁGINA 57

SOLUCIONES

1. Operando obtenemos:

a) x 2 4 x 4 x 2 5 x 5 x 1
Esta igualdad sólo se verifica para x 1.

b) Esta igualdad se verifica para todos los valores de x.

2. En cada uno de los casos:

x
Son números x, y que verifican: x y x y. Es decir: y.
x 1
x
Todos los números x; con x 1 dan el mismo resultado al sumar y al multiplicar.
x 1
x y
En el caso de tres números, éstos quedarían de la forma: x; y ; con x y 1 y se
xy 1
obtienen de forma análoga al caso anterior.

35
3. Llamamos x a los partidos ganados. Como participan 12 equipos a doble vuelta se juegan 22
partidos. Por tanto:
2 x 1 22 x 36
2 x 22 x 36
x 14 partidos ganados.

4. Consideremos el siguiente esquema:

Imponiendo las condiciones del problema:

x y 24 x 10 km de Abejar a Buitrago
y z 32 y 14 km de Buitrago a Cidones
x z 28 z 18 km de Abejar a Cidones

36
PÁGINA 77

SOLUCIONES

1. Veamos si el producto de cuatro números enteros x 1 x x 1 x 2 es un cuadrado perfecto


menos una unidad.
( x 1) x ( x 1) ( x 2) x4 2x 3 x2 2x
Luego ( x 1) x ( x 1) ( x 2) (x2 x 1)2 1
2 2 4 3 2
(x x 1) x 2x x 2x 1

3 000 000
2. Ambos cohetes tardan 60 segundos en alcanzar Venus. Durante este tiempo la
50 000
señal, en sus idas y venidas ha recorrido:

300 000 60 18 000 000 km .

3. Planteamos lo siguiente:

71 7 termina en 7
2
7 49 termina en 9
3
7 343 termina en 3
4
7 2 401 termina en 1
1
7 16 807 termina en 7
Por tanto hay cuatro terminaciones distintas que se repiten cíclicamente; de modo que:

83 578 4
R 2 20 894

Es decir, 783578 termina en el mismo número que 72 , es decir, termina en 9.

37
PÁGINA 80

38
SOLUCIONES

1. Las soluciones son:


6 3
a) x b) x 3 c) x 4 d) x
5 5

2. Las soluciones son:

a) 2 x ( x 3) 3( x 1) 2 x 2 3 x 3 0 No tiene soluciones reales.

6
b) x 1 x2 x 6 0 x1 2; x2 3
x

c) ( x 2)( x 2) 2( x 5) 21 x 2 2 x 35 0 x1 7; x2 5

9 x
d) 2 x 2 6 x 27 0 x1 3; x2 9
x 3

e) ( x 2 5)( x 2 3) 1 x 4 8 x 2 16 0 x1 2; x2 2

f) x 4 13 x 2 36 0 x1 2; x2 2; x3 3; x4 3

g) x 2 (a b )x ab 0 x1 a; x2 b

h) x 2 2ax (a 2 b 2 ) 0 x1 b a ; x2 a b

39
3. Las soluciones quedan:

a) Si una de las raíces de la ecuación es 8, ésta verificará la misma; es decir:


82 8m 24 0 m 5
b) Si las raíces de la ecuación son 2 y 3, éstas deben verificar la ecuación, por lo tanto:

4 2a b 0 a 1
9 3a b 0 b 6

c) Las dos raíces son iguales si el valor del discriminante es nulo, es decir:

b 2 4ac 0 b 2 4 2 50 0 b 20

d) Sean x1 y x2 las raíces de la ecuación. Sabemos que:

x1 x2 K 5
x2 1 Si x1 2; x2 1 K 2
x1 x2 2 x1 2 Si x1 2; x2 1 K 8
x1 2 x2

4. Las soluciones quedan:

a) x 3 2 x 2 5 x 6 0 ( x 1)( x 2)( x 3) 0
Las soluciones son : x1 1; x2 2; x3 3

b) x4 x 3 4x 2 4x 0 x( x 1)( x 2 4) 0
Las soluciones reales son : x1 0; x2 1

1 1
c) 4 x 4 65 x 2 16 0. Las soluciones de la ecuación son : x1 ; x2 ; x3 4; x4 4
2 2

2 2
d) 9 x 4 5 x 2 4 0. Las soluciones reales de la ecuación son : x1 ; x2
3 3

e) x 6 19 x 3 216 0. Las soluciones reales de la ecuación son : x1 2; x2 3

2
f) x 2 3 x 1 2
x 4 6 x 3 10 x 2 3 x 2 0
x 3x
Factorizando la ecuación obtenemos : ( x 1)( x 2)( x 2 3 x 1) 0
3 13 3 13
Las soluciones son : x1 1; x2 2; x3 ; x4
2 2

40
5. Las soluciones quedan:

a) Elevando al cuadrado ambos miembros y operando obtenemos: x 2 9 0 ; así las soluciones


quedarían: x1 3; x2 3

b) Elevando al cuadrado ambos miembros y operando obtenemos: 3 x 2 x 2 0 ; así las


2 2
soluciones quedarían: x1 1; x2 . La solución que verifica la ecuación dada es x .
3 3

c) Operando de forma análoga a los casos anteriores obtenemos:

x 4 43 x 2 432 0 x1 3 3 ; x2 3 3 ; x3 4; x4 4

Las soluciones que verifican la ecuación dada son: x1 4; x2 4

d) Operando de forma análoga a los casos anteriores obtenemos:

3
4 x 2 21x 18 0 x1 6; x2 donde la solución buscada es : x1 6
4

e) Elevando al cuadrado ambos miembros y operando obtenemos: 1 2 x 4 y elevando de


5
nuevo se obtiene x , sin embargo esta solución no verifica la ecuación inicial, por lo que
2
se concluye que no existe solución.

f) Elevando al cuadrado y operando:

3
x 3 x 6 3 x 3 ( x 6)( x 3) x2 9 x 18 x
x 3
Elevando al cuadrado se obtiene : 9 x 18 0 x 2

6. Las condiciones del problema nos dan:

1081 2 x
x x

De donde se extrae: 1081 2 x 2 x cuyas soluciones son : x1 23; x2 23,5 .


El divisor de esta división es 46 ó 47.

41
7. El triángulo tiene por catetos x, x 5 y por hipotenusa 25, por lo tanto:

x 2 ( x 5)2 252 x 2 5 x 300 0 x 20 cm

Un cateto mide 20 cm y el otro 15 cm.

8. Llamando x al número e imponiendo las condiciones del enunciado obtenemos:

1 34 5 3
x 15 x 2 34 x 15 0 Las soluciones son : x1 ; x2
x 15 3 5

9. La expresión sería: ( x 1)2 x 2 ( x 1)2 365 x 11


Los números son: 10, 11 y 12.
Los números consecutivos a éstos son: 13 y 14, y se cumple también que 132 142 365.

42
PÁGINA 81

43
SOLUCIONES

10. Llamamos x al número de estudiantes del curso e y a la cantidad de dinero que paga cada uno.
Imponiendo las condiciones del enunciado obtenemos:

x y 1 620 x 27 estudiantes
( x 2)( y 4,8) 1 620 y 60 euros paga cada uno

11. Los sistemas resueltos quedan:

2 3
a) 7 Por reducción obtenemos:
x y
1
3 2 11 x 2 ; y
2
x y 2

b) y x2 3 Por sustitución obtenemos:


x y 3
x 2 x 3
;
y 1 y 6

c) 2y –x =4
x2 y 2 5 Por sustitución obtenemos:
2
x
x 2 3
ó
y 3 7
y
3

d) x 2 y 2 9 Por sustitución obtenemos:


x y 20
x 5 x 5
ó
y 4 y 4

Por sustitución obtenemos:


e) x 2 y 2 117
x 9 x 9
x y 54 ó ó
y 6 y 6
x 6 x 6
ó
y 9 y 9

44
f) x y 41
Por sustitución obtenemos:
x y 9
x 25 x 16
ó
y 16 y 25

g) x 2 xy y 2 57 Sumando ambas ecuaciones obtenemos x 2 y 2 50


x 2 xy y 2 43 Restando ambas ecuaciones obtenemos xy 7

Resolviendo este sistema por sustitución obtenemos:

x 7 y 1 x 7 y 1 x 1 y 7 x 1 y 7

12. Llamando x a la longitud de la altura, la base tendrá por longitud (7 x ) . Conocida el área se
verifica:
x (7 x ) 60 x 5 cm

El rectángulo mide 5 cm de altura y 12 cm de base.

13. Llamando x a la longitud de la base e y a la altura e imponiendo las condiciones del problema
obtenemos:

2x 2y 20 x 6 cm x 4 cm
o bien
x y 24 y 4 cm y 6 cm

14. Llamando x al área de un cuadrado e y al área del otro obtenemos:

x y 3 250 x 2 025 m2
x y 800 y 1225 m2

De donde el lado de un cuadrado mide 35 m y el del otro 45 m.

15. Llamando x al tiempo que tarda él solo en hacer el trabajo obtenemos:

1 1 1
x 12 horas tardaría él solo.
4 x 3

45
16. Las soluciones quedan:

a) x y z 3 x 0
x y 2 y 2
y z 3 z 1

b) x y 2z 7 x 1
2 x y 5z 10 y 2
x y 4z 9 z 2

c) x 3y 2z 1 x 1
x z 2 y 2
2x 5y 8 z 3

d) x y z 2 x 1
2 x 3 y 5z 11 y 2
x 5 y 6z 29 z 3

e) x 4 y 8 z 8 x 0
4 x 8 y z 14 y 2
8 x y 4z 10 z 2

f) 3 x 4 y z 3 x 1
6 x 6 y 2z 16 y 1
x y 2z 6 z 2

g) x y 1 x 3 m
y z 1 y 2 m
z 1 m
z t 1 t m
t x 3 m R

h) x y t 6 x 1
x z t 1 y 2
y z t 6 z 1
x y z 0 t 3 46
i ) x 2y 3z 5
x 1
2x y z 3
y 1
x y 3z 6
z 2
3 x y 2z 0

17. Sea el número xyz.


De las siguientes condiciones del enunciado obtenemos el siguiente sistema:

x y z 7 x y z 7
x y 2z x y 2z 0
xyz zyx 297 (100 x 10 y z ) (100z 10 y x ) 297

Resolviendo el sistema obtenemos: x 4, y 2, z 1


El número buscado es el 421.

47
PÁGINA 82

48
SOLUCIONES

18. Llamando x a la edad del padre e y a la edad del hijo obtenemos:

x x
y
3 2 x 6y 0 x 48
x 4 11 (y – 4) x 11y 40 y 8

El padre tiene 48 años y el hijo 8 años.

19. Sea el número xyz.


Imponiendo las condiciones del enunciado obtenemos:

x y z 18 x y z 18 x 9
(100 x 10 y z ) (100 z 10 y x) 594 x z 6 y 6
x z x 2y z 0 z 3
y
2

El número es el 963.

20. Llamamos x a la edad del padre, y a la edad de la madre y z a la edad de la hija. Obtenemos:

x y z 86 x 38
y 3z y 36
x z 26 z 12

El padre tiene 38 años, la madre 36 años y la hija 12 años.

21. Las soluciones quedan:

2 2
a) 128 x 1
2x x 2
27( x 1)
2x x 2
x 2 8x 9 0 x1 9; x2 1
x x 1 x 2 x x x x
b) 2 2 2 7 2 2 2 4 2 7 2 1 x 0
c) 9 x 2 3 x 2
81 0 32 x 18 3 x 81 0 x 2
x 1 x 3 2x x
d) 4 2 320 0 4 2 8 2 320 0 x 3
75
e) 5 x 1
10 3 52 x
5 5x 10 5 52 x 10 5 x 75 0 x 1
5x
6
f) 61 x
6x 7 6x 7 62 x 7 6 x 6 0 x1 0; x2 1
6x
g) 3 x 9 x 93 33 x 3 6 x 2

49
x 9
h) 2 83 x
2 x
29 3x
x
2
4
ln
4 7
i) 2x 2x 1
2x 2
1 4 2x 2 2x 2x 4 2x x
7 ln 2

22. Las soluciones quedan:

5
a) log2 32 x 2x 32 x
2

b) ln e5 x 5 ln e x x 5

1 1 1
c) logx 3 x3 x
8 8 2

1
d) log x 1 x
10

log3 log3
e) x log 3 x 2
3 log 3 1
log3
2

f) logx 0,0001 4 x4 0,0001 x 0,1

1
g) ln x x e
2
2
1
h) log 1 x 2 x x 4
2
2

23. Las soluciones quedan:

a) log x 1 log(22 x ) log x log10(22 x ) x 20

3x 1 10
b) log(3 x 1) log(2 x 3) log 25 1 log log x 1
2x 3 25

2
5x 4 36
c) 2log(5 x 4) log 4 log( x 4) log log( x 4) x1 0; x2
4 25

log 5 x 2 1
d) 2 log 5 x 2 log(2 x )2 x
log(2 x ) 4

50
2 5x 9 2 5x 9
e) ( x 2 5 x 9)log2 log125 3 log(125 2 x ) log1000 2x 8
2
x 5x 6 0 x1 2; x2 3

x2
f) 2log2 x log2 ( x 16) log2 64 log2 log2 64 x1 32
x 16

24. Las soluciones de los sistemas quedan:

a) 2x 5y 9 2x 5y 9 2x 4 x 2
x+2 y +1 x y y
2 5 9 4 2 5 5 9 5 5 y 1

b) 2 x 2y 24 2x 2y 24
x+ y x+ y
x 4; y 3 ó x 3; y 4
2 128 2 27

e11
c) ex x 11 y x 8
ey
(x y ) (x y ) 55 y 3
log (x y ) log (x–y ) log 55

d) log3 x log3 y 0 x y 1
x 2,62; y 0,38 ó x 0,38; y 2,62
x y 3 x y 3

e) logx ( y 18) 2
y 18 x 2 3 81
1 x ; y
logy ( x 3) x 3 y 2 4
2

f) log2 x 3log2 y 5 log2 x 3log2 y 5 log2 x 2 x 4


2
log2 x log2 y 3 2log2 x log2 y 3 log2 y 1 y 2

51
25. Llamando:

x: número de bricks de leche entera


y: número de bricks de leche semidesnatada
z: número de bricks de leche desnatada

Imponiendo las condiciones del problema obtenemos el siguiente sistema:

x y z 10 400 x 3 900 bricks de leche entera


0,6 x 0,55 y 0,5 z 5 765 y 3 500 bricks de leche semidesnatada
x 0,6 ( y z ) z 3 000 bricks de leche desnatada

52
PÁGINA 83

53
SOLUCIONES

26. Las soluciones son:

a) ,2 d) , 3 3, g ) 0,1
b) 0,10 e) 3 h) ,4
1 1
c) , f) , i) 1,4
2 4

27. Operando con cada una de las inecuaciones de estos sistemas obtenemos los siguientes
intervalos como solución:

a) 8,3 b) 0,7

28. Por ensayo y error dirigido obtenemos que el número es 142 857. También se puede hacer
mediante ecuaciones:

Número 100 000 x 3(100 000 x ) x 10 1


x 42857 Número 142857

29. Llamamos x al tiempo que invertiría la tercera ella sola. Obtenemos:

1 1 1 1
x 15 días tarda la 3ª
12 10 x 4

30. Llamamos x a la longitud de la arista de la caja, obtenemos:

x 3 71 ( x 1)3 200 x 9 cm

Las aristas de las cajas son 9 cm y 10 cm, y hay 800 cubitos de 1 cm3.

31. En el equipo A hay x futbolistas y en el equipo B hay y futbolistas.

x 3 y 3 x 18 futbolistas en el equipo A

x 7 (y 7)2 y 12 futbolistas en el equipo B

54
32. Cada uno de los apartados queda:

a) Resolviendo la ecuación obtenemos :

(m 1) (m 1)2 4 2 (m 3)
x
4
Imponiendo la condición del enunciado:

( m 1) ( m 1)2 8 ( m 3) ( m 1) ( m 1)2 8 ( m 3)
1
4 4

( m 1)2 8 ( m 3) 2 m2 6m 27 0 m 9 ó m 3

b) Llamamos y, z a las soluciones de la ecuación. Obtenemos:

14
y z m 2
m
z 1
12
y z
m y 6
y 6z

1
c) Si una solución es x1 , ésta verifica la ecuación, por tanto:
4

2
1 1 17
m 1 0 m La otra solución es 4.
4 4 4

d) Resolviendo la ecuación x 2 mx 4 0 obtenemos:

m m2 16
x
2

Las dos raíces son iguales si: m 2 16 0 m 4

33. Quedaría del siguiente modo:

1
log
1 log a b log a log b
log 1 a + logb 2
b 1 log b log a log b
a
log
a

55
PÁGINA 84

56
SOLUCIONES

34. Las soluciones quedan:

a) x 2 2 x 2 2 12 x 4 16 x 2

b) Elevando al cuadrado ambos miembros y operando obtenemos: x 2 2 2 x 2 3 , y


elevando de nuevo obtendríamos: x 4 8 x 2 16 0 x 2 y ambas soluciones son
válidas.

c) Factorizando obtenemos: x 2 x 1 x 1 2 x 3 0 y sus soluciones serán las siguientes:


3
x 0 doble ; x 1; x 1; x .
2

d) Operando obtenemos: x 4 2 x 2 3 0 cuyas soluciones son: x 3; x 3.

35. Los sistemas son:

a) Las soluciones son: x 3 e y 1 ó x 2 e y 4

b) Las soluciones son: x 20; y 30; z 50

c) Las soluciones son: x 3; y 1; z 3

2
d) Sumando ambas ecuaciones obtenemos: x y 36 x y 6 ó x y 6 y la
solución provendrá de la resolución de los dos sistemas siguientes:

x y 6 x 5
2
x xy 30 y 1
x y 6 x 5
2
x xy 30 y 1

57
36. Las soluciones quedan:

a) La solución sería: x 2

b) Haciendo 3 x z obtenemos la ecuación: 9z 2 8z 55 0 cuyas soluciones quedan como


z 2,96 y z 2,07 ; por tanto:

log2,96
3x 2,96 x 0,99
log3

c) Operando obtenemos:

3
log 5 x 3 3 log 1 x 5 x3 1 x 3x 2 3x 4 0
x 1,76 solución no válida y x 0,76 que es la solución válida.

d) Obtenemos la ecuación: x 3 2 x 2 x 2 0 cuyas soluciones no verifican la ecuación


original. Diremos, por tanto, que carece de soluciones.

37. Las soluciones quedan:

3x 1 1
a) Operando obtenemos la inecuación: < 0 cuya solución es el intervalo ,1
x 1 3
b) Operando la ecuación: x 3 11x 2 10 x 0 cuya solución se define por ,0 1,10

38. Llamando x, y a las dimensiones del jardín e imponiendo las condiciones del problema
obtenemos el siguiente sistema:

2 x 2y 36
x 2 y 2 xy 40

Este sistema tiene indefinidas soluciones, todos los valores de x e y que verifiquen la siguiente
expresión: x y 18 con x 0,18 e y 0,18 .

39. Llamando x, y, z a los números e imponiendo las condiciones del problema obtenemos:

x y z 5x x 9
x y z 2y 9 y 18
x y z 2z 9 z 18

58
40. Cuando la edad del hijo esté en el intervalo 0,3 .

41. Llamando x al número de móviles vendidos obtenemos la ecuación: 5 x 1500 10 x 1200


cuya solución es x 60 . Luego la solución es el conjunto de números enteros de x
comprendidos en el intervalo 0,60 .

42. Llamamos x al número de kilómetros hacia arriba a la ida, y al número de kilómetros hechos en
llano y z al número de kilómetros hacia abajo. Imponiendo las condiciones del problema
obtenemos:

x y z 920
x y z x 240 km
9
80 100 120 y 200 km
x y z z 480 km
10
120 100 80

43. Las soluciones quedan:

a) x 2 8 1 x 3 y x 7
b) 2 x 3 x 9 x 12 o bien 2 x 3 x 9 x 2

59
Unidad 4 – Trigonometría I
PÁGINA 87

SOLUCIONES

1. Sabemos que cos 0,2 y que 90º 180º . Utilizando la fórmula sen2 cos2 1
hallamos sen 0,98 . Por otro lado quedaría:

sen
tg 4,9
cos

2. La discusión quedaría:

a) Falsa pues sen 1,1

b) Verdadera pues tg ,
c) Verdadera pues cos 720º cos 360º 1.

60
3. Según el esquema:

360º:5 72º

En el triángulo rayado calculemos el valor de la apotema del pentágono.

6
tg36 a 8,26 cm
a
5 12 8,26
Área Área 247,8 cm2
2

4. Según el esquema:

De los dos triángulos rectángulos de la figura obtenemos:

h 1500 tg 40º tg 25º


tg 40º h
x tg 40º tg 25º
h
tg 25º
1500 x h 449,61m

61
PÁGINA 103

SOLUCIONES

1. Hay que buscar un número que sea a la vez triangular y cuadrado.

n2 n
Números triangulares : 1,3, 4,10,15,21,...,
2
Números cuadrados : 1,4,9,16,25,..., n 2
n2 n 82 8
x2 esto se cumple para n 8, pues x2 x 2 36.
2 2
Como dice que hay más de 36 cajas, hay que buscar otra solución, y ésta es :
492 49
n 49, pues 352 1225 Luego x 2 1225 cajas tiene.
2

62
2. Observamos que:
1 1 1
con n 2.
n 1n n 1 n

Luego :
1 1 1
12 1 2
1 1 1
2 3 2 3
1 1 1
3 4 3 4
... ... ...
1 1 1
998 999 998 999
1 1 1
999 1000 999 1000

Sumando :
1 1 1 1 1
... 0,999
12 2 3 3 4 998 999 999 1000

3. Sean A, B, C, las tres rebanadas. Con A1 indicamos que se tuesta la cara 1 y con A2 indicamos
que se tuesta la cara 2.

1.º A1B1 tarda : 30 s : tostar cara A1 y B1


5 s :colocar A1
5 s :colocar B1
5 s :sacar B1
2.º A2C1 tarda : 3 s :dar la vuelta A1
5 s :meter C1
30 s :tostar cara A2 y C1
3 s :dar la vuelta C2
3.º B2C2 tarda : 5 s :sacar A2
5 s :meter B2
30 s :tostar cara B2 y C2
5 s :sacar B2
5 s :sacar C2

En total se necesitan: 136 s en tostar las 3 rebanadas.

63
PÁGINA 106

64
SOLUCIONES

1. La tabla queda:

90º 45º 120º 270º 225º


2 3 5
rad rad rad rad rad
2 4 3 2 4
240º 39º 42' 57º 20' 135º 22' 42" 143º 19'
4
rad 0,22 rad 1rad 0,75 rad 2,5rad
3

2. La resolución de los triángulos queda:

I ) Cˆ 50º; b 130 sen 40º 83,56 m;


c 130 cos 40º 99,59m

95
II) Cˆ 35º ; c 66,52 m;
tg 55º
95
a 115,97 m
sen 55º

40
III) cos Cˆ Cˆ 52º 1' ; Bˆ 37º 59 ';
65
c 652 402 51,23 m

IV) a 1402 1252 187,68 m;


140
tg Cˆ Cˆ 48º 14 ' 23 "
125
Bˆ 41º 45 ' 37 "

65
3. Sea la figura:

Queda:
h
tg 42º
x 1
tg tg 42º 0,45; 24º 14 ' 15 "
h 2
tg
2x

Si nos colocamos a distancia triple se verificará:

h 1
tg tg 42º 0,3 16º 42' 23 "
3x 3

4. Sea la figura:

Queda:
h
tg 45º
x 30 tg 45º tg 30º
h h 40,98 m
h tg 45º tg 30º
tg 30º
30 x

66
5. Sea la figura:

Queda:
60
tg 75º 4 ' 7 "
16
90 14º 55 ' 53 "

Los ángulos del trapecio miden 75º 4 ' 7 " los dos agudos y 104º 55 ' 33 " cada uno de los dos
obtusos.

6. Sea la figura:

Llamamos l al lado del pentágono. De la figura obtenemos:

l
2
sen 36º l 30 sen 36º l 17,63 cm
15
Perímetro 5 17,63 88,15 cm

7. Sea la figura:

Llamamos al ángulo que forman las ramas del triángulo rectángulo de la figura. Obtenemos:
4
sen 2 30º 55 ' 55 "
15

67
8. Quedan:
119 119
sen 0,91; tg 2,18
12 5

9. Quedan:
1 3 4
cos2 x cos x y sen x
1 tg2 x 5 5

10. Quedan:
3
Si tg A 0 0 A ó A
2 2
3 3
Como sen A A
5 2
4 3
sen2 A cos2 A 1 cos A y tg A
5 4

68
PÁGINA 107

69
SOLUCIONES

11. Las simplificaciones quedan:

1 tg2 1 tg2
a) tg2
1 cotg2 1
1
tg2

cos2 1 sen2
b) 1 sen
1 sen 1 sen

1
2 2
cos2
cos (1 tg ) cos2
c) tg
cotg 1
tg

d) sen cos (tg cotg ) 1

e) sen3 sen cos2 sen

f) cos3 sen3 cos2 sen cos sen2 (sen cos )(cos2 sen2 ) sen cos

sen2 1 cos2
g) 1 cos
1 cos 1 cos

1
sec cos
h) cos
1 tg2 1
cos2

i) tg2 tg2 sen2 tg2 (1 sen2 ) tg2 cos2 sen2

j) sen4 cos4 (sen2 cos2 ) (sen2 cos2 ) sen2 cos2

70
12. Queda:

3
a) sen 120º
2

2
b) sen 1215º sen135º sen 45º
2

3
c) cos 210º cos 30º
2

d) tg ( 60º ) tg 60º 3

e) tg 300º tg 60º 3

23 2
f) sec sec 330º
6 3

g) cotg 225º cotg 45º 1

29
h) cosec cosec 225º 2
4

13. Los cálculos quedan:

a) sen (180º ) sen 0,6

4
b) tg (90º ) cotg
3

c) cos (180º ) cos 0,8

d) sen (270º ) cos 0,8

e) cos (90º ) sen 0,6

4
f) cotg (360º ) cotg
3

71
14. Se comprueba del siguiente modo:

tg a tg b tg a tg b (tg a tg b ) tg a tg b
a) tg a tg b
cotg a cotg b 1 1 tg a tg b
tg a tg b

b) sen4 x sen2 x sen2 x (sen2 x 1) (1 cos2 x ) ( cos2 x ) cos4 x cos2 x

1 sen cos
c) 1 sen2 cos2
cos 1 sen

sen2 x
d) tg2 x tg2 x sen2 x tg2 x (1 sen2 x ) 2
cos2 x sen2 x
cos x

Todas las igualdades son verdaderas.

15. Sea la representación del problema:

Por Pitágoras obtenemos:


852 x2 (50 h )2
h 5m
652 h2 x2

También podemos calcular el ángulo por el teorema del coseno:

85 2 50 2 65 2 2 65 50 cos 94º 24 ' 42 "

h
Por tanto 180º 85º 35 ' 18 " cos h 65 cos 5m
65

72
16. Los triángulos se resuelven del siguiente modo:

a) 502 602 c 2 2 60 c cos 42º c 74,39m ó c 14,79m

502 602 74,392


Si c 74,39m cos C C 84º 35 ' 9 " y B 53º 24 ' 51"
2 50 60

502 602 14,792


Si c 14,79 m cos C C 11º 25 ' 5 " y B 126º 34 ' 55 "
2 50 60

b) 6 2 10 2 a2 2 10 a cos 45º a no es un número real. Este triángulo no tiene solución.

c) c 2 92 102 2 9 10 cos 70º c 10,93 m


102 92 10,932 2 9 10,93 cos A A 59º 17 ' 35 " y B 50º 42 ' 25 "

122 202 82
d) cos A A 0º
2 12 20

Imposible. Además un lado es igual a la suma de los otros dos, por tanto no existe este
triángulo.

e) 82 42 c2 2 4 c cos 40º c 10,64 m


2
8 10,642 42
cos A= A 18º 44 ' 44 " y C 121º 15 ' 16 "
2 8 10,64

f) B 60º.

Utilizando el teorema del seno obtenemos :


a 10
a 8,16 m
sen 45º sen 60º
c 10
c 11,15 m
sen 75º sen 60º

73
PÁGINA 108

74
SOLUCIONES

17. Un esquema del problema sería:

El ángulo C 75º . Utilizando el teorema del seno obtenemos:

a 80
a 71,73 m
sen 60º sen 75º
b 80
b 58,56 m
sen 45º sen 75º

Las distancias pedidas son 71,13 m y 58,56 m.

18. Un esquema del problema es el siguiente:

El ángulo C 24º .
Determinamos la distancia BC (lado a) mediante el teorema del seno:

1500 BC
BC 2 652,85 m
sen 24º sen 46º

75
19. La figura queda:

Mediante el teorema del coseno:

c2 400 2 5202 2 400 520 cos 40º c 334,25 m AB

20. Como el decágono está circunscrito a la circunferencia, el radio de ésta es la apotema del
polígono. El ángulo central del polígono es 36º.
Obtenemos el lado del triángulo de la figura:

El cálculo queda:

l 10 6,5 10
tg 18º 2
l 6,5 cm Área 325 cm2
10 2

21. la figura es:

En el triángulo rayado aplicamos el teorema del coseno y obtenemos la base mayor B.

152 B2 52 2 B 5 cos 60º B 16,86 cm

La altura h del triángulo mide: h 5 sen 60º 4,33 cm


La base menor mide: B 2 x B 2 5 cos 60º 11,86 cm
El área total queda:
B b 16,86 11,86
Área h 4,33 62,18 cm2
2 2

76
22. La figura queda:

Los cálculos son:

l2 82 102 2 8 10 cos 120º l 15,62 cm


a 2 82 102 2 8 10 cos 60º a 9,17 cm
Perímetro 2 15,62 2 9,17 49,58 cm

a 2 202 l2 2 20 l cos
2
9,17 202 15,622 2 20 15,62 cos 26º 20 ' 50 "
h h
sen sen 26º 20 ' 50 " h 8,88 cm
20 20
Área base altura = l h 15,62 8,88 Área 138,71cm2

23. La distancia d que los separa viene dada por:

d2 100 2 130 2 2 100 130 cos 60º d 117,9 km

24. Las soluciones en cada uno de los casos:

El radio de la circunferencia inscrita es la apotema


del dodecágono, por tanto:

3
tg 15º r 11,20 dm
r

El radio de la circunferencia circunscrita lo


calculamos en el triángulo:

3
sen 15º R 11,59 dm
R

77
25. La solución queda:

r 12 sen 30º 6 m r2 h 62 10,39


V 391,7 m3
h 12 cos 30º 10,39m 3 3

26. Según la figura:

Los cálculos quedan:

Sean P y Q los árboles.


En el triángulo ABP hallamos PB :

PB 120
PB 225,53 m
sen 110º sen 30º

En el triángulo ABQ hallamos BQ :

BQ 120
BQ 112,22 m
sen 50º sen 55º

En el triángulo PBQ hallamos x :

x2 PB 2 BQ 2 2 PB BQ cos 35º x 148,30 m

78
Unidad 5 – Trigonometría II
PÁGINA 111

SOLUCIONES

1. Las tres igualdades son falsas. Para probarlo basta con utilizar la calculadora.

2. Calculamos el área del octógono circunscrito y le restamos el área del octógono inscrito
obteniendo la superficie pedida.
El octógono circunscrito consta de ocho triángulos como el de la figura:
22,5º l
tg22,5º 2 l 8,28 cm
10
10
Área del octógono circunscrito 331,2cm2

El octógono inscrito consta de ocho triángulos como el de la figura:


l
22,5º sen 22,5º 2 l 7,65 cm
10
a 10 a
cos 22,5º a 9,24 cm
10
l Área del octógono inscrito 282,744 cm2

El área comprendida entre ambos será: 331,2 282,744 48, 456 cm2

79
3. Las soluciones quedan:

1 x 5º 360º K
a) sen ( x 25º )
2 x 125º 360º K

x 45º 360º K
b) sen x cos x
x 225º 360º K

c) tg (2x ) 3 x 30º 90º K

4. Supongamos conocidos los lados b y c y el ángulo A comprendido:

Calculamos la altura: h b sen A


El área será:
base altura c b sen A 1
Área Área b c sen A
2 2 2

80
PÁGINA 123

SOLUCIONES

1. Llamemos B a las vacas blancas y N a las vacas negras:

5 (4B 3N ) 4 (3B 5N ) 20B 15N 12B 20N 8B 5N

Dan más leche las vacas negras.

2. El número de naranjas en la pirámide es:

12 22 32 42 ... 142 152 1 240 naranjas.

81
PÁGINA 126

82
SOLUCIONES

1. Quedan:
12 5 12
sen a cos a y tg a
13 13 5
24 7 24
tg b cos b y sen b
7 25 25
36 323 36
sen (a b) ; cos (a b) ; tg (a b)
325 325 323

2. Los cálculos son los siguientes:

sen 90º sen ( 30º 60º ) 1

cos 90º cos ( 30º 60º ) 0

3
sen 120º sen ( 60º 60º )
2
1
cos 120º cos ( 60º 60º )
2
tg 120º tg ( 60º 60º ) 3

2 6
sen 105º sen ( 45º 60º )
4
2 6
cos 105º cos ( 45º 60º )
4
tg 105º tg ( 45º 60º ) 2 3

3. Quedan:
3 4
sen a cos a
5 5
3 3 4
sen (a 30º ) sen a cos 30º sen 30º cos a
10
3 4 3
cos (a 30º ) cos a cos 30º sen a sen 30º
10
sen (a 30º ) 48 25 3
tg (a 30º )
cos (a 30º ) 39

4. Para ello es suficiente con utilizar los teoremas de adición para el seno, coseno y tangente
estudiados en esta unidad didáctica.

83
5. Queda de la forma:

sen a sen( b c ) sen b sen(a c ) sen c sen(a b )


sen a sen b cos c sen a sen c cos b sen b sen a cos c sen b sen c cos a
sen c sen a cos b sen c sen b cos a 0

6. Queda de la forma:

cos(a b ) cos(a b ) (cos a cos b sen a sen b ) (cos a cos b sen a sen b )
cos2 a cos2 b sen2 a sen2 b

A partir de esta expresión obtenemos las dos igualdades:

cos2 a cos2 b sen2 a sen2 b cos2 a (1 sen2 b ) (1 cos2 a ) sen2 b cos2 a sen2 b
cos2 a cos2 b sen2 a sen2 b cos2 b (1 sen2 a ) (1 cos2 b ) sen2 a cos2 b sen2a

7. Queda de la forma:

sen(a b ) sen(a b ) (sen a cos b sen b cos a ) (sen a cos b sen b cos a )
sen2 a cos2 b sen2 b cos2 a

A partir de esta expresión obtenemos las dos igualdades:

sen2 a cos2 b sen2 b cos2 a sen2 a (1 sen2 b ) (1 sen2 a ) sen2 b sen2 a sen2 b
sen2 a cos2 b sen2 b cos2 a cos2 b (1 cos2 a ) (1 cos2 b ) cos2 a cos2 b cos2 a

8. Queda:

a) cos 3a cos (2a a ) 4 cos3 a 3 cos a


b) sen 4a sen 2 2a (4 sen a 8 sen3 a ) 1 sen2 a

También se puede resolver esta actividad mediante la fórmula de De Moivre.

9. Las razones trigonométricas quedan:

3 24 1
tg a 24 y a sen a ; cos a
2 5 5
2 24 23 2 tg a 2 24
sen 2a 2 sen a cos a ; cos 2a cos2 a sen2 a ; tg 2a
25 25 1 tg2 a 23

84
10. La tangente queda:

2 tg a 3
tg 2a 3 3 3 tg2 a 2 tg a 3 0 tg a o bien tg a 3
1 tg2 a 3

11. Las simplificaciones quedan:

sen 2a 2 sen a cos a


a) tg a
1 cos 2a 2 cos2 a

sen 2a 1 cos 2a sen 2a cos a


b) : 1
sen a cos a sen a (1 cos 2a )

12. Partimos del segundo miembro para llegar al primero:

1 2 1 2 (1 tg2 x ) 1 1 tg2 x
cotg x 2 cotg 2 x tg x
tg x tg 2 x tg x 2 tg x tg x

13. Partiendo del primer miembro obtenemos:

tg a tg a (1 tg2 a ) tg a
cos 2a
tg 2a tg a 2 tg a tg a (1 tg2 a )
tg a
1 tg2 a

14. Quedan:

45º 1 cos 45º 2 2


sen 22º 30 ' sen
2 2 2

45º 1 cos 45º 2 2


cos 22º 30 ' cos
2 2 2

45º 1 cos 45º 2 2


tg 22º 30 ' tg
2 1 cos 45º 2

150º 1 cos 150º 2 3


sen 75º sen
2 2 2

85
150º 1 cos 150º 2 3
cos 75º cos
2 2 2

150º 1 cos 150º


tg 75º tg 2 3
2 1 cos 150º

86
PÁGINA 127

87
SOLUCIONES

15. Quedaría:

3 a 1 2
cotg a 2 a 2 tg a ; cos a
2 2 2 2 5
a 1 cos a 5 2 a 1 cos a 5 2 a 1 cos a
sen ; cos ; tg 2 5
2 2 2 5 2 2 2 5 2 1 cos a

16. Queda expresado del siguiente modo:


a a a
1 cos2 sen2 2 sen2
1 cos a 2 2 2 a
a) tg
sen a a a a a 2
2 sen cos 2 sen cos
2 2 2 2

a a
2 2 sen cos cos2 a
sen 2a cos a 2 sen a cos a cos a 2 2 a
b) tg
1 cos 2a 1 cos a 1 cos a sen a 1 cos2 a sen2 a
2 2
a
2 cos a (2 cos2 )
2 2
2 2 2

17. Ambas razones trigonométricas quedan:

x 2 1 cos x 2 1 4 5
cos cos x ; sen x
2 3 2 3 9 9

18. Las simplificaciones quedan:

sen 40º sen 20º 2 sen 30º cos 10º 1


a) tg 30º
cos 40º cos 20º 2 cos 30º cos 10º 3

sen 195º sen 75º 2 cos 135º sen 60º


b) cotg 135º tg 60º 3
sen 195º sen 75º 2 sen 135º cos 60º

cos 60º cos 40º 2 sen 50º sen 10º


c) tg 50º 1,19
sen 60º sen 40º 2 cos 50º sen 10º

19. Se demuestra del siguiente modo:

cos( x y ) cos( x y ) 2 sen x sen ( y )


tg y
sen( x y ) sen( x y ) 2 sen x cos y

88
20. La solución queda:

5
a) x K ó x K
24 24

11 2K 19 2K
b) x ó x
36 3 36 3

K
c) x
6 2

2 8
d) x 4K ó x 4K
3 3

e) x 10º 120º K ó x 50º 120º K

f) x 15º 360º K

21. Las soluciones quedan:

a) sen 2 x 2 cos x 2sen x cos x 2cos x 2sen x cos x 2 cos x 0


cos x 0; sen x 1 x 90º 180º K

b) sen x sen 3 x cos x 2sen 2 x cos x cos x 2sen 2 x cos x cos x 0


1
cos x 0; sen 2 x x 90º 180º K x 15º 180º K x 75º 180º K
2

c) sen 4 x sen 2 x 2sen 2 x cos 2 x sen 2 x 2sen 2 x cos 2 x sen 2 x 0

1 x 0º 90º K x 30º 180º K


sen 2 x (2 cos 2 x 1) 0 cos 2 x ; sen 2 x 0
2 x 150º 180º K

d) cos 2 x cos 6 x sen 5 x sen 3 x 2 sen 4 x sen( 2 x ) 2sen 4 x cos x


2 sen 4 x (sen 2 x cos x ) 0 sen 4 x 0 x 45º K sen(2 x ) cos x 0

1 x 90º 180º K x 30º 360º K


cos x (2sen x 1) 0 cos x 0; sen x
2 x 150º 360º K

89
e) sen 2 x cos x 6sen3 x 2 sen x cos2 x 6sen3 x 0 2 sen x (cos2 x 3sen2 x ) 0
sen x 0 x 0º 180º K cos2 x 3sen2 x 0 cos2 x 3(1 cos2 x ) 0
3
cos x x 30º 180º K x 150º 180º K
2

1
f) 2sen x tg x sen x (2 cos x 1) 0 sen x 0 ; cos x
2
x 0º 180º K x 60º 360º K x 300º 360º K

22. La solución de cada ecuación queda:

a) sen x 1 2 cos2 x sen x 1 2 2 sen2 x 2 sen2 x sen x 3 0


sen x 1 x 90º 360º K

1 sen x
b) sec x tg x 0 0
cos x cos x
sen x 1 x 270º 360º K

2
2x 1 cos x
c) 6 cos cos x 1 6 cos x 1 3 3 cos x cos x 1
2 2
1
cos x x 120º 360º K x 240º 360º K
2

d) 6 cos2 x 6sen2 x 5 sen x 6 5 sen x


sen x 1 x 90º 360º K

2 tg2 x
e) tg 2 x tg x 1 1
1 tg 2 x

1 x 30º 180º K
tg x
3 x 150º 180º K

f) cos2 x 3sen2 x 1 4 sen2 x

1 x 30º 180º K
sen x
2 x 150º 180º K

90
23. La solución queda:
1 3
a) sen x 3 cos x 2 sen x cos x 1
2 2
sen( x 60º ) 1 x 30º 360º K

1 2
b) sen x cos x 2 sen x cos x 1
2 2
sen( x 45º ) 1 x 45º 360º K

5
c) sen x cos x Imposible sen x cos x 2
2

24. Las soluciones de los sistemas quedan:

a) x sen2 y 2 Sumando ambas 2x 1 3 x 1


x cos y2
1 ecuaciones:
3
sen2 y 1 sen y 1 y y
2 2

3
b) sen x cos y
4 Sumando ambas sen ( x y) 1 x y 90º x 90º y
1 ecuaciones obtenemos:
cos x sen y
4
3 3 3
Sustituyendo en la 1ª ecuación: sen (90 y ) cos y cos2 y cos y
4 4 2
y 30 ; x 60 y 150 ; x 300 y 210 ; x 240 y 330 ; x 120

c) cos x cos y 1 De la 2ª ecuación x y 0º x y


cos ( x y ) 1 obtenemos:
Sustituyendo en la 1ª ecuación: cos ( y ) cos y 1 2 cos y 1
1 y 60º y 300º
cos y
2 x 300º x 60º

d) x y 90º
y 90º x
6 Sustituyendo en la 2ª ecuación
sen x sen y
2
6 6
sen x sen(90º x ) 2 sen 45º cos (90º x )
2 2
3 y 75º y 15º
cos (45º x ) x 45º 30º ó x 45º 330º ó
2 x 15º x 75º

91
PÁGINA 128

92
SOLUCIONES

25. Según la figura siguiente:

Llamando al ángulo bajo el cual se ve el pedestal, tenemos:

60 1,8
tg tg 60 9 x x 69 61,8 x
tg ( ) ;
1 tg tg x 60 1,8 x x 2 108
1
x x
7,2 x 2 7 452 x 32,17 m

La anchura del río es de 32,17 metros.

26. Queda del siguiente modo:

A 3 1 cos A 3 1 24
tg cos A ; sen A cos A
2 2 1 cos A 2 5 5

Hallamos :
96
cos 3 A cos A 2 sen 2 A sen A
125

93
27. Sea el esquema:

Los cálculos quedan:

H
tg 60 H 6 3 H 10,39 m
6
h 1
tg 30 h 6 h 3,46 m
6 3
h
2
2 tg H h H 6
tg 2 . Sustituyendo tg 2 y tg obtenemos:
1 tg 2 6 6 6 h2
1
36

6 36 H 2 36
H h2 72h 36H 0 h
H

Ésta es la relación que liga ambas alturas. Relación que se verifica para los valores obtenidos
anteriormente.

28. A partir del desarrollo de tg( A B C ) y de sustituir A B C 180º , obtenemos la expresión


buscada:

tg A tg (B C ) tg A tg B tg C tg A tg B tg C
tg ( A B C ) tg A (B C )
1 tg A tg (B C ) 1 tg B tg C tg A tg B tg A tg C

Como A B C 180º entonces tg ( A B C ) 0 y queda tg A tg B tg C tg A tg B tg C

94
29. Sea un triángulo:

El área del triángulo es:

1 1
S base altura c h
2 2

Vamos a calcular h:

h
tg A
x tg A tg B
h c
h tg A tg B
tg B
c x

De modo que sustituyendo en el área obtenemos la fórmula buscada:

1 tg A tg B 1 2 tg A tg B
S c c c
2 tg A tg B 2 tg A tg B

95
Unidad 6 – Números complejos
PÁGINA 131

SOLUCIONES

1. Las soluciones de las ecuaciones dadas son:


x2 4 0 x 2 x2 4 0 x 2i

2. En cada uno de los casos:

1) 2a 2b 5,2 a 0,8 unidades a 1,8 u


o
a b 1,44 b 1,8 unidades b 0,8 u

2) 2a 2b 4,8
no tiene soluciones reales
a b 1,69

3. Llamando x, y a los catetos del triángulo rectángulo obtenemos:

x y
7
2 12 x 2 86 x 168 0
x2 y 2 (12 x 2
y)

Esta ecuación no tiene soluciones reales, por tanto no es posible construir el triángulo que
proponía Diofanto.

96
PÁGINA 145

SOLUCIONES

1. Si cada punto representa una lámpara, la solución quedaría del siguiente modo:

2. Si hay n calles el número máximo de cruces es:


n2 n
Cn,2 .
2
n2 n
Luego si hay 66 farolas 66 cruces n 2 n 132 0 n 12 calles como
2
mínimo tenía el pueblo.

3. Ésta es una de las disposiciones en que quedó la cava.


1 20
Como máximo se pudo robar:

60 42 18 botellas.
20 1
La disposición de las 42 botellas en la cava admite muchas
formas diferentes.

97
PÁGINA 148

98
SOLUCIONES

1. Las soluciones son:

a) 6 3i b) 13 12i c) 7 3i d) 11 9i

2. Las soluciones son:

a) 26 2i b) 22 21i c) 10
1 6 5
d) i e) i f) i
10 5 2

3. Las soluciones son:

a) 5 2i b) 3 3i c) 7 24i d) 8i e) 15 8 i
7 1 5 99
f) 14 2i g) 7 i h) i i) i j) 1 i
2 2 4 4

4. Las soluciones son:

a) i17 i b) i50 i2 1
301 1 723 3
c) i i i d) i i i
i20 i i
e) i1 999 i3 i f) i i2 i3 ... i20 0
i 1
i2 000 i i i302 i2 1 1 1
g) i i2 i3 ... i2 000 1 h) i
i 1 i485 i274 i i 2
i 1 2 2
3 6 11 13
i i i i i 1
i) i
1 i3 i 1

5. Quedan del siguiente modo:

2
a) x 3 i x 2 9 6x i imaginario puro x 2 9 0 x 3

b) 2 i a 3i 2a 6 i a i 3 nº real 6 a 0 a 6

7 i (7 i) (2 i)
c) (a b i) (2 i) 7 i (a bi) 3 i
2 i (2 i) (2 i)

2 xi (2 x i)(1 x i) 2 x 2 3x i
d) nº real 3x 0 x 0
1 xi (1 x i)(1 x i) 1 x2

99
6. La tabla queda del siguiente modo:

Forma
Afijo Forma polar Forma trigonométrica
binómica

(2,2) 2 2i 2 2 2 2 (cos 45º i sen 45º )


45

(1, 1) 1 i 2 315 2 (cos 315º i sen 315º )


( 3,4) 3 4i 5127 5 (cos 127º i sen 127º )

(0,1) i 190 1(cos 90º i sen 90º )

( 1,1) 1 i 2 135 2 (cos 135º isen 135º )

(2 3,2) 2 3 2i 430 4 (cos 30º i sen 30º )

( 2, 2) 2 2i 2225 2(cos 225º i sen 225º )

(2,2 3) 2 2 3i 4 60 4 (cos 60º i sen 60º )

7. Las representaciones quedan del siguiente modo:

100
8. Las soluciones son:

2 2
a) 420 325 1245 12 (cos 45º i sen 45º ) 12 i 6 2 6 2i
2 2

1254 3 1
b) 430 4 (cos 30º i sen 30º ) 4 i 2 3 2i
324 2 2

2083 1 3
c) 460 4 (cos 60º i sen 60º ) 4 i 2 2 3i
523 2 2

d) 6 (cos130º i sen130º) 3(cos 80º i sen 80º) 6130 380 18210 18 (cos 210º i sen 210º)
3 1
18 i 9 3 9i
2 2

4 (cos 330º i sen 330º) 4330 1 3


e) 2300 2(cos 300º i sen 300º) 2 i 1 3i
2 (sen 30º i sen 30º) 230 2 2

101
PÁGINA 149

102
SOLUCIONES

9. Las potencias quedan:

3
a) 130 190 i

2
b) 4225 16450 1690 16 i
3 3
c) 4 cos 20º i sen 20º 420 6460º 32 32 3 i

10. Cada uno de los casos quedan:

a) 1 i 2 45
2
(1 i)2 2 45 290 2i
3
(1 i)3 2 45 2 2 135 2 2i
4
(1 i)4 2 45 4180 4

b) 1 i 2 315
2
2
(1 i) 2 315 2630 2270 2i
3
(1 i)3 2 315 2 2 945 2 2 225 2 2i
4
(1 i)4 2 315 41260 4180 4

c) (2 3i)2 5 12i
(2 3i)3 ( 5 12i)(2 3i) 46 9 i
(2 3i)4 ( 5 12i)2 119 120 i

d) ( 2 i)2 3 4i
( 2 i)3 (3 4i)( 2 i) 2 11i
( 2 i)4 (3 4i)2 7 24i
2 3 4
e) 245 490 ; 245 8135 ; 245 16180
2 3 4
f) 1225 190 ; 1225 1315 ; 1225 1180
2 3 4
g) 390 9180 ; 390 27270 ; 390 810
2 3 4
h) 3180 90 ; 3180 27180 ; 3180 810

103
11. Las soluciones quedan:

4
a) ( 4 4i)4 32 315 1024180

5
5
3 i 230 32150
b) 3 3
8 2 75 8 2 285
1 i 2 315 2 2 225

i8 i5 1 i 1 i 2 135
c) 1135
2i 2i 2 20

7 7 7
d) i39 4 4 3i i 4 4 3i 1270 8240 1270 87 240 87150

8
8
2 3 2i 4330 48120 1
e) 6 6
4 2 4 2i 8135 8690 4 30

73 3 3 3 1 3330
f) i i i 3330 3240
2 2 190

12. Las raíces quedan:


a) i 190 190 130 Las raíces son: 130 ; 1150 ; 1270
3 3
360 K 120 K
3

b) 25 25180 25 180 360 K 590 180 K Las raíces son: 590 ; 5270
2

c) 4 1 i 2 45 2 45 2 11 15' Las raíces son: 2 11 15' ; 2 101 15' ; 2 191 15' ; 2 281 15'
4 8 8 8 8 8 8
360 K 90 K
4

d) 27 27180 3 180 360 Las raíces son: 360 ; 3180 ; 3300


3 3
360 K 120 K
3

3 6 6 6 6 6
e) 3 1 i 2 315 2 315 360 K 2 105 120 K Las raíces son: 2 105 ; 2 225 ; 2 345
3

1 i 3
f) 3 i 3
1270 1270 360 K 190 120 K Las raíces son: 190 ; 1210 ; 1330
1 i 3

104
32
g) 5 5
32i 5
3290 2 90 360 K 218 72 K Las raíces son: 218 ; 290 ; 2162 ; 2234 ; 2306
i 5

6
h) 6 729 i 6
72990 729 90 360 K 315 60 K Las raíces son: 315 ; 375 ; 3135 ;3195 ; 3225 ; 3315
6

6 6
i) i 6
190 1 90 360 K 115 60 K Las raíces son: 115 ; 175 ; 1135 ; 1195 ; 1225 ; 1315
6

i5 i 5
3 3
j) 3
1 3
10 10 360 K 1120 K Las raíces son: 10 ; 1120 ; 1240
2i 3

13. Quedan del siguiente modo:

6
a) 1 6
10 10 360 K 160 K Las soluciones son: 10 ; 160 ; 1120 ; 1180 ; 1240 ; 1300
6

Gráficamente obtenemos los vértices de un hexágono regular centrado en el origen de


coordenadas.

3
b) 1 3
1180 1180 360 K 160 120 K Las soluciones son: 160 ; 1180 ; 1300
3

Gráficamente obtenemos los vértices de un triángulo equilátero centrado en el origen de


coordenadas.

c) 4 16 i 4
1690 2 90 360 K 222,5 90 K Las soluciones son: 222,5 ; 2112,5 ; 2202,5 ; 2292,5
4

Gráficamente obtenemos los vértices de un cuadrado centrado en el origen de coordenadas.

5
d) 32 5
32180 2180 360 K 236 72 K Las soluciones son: 236 ; 2108 ; 2180 ; 2252 ; 2324
5

Gráficamente obtenemos los vértices de un pentágono regular centrado en el origen de


coordenadas.

e) 3
27 i 3
27270 3 270 360 K 390 120 K Las soluciones son: 390 ; 3210 ; 3330
3

Gráficamente obtenemos los vértices de un triángulo equilátero centrado en el origen de


coordenadas.

105
14. Sustituyendo en la ecuación original cada una de las raíces obtenemos:

2
4 3i 8 4 3i 25 16 24 i 9 32 24 i 25 0
2
4 3i 8 4 3i 25 16 24 i 9 32 24 i 25 0

15. Las ecuaciones quedan del siguiente modo:

Toda ecuación de segundo grado se puede construir del siguiente modo a partir de sus dos
soluciones: z 2 S z P 0 con S suma de soluciones y P producto de soluciones.

a) S i ( i) 0
z2 1 0
P i ( i) i2 1

b) S (2 2i) (2 2i) 4
z 2 4z 8 0
P (2 2i) (2 2i) 8

c) S (2 3i) (2 3i) 4
z 2 4z 13 0
P (2 3i) (2 3i) 13

d) 245 2 2i S 2 2i 2 2i 2 2
z2 2 2 z 4 0
2315 2 2i P 2 2i 2 2i 4

16. Las ecuaciones quedan:

6
a) z 6 1 0 z 1 6
10 10 360 K 160 K z0 10º ; z1 160º ; z2 1120º ; z3 1180º ; z4 1240º ; z5 1300º
6

1 3 1 3i 1 3 1 3
b) z 2 z 1 0 z z0 i; z1 i
2 2 2 2 2 2

4
c) z 4 1 0 z 1 4
1180 1180 360 K 145 90 K
z0 145º ; z1 1135º ; z2 1225º ; z3 1315º
4

d) z 3 6z 2 10z 8 0 ( z 4) ( z 2 2z 2) 0 z0 4; z1 1 i; z2 1 i

4
e) z 4 81 0 z 81 4
810 30 360 K 390 K z0 30º ; z1 390º ; z2 3180º ; z3 3270º
4

6
f) z 6 64 0 z 64 6
640 20 360 K 260 K z0 20º ; z1 260º ; z2 2120º ;
6

z3 2180º ; z4 2240º ; z5 2300º

106
17. Las soluciones quedan:

1 1 4 1 3i
a) z 2 z 1 0 z
2 2
1 3 1 3
z1 i 1120 ; z2 i 1240
2 2 2 2

12 12 16 2 3 2i
b) z 2 12 z 4 0 z 3 i
2 2

z1 3 i 230 ; z2 3 i 2330

i i2 8 i 9 1 3i
c) z 2 i z 2 0 z
2 2 2
z1 i; z2 2i

28 784 108 28 26 27
d) z 6 28z 3 27 0 z3
2 2 1
3 3
27 z 27 27 0 3120 K z0 30 ; z1 3120 ; z2 3240
Para cada solución : 3 3
1 z 1 10 1120 K z3 10 ; z4 1120 ; z5 1240

e) z 3 1 0
3
z 1 3
1180 1180 360 K 160 120 K
3

z0 160 ; z1 1180 ; z2 1300

f) z 3 64 i 0 z 3
64 i 3
6490 4 90 360 K 430 120 K
3

z0 430 ; z1 4150 ; z2 4270

107
18. Las demostraciones quedan:

• (cos i sen )2 cos 2 i sen 2 cos2 sen 2 2 sen cos i cos 2 i sen 2
2 2
cos 2 cos sen
sen 2 2 sen cos

• (cos i sen )4 cos 4 i sen 4 cos4 sen 4 6 cos2 sen 2


4 sen cos3 i 4 sen 3 cos i cos 4 i sen 4
4 4 2 2
cos 4 cos sen 6 cos sen
sen 4 4 sen cos3 4 sen 3 cos

19. Quedan resueltas del siguiente modo:

2 2
b 4i b 4 4 b b 4 4 b b 2 16
a) i 26 26 b 2 36 b 6
1 i 2 2 2 2 2

b) (a b i) (a b i) 24 a 12
Los números complejos son: (12 5i); (12 5i)
a 2
b 2
26 b 5

c) (a b i) (c d i) 5 3i a c 5 a 4 a 4
a bi b d 3 c 1 c 1
imaginario puro ó
c di bd 4c 0 b 4 b 1
a 4 a 4 d 1 d 4

Los números complejos son: (4 i) y (1 i) o bien (4 i) y (1 4i)

108
PÁGINA 150

109
SOLUCIONES

20. Queda:

z 125i3
3
z 5i z 125i

Las otras raíces son:


5 3 5 5 3 5
3
125 i 3
125270 590 120 K z0 590 5i; z1 5210 i; z2 5330 i
2 2 2 2

21. La solución queda:

z 1 i 2 45º z 5 z 4 z 4 2 225º
2
z2 1 i 2i 290º z 6 z 5 z 8270º
3
z3 1 i 2 2 135º z 7 z 6 z 8 2 315º
z 4 z 2 z 2 290º 290º 4180º z 8 z 7 z 16360º

22. Los vértices del hexágono regular son:

2180 ; 2240 ; 2300 ; 20 ; 260 ; 2120


Las coordenadas:
(2,0); (1, 3); (–1, 3); (–2,0); (–1, – 3); (1, – 3)

23. Los vértices del cuadrado son:

290 ; 2180 ; 2270 ; 20


Las coordenadas:
(0,2); (–2,0); (0, – 2); (2,0)

24. La solución queda:

Se obtiene el número complejo girando 90º, es decir, si el número complejo tiene como afijo
(a, b) obtenemos, al multiplicar por i, el número complejo de afijo ( – b, a).

Al dividir el número complejo i, se obtiene el mismo número complejo girado 270º.

a bi
b a i. Su afijo es (b, a )
i

25. Los vértices del cuadrado son: (3,4); (– 4,3); ( 3, – 4); (4, 3)

110
26. Los vértices son: 2 i; 5 3i; 3 6i; 1 5i

27. Quedaría del siguiente modo:

Sea el complejo : z a b i
1 1
Su módulo es :
1 1 a b a 2
b 2 z
i
z a b i a b2
2
a 2
b2 b
Su argumento : tg
(es el opuesto del argumento de z )
a
Gráficamente, el inverso de z se obtiene por una homotecia
1
de razón K y ángulo ( arg z )
z

28. Los vértices de la figura se obtienen a partir vienen de la siguiente ecuación:

3
64 3
64180 4180 360 K 460 120 K 460º ; 4180º ; 4300º
3

Calculamos el lado l mediante el teorema del coseno:

l 2 42 42 2 4 4 cos120º l 6,93 u

Perímetro 3 l 20,79 u

l 3
l
2 l2 3
Área 20,78 u2
2 4

29. La solución queda:

z4 4 0
4
z 4 4
4180 2 180 360 K
2 45 90 K
4

Los vértices son: 2 45 1 i 2 135 1 i 2 225 1 i 2 315 1 i

111
30. Sean los complejos (a b i) y (a b i). El triángulo que se forma es el de la figura:

Para que sea equilátero se debe cumplir que: a2 b2 2b


2b a
Para que su área sea 2 3 se debe cumplir: 2 3
2
Resolviendo el sistema obtenemos:

a2 b2 2b a 2 3b 2 a 6
ab 2 3 ab 2 3 b 2

Los complejos son : ( 6 2 i) y ( 6 2 i)


( 6 2 i) y ( 6 2 i)

31. Los vértices de la figura se obtienen a partir vienen de la siguiente ecuación:

6
64 i 6
64270 2 270 360 K 245 60 K 245º ; 2105º ; 2165º ; 2225º ; 2285º ; 2345º
6

Calculando el valor del área obtenemos: 6 3 u2 .

112
Unidad 7 – Geometría analítica en el plano
PÁGINA 153

SOLUCIONES

1. La ecuación de la recta que pasa por A y B es: x 2y 9 0 . El punto C no pertenece a la recta


pues no verifica la ecuación. Por tanto A, B y C no están alineados.

2. Calculemos la longitud de los lados del triángulo:

d ( A, B ) 20 d ( A,C ) 20 d (B,C ) 8

Por lo que el triángulo es isósceles.


El perímetro y el área se calculan del siguiente modo:

Perímetro 20 20 8 4 5 2 2u

d(B,C) 8 2 2 ; P es el punto medio de BC P (6,5) d(A, P ) 3 2

2 2 3 2
Área 6 u2
2

113
3. El baricentro de un triángulo tiene de coordenadas:

x1 x2 x3 y 1 y2 y3
, En nuestro caso queda g (2, 2)
3 3
Las ecuaciones de las medianas quedan:

Desde A al punto medio de BC queda : x y 0


Desde B al punto medio de AC queda : 2 y 6 0
Desde C al punto medio de AB queda : x 2

4. Las ecuaciones pedidas son:

a) y 3x 3 b) y x 3

114
PÁGINA 167

SOLUCIONES

1. Podemos resolver el problema mediante ecuaciones pero es un camino muy complicado.


Intentaremos representar la situación:

Finca grande Finca pequeña

x x x
x x x 2 2 2
3 3 3 3 3 3
1 día 1 día 1 día media cuadrilla Sin segar
2 2 2
toda la cuadrilla media
cuadrilla

Superficie finca grande x Superficie finca pequeña x


2

Las condiciones del problema nos muestran que si toda cuadrilla trabajó durante la mitad del día
en la finca grande y sólo la mitad de la cuadrilla el otro medio día. Entonces la mitad de la
cuadrilla vendimió la tercera parte de la finca grande en medio día, es decir, x . Luego en la
3
finca pequeña durante medio día vendimiaron el equivalente a la finca grande, es decir,
x 2 x , luego quedó sin vendimiar x de la finca pequeña que la vendimió 1 trabajador al
3 6 6
día siguiente.

Si un trabajador vendimia x en un día y se vendimiaron el campo grande 3 x más el


6 3
pequeño 3 x x todos los trabajadores en 1 día, entonces el primer día se hicieron:
6 6

3x 3x 3x 6x 2x 8x x
8
3 6 6 6 6 6 6

Es decir, en la cuadrilla había 8 vendimiadores.

115
2. Hay que ver que x 2 1 12 .

x 1 3 y x 1 4
2
x 1 x 1 x 1 Al ser x primo 3 o

x 1 4 y x 1 3

En ambos casos, x 2 1 3 4 12

3. Hacemos el siguiente diagrama:

Páginas
1 9 10 99 100 999 1 000 1 025
numeradas
Dígitos
9 180 2 700 100
usados
Total dígitos 9 180 9 180 9 2 700 2 889 2 889 100

En total hacen falta: 2 889 100 2 989 dígitos.


100 dígitos son 25 páginas, entonces hacen falta 999 25 1 024 páginas.
El libro tiene 1 024 páginas.

4. Por medio de ensayo y error dirigido se obtiene:

Con la información referida a los Reyes (R) y las Damas (D) llegamos a que puede ser RDD o
DRD.

Con la información referida a los Corazones (C) y las Picas (P) llegamos a que puede ser
PCP o PPC.

Juntando los resultados obtenidos llegamos a que la solución es: Rey de Picas – Dama de Picas
– Dama de Corazones.

116
PÁGINA 170

117
SOLUCIONES

1. El módulo queda:

V AB (3,2) B (4, 2) y el módulo quedará V AB 3 2 22 13

2. Consideramos el rectángulo ABCD:

V AD V BC ( 3,3)
Como A (1,1) y V AD ( 3,3) D ( 2, 4)

3. Quedan del siguiente modo:

a) v 26 ; w 5; u 13

17 65 33
b) cos(v , w ) ; cos(v , u ) ; cos(w , u )
5 26 13 26 65

c) (v , w ) 48º10 47 ; (v , u ) 11º18 36 (w , u ) 59º 29 23

d) v w u (3,21) '

e) Vector normal a w ( 3,4) es n (4,3)

f) 3 v 3 (1,5) (3,15)

g) Un vector paralelo a v (1,5) es p (1,5) ó q (2,10)

4. La solución es:

a) v w v w cos(v , w ) 4 6 cos 45º 12 2 16,97


9
b) v w v w cos(v , w ) 3 3 cos 60º 4,5
2
c) v w (3, 4) ( 12, 5) 16
d) v w ( 3,4) (15, 20) 125

118
5. La demostración queda:

Como a y b son unitarios a b 1


Calculemos el producto escalar de (a b ) por (a b ):
(a b ) (a b ) a a b b a 2 b 2 0
Al ser su producto escalar nulo,podemos decir que son ortogonales.

6. Quedaría:

3y 5 x 0 36 36
v w 0 x x
5 5
w 13 y 2 25 13 y 12 y 12

7. Quedaría:
1
x
c a 1 x 5y 1 16 1 3
El vector queda: c ,
c b 0 3x y 0 3 16 16
y
16

8. Quedaría del siguiente modo:

V AB (2,4)

Mediante los vectores perpendiculares y paralelos


a V AB obtenemos las dos soluciones del problema
como se observa en la figura.

119
9. La tabla queda:

120
PÁGINA 171

121
SOLUCIONES

1 2
10. Un punto de esa recta es P 0, . Su pendiente es m y su vector director v 1 (6,2) o
2 6
v2 (3,1) .
Con estos datos obtenemos las ecuaciones:
x 0 3t
1
Ecuación vectorial: ( x, y ) 0, t (3,1) Ecuaciones paramétricas: 1
2 y t
2
1
y
x 0 2 1 1
Ecuación continua: Ecuación explícita: y x
3 1 3 2

11. Para cada valor queda:

a) El punto (3,1) debe verificar la ecuación de la recta 3a 3 9 0 a 2


a a
b) La recta ax 3 y 9 0 en forma explícita es : y x 3 m 1 a 3
3 3
4 a 4
c) Uno de sus vectores es v (6, 4) m . Como m a 2
6 3 6

12. Ambos ejes quedan:

Eje OX: Pasa por el punto O 0,0 y uno de sus vectores directores es v (1,0) .
La ecuación será: y 0

Eje OY: Pasa por el punto O 0,0 y uno de sus vectores directores es w (0,1) .
La ecuación será: x 0

Bisectriz 1.er y 3.er cuadrante: Pasa por el punto O 0,0 y forma un ángulo de 45º con el eje
OX, es decir: m tg 45º 1.
La ecuación será: y x

Bisectriz 2.o y 4.o cuadrante: Pasa por el punto O 0,0 y forma un ángulo de 135º con el eje
OX, es decir: m tg135º 1.
La ecuación será: y x

122
13. Las posiciones son:

a) Paralelas.
b) Secantes no perpendiculares.
c) Secantes perpendiculares.
d) Coincidentes.
e) Secantes no perpendiculares.
f) Paralelas.

14. Los vectores en cada caso son:


Vamos a llamar v al vector director y n al vector normal.

a) v (5,2) y n ( 2,5)
b) v ( 2,3) y n (3,2)
c) v (1, 4) y n ( 4,1)
d) v (3,1) y n (1, 3)

15. Aplicamos la definición de producto escalar a los dos vectores directores de las rectas en cada
uno de los casos:
vr ws 2 32
a) v r (1,4) w s (2,8) cos(v r , w s ) 1 (v r , w s ) 0º
vr ws 17 68
vr ws 6 6
b) v r (2, 3) w s (3,2) cos(v r , w s ) 0 (v r , w s ) 90º
vr ws 13 13

16. Calculamos la longitud de los lados como el módulo de cada vector:

V AB (1, 1) V AB 2
V AC ( 2, 2) V AC 8 Perímetro 2 8 10 3 2 10 7,4 u
V BC ( 3, 1) V BC 10

17. Queda:
P ( 2,0) ( 2) 3 2 0 2 4
d (P , r ) 1,11 u
r 3 x 2y 2 0 3 2
2 2
13

123
18. La longitud del cuadrado es la distancia entre las rectas:

r 4x y 5 0
Tomando un punto de la recta r Pr (0,5)
s 8 x 2y 12 0
2
8 0 2 5 12 2 2 2 4
l d (r , s ) d (Pr , s ) Área l 0,059 u2
8 2
( 2)2
68 68 68

19. Cada caso queda:

a) Todas las son de la forma: y 2 x K y la que pasa por P (1,1) es: y 2 x 3.


Todas las son de la forma: x 2y K 0 y la que pasa por P (1,1) es: x 2y 1 0 .

b) Todas las son de la forma: 2 x 4 y K 0 y la pedida: 2 x 4 y 12 0


Todas las son de la forma: 4 x 2y K 0 y la pedida es: 4 x 2y 6 0 .

x 0
y 0
La recta paralela es: 3 x y 0.
31
x 0 y 0
La recta perpendicular es: x 3 y 0.
3 1

20. Queda del siguiente modo:

x 2y 4 0
Punto de intersección es P 2, 3
y 4x 5

Todas las paralelas a 3 x 2y 0 son de la forma 3 x 2y K 0 , la que pasa por el punto


P 2, 3 cumple: 6 6 K 0 K 12 , es decir, la recta pedida es: 3 x 2y 12 0 .

21. Queda del siguiente modo:

3 6 3 6
a) m1 y m2 Si son paralelas : m1 m2 m 10
5 m 5 m

3 6 3 6 18
b) m1 y m2 Si son perpendiculares : m1 m2 1 1 m
5 m 5 m 5

c) No existe ningún valor de m para que sean coincidentes.

d) P (6,5) 6 x my 1 36 5m 1 m 7

124
22. La mediatriz del segmento AB pasa por el punto medio del segmento AB, Pm 3,2 y tiene como
vector director el perpendicular al segmento AB:

V AB (4,0) V (0, 4) La recta pedida es : x 3 0

23. Hallamos la recta AB, que pasa por A y es perpendicular a la recta dada. Donde se corten
ambas rectas es el punto medio entre A y B.

rAB 3 x 4 y – 3 0
Pm (9, 6) B (17, 12)
4 x 3 y 54

125
PÁGINA 172

126
SOLUCIONES

24. El segmento tiene de extremos A (6,0) B (0,4). La mediatriz del segmento AB pasa por el punto
medio del segmento AB y es perpendicular a la recta dada:

Pm (3,2) mediatriz : 3 x 2y 5 0

25. La solución queda:

a) El simétrico de A respecto a B es el punto A´ de modo que B es el punto medio del segmento


AA´, es decir, A´(5,3).

b) Para hallar el simétrico B´, del punto B respecto a la recta dada hallamos la recta
perpendicular a la dada pasando por B. En el punto en el que ambas se cortan es el punto
medio P de B y B´.
recta dada x–y 5 0
P (0,5) B´( 3,8)
recta por B x y 5 0

26. Hallamos la ecuación de la recta perpendicular a la dada pasando por P ; el punto en que se
cortan ambas rectas es el punto medio entre P y P´.

x 2y 1 0
Pm (1,1) P '( 1,0).
2x y 3 0

Veamos que la distancia de P y P´a la recta r 2 x y 3 0 es igual:

2 3 2 3 5 2 ( 1) 0 3 5
d (P , r ) 5 d (P´, r ) 5.
22 12 5 22 12 5

27. Todas las rectas paralelas a la dada tienen la ecuación: 3 x 4 y K 0 . Basándonos en esto
calcularemos el valor de K que cumpla las condiciones dadas.
Tomamos un punto de la recta 3 x 4 y 1 0 P ( 1,1).

3 ( 1) 4 (1) K K 1
3 3 K 1 15 K 14 K 16 .
3 2
4 2 5

Hay dos rectas paralelas que disten 3 unidades de la dada y son las rectas de ecuaciones:

3 x 4 y 14 0 3 x 4 y 16 0

127
28. Todas las rectas perpendiculares a la dada tienen por ecuación: 3 x 4 y K 0 . Calcularemos
el valor de K para que cumpla las condiciones del problema. Sea P (0,0) el origen de
coordenadas, se ha de cumplir:

3 0 4 0 K K
5 5 K 25
3 2 2
( 4) 5

Las rectas que cumplen las condiciones son:

3 x 4 y 25 0 3 x 4 y 25 0

29. El vértice B es de la forma B (a, 2 2a ) por pertenecer a la recta.


Si el triángulo es rectángulo en A V AB V AC V AB V AC 0 (a 1, 5 2a) (2, 3) 0
13 13 5
De aquí se obtiene que a B ,
8 8 4

30. La solución queda:

Los vértices B 1, 3 y C 3,1 forman el lado desigual del triángulo isósceles.


El vértice A está situado en el punto de intersección de la recta dada x y 3 0 y de la altura
del triángulo que, partiendo de A, va a pasar a la base BC.
Veamos la ecuación de la altura: pasa por el punto medio de BC: Pm 2, 1 y es perpendicular a
la base.
V BC (2,4) V altura ( 4,2)

x 2 y 1 x 2y 0
x 2y 0 A ( 6,3)
4 2 x y 3 0

El área del triángulo es:

base·altura base d (B,C ) 2 5 2 5 4 5


Área Área 20 u2
2 altura d ( A, Pm ) 4 5 2

128
31. El vértice C por pertenecer a la recta es C a, 3 a

base d ( A, B ) 17
base·altura
Área 5a 4
2 altura d (C, rAB )
17

Quedando :
16 16 31
a C ,
1 5a 4 5 5 5
6 17 5a 4 12
2 17 8 8 7
a C ,
5 5 5

32. La solución queda:

El punto C, por pertenecer a la recta,


será de la forma: C a, 2a 2 .
A la vista del dibujo se debe cumplir:

V AB V AC 0 (a 3, 2a 2) ( 2,3) 0
a 0 C (0, 2)

33. Hay tres soluciones:


Denotemos a los vértices dados por A (2,3), B (5,1),C (4,0) y al que hemos de hallar por D. Las
soluciones son:
1. Una solución es el paralelogramo del dibujo ABCD, en el cual:

V AB V DC (3, 2) (4 x,0 y )
x 1; y 2 vértice D (1,2)

2. Otra solución es el paralelogramo del dibujo ACBD, en el cual:

V AC V DB (4 2,0 3) (5 x,1 y )
x 3; y 4 vértice D (3,4)

129
3. La tercera solución es el paralelogramo del dibujo ACDB, en el cual:

V AC V BD (2, 3) ( x 5, y 1)
x 7; y 2 vértice D (7, 2)

34. El vértice C está en la intersección de la recta perpendicular a AB por B y la bisectriz del 4º


cuadrante.

V AB (4, 4) la recta perpendicular tiene por vector


x 7 y 1
w (4,4) y pasa por B (7,1) x y 6 0
4 4
x y 0
C (3, 3)
x y 6 0
El vector V BC ( 4, 4) es paralelo e igual a V AD ,
luego D ( 1,1)
Área del rectángulo base·altura d (B,C )· d ( A, B )
42 42 · 42 ( 4)2 32u2

35. Calculamos el área del cuadrilátero descomponiendo éste en dos triángulos y sumando las
áreas de éstos.

Área ABD. Calculamos la longitud de los lados:

d ( A, B ) 8,06; d (B, D ) 7,81; d ( A, D ) 4,47;

Área 10,17·(10,17 8,06)·(10,17 7,81)·(10,17 4,47) 17

Área BCD. Calculamos la longitud de los lados:

d (B, D ) 7,81; d (B,C ) 8,06; d (D,C ) 5,83;

Área 10,85·(10,85 8,06)·(10,85 7,81)·(10,85 5,83) 21,5

La solución puede expresarse como suma de los valores anteriores:


Área del cuadrilátero 38,5 u2

130
36. Los vértices del paralelogramo buscado son: A(4,6) ; el punto B es el punto en el cual se corta
la recta y 5 x 2 y la paralela a x 3y 10 0 , pasando por A(4,6) .
Es decir:

y 5x 2
B (1,7)
x 3 y 22 0

y 5x 2
C ( 1, 3)
x 3 y 10 0

x 3 y 10 0
D (2, 4)
y 5 x 14

Área 32 u2

131
Unidad 8 – Lugares geométricos. Cónicas
PÁGINA 175

SOLUCIONES

1. La elipse es una cónica obtenida al cortar una superficie cónica por un plano oblicuo al eje y que
corte a todas las generatrices.

La hipérbola es una cónica obtenida al cortar una superficie cónica por un plano oblicuo al eje,
paralelo a dos generatrices y que corte a todas las demás.

La parábola es una cónica obtenida al cortar una superficie cónica por un plano oblicuo al eje,
paralelo a una generatriz y que corte a todas las demás.

2. Si se corta por un plano paralelo a la base se obtiene una circunferencia. Si el corte es por un
plano oblicuo se obtiene una elipse.

3. El circuncentro es el punto de corte de las mediatrices.

Hallamos dos mediatrices:

mediatriz lado AB x y 1 0
Circuncentro (0,1)
mediatriz lado AC x y 1 0

132
4. Como puede observarse el dibujo, la curva obtenida es una parábola.

133
PÁGINA 191

SOLUCIONES

1. Los pasos a seguir son los siguientes, llamando AB a los montañeros que suben y abc a los que
bajan.

134
2. Señalamos las monedas con C y X.

Consideramos el caso de que sólo tengamos 9 monedas.


En este caso hay 5 caras C y 4 cruces X.
Si la abeja parte de una moneda marcada con C, puede hacer
el recorrido:
CXCXCXCXC

Pero si parte de una moneda marcada con X, no puede:


XCXCXCXC… falta una C.

En nuestro caso hay 13 caras C y 12 cruces X.


Si la abeja parte de una moneda marcada con C, es posible el recorrido, pero si la abeja parte
de una moneda marcada con X no es posible.

3. La solución queda:

Sea el número inicial xyz 100 x 10 y z 100z 10 y x 100 x z z x


Si x z z x 0 hay que escribir la expresión anterior de la forma :
x z 1 100 100 z x x z 1 100 9·10 10 z x
a
La 1. cifra de este número es : x z 1.
La 2.a cifra de este número es : 9.
La 3.a cifra de este número es : 10 z x.

Observamos que x z 1 10 z x 9 , es decir, la 1ª 3ª siempre da 9 y la 2ª también da


9. Luego siempre se cumple el resultado del problema.

135
PÁGINA 194

136
SOLUCIONES

1. Queda del siguiente modo:

Sea P ( x, y ) un punto genérico del lugar geométrico buscado. Dicho punto debe cumplir:

d (P , A ) d ( P , B )
De donde:
2 2 2 2
x 7 y 2 x 12 y 2

Elevando ambos miembros al cuadrado y operando obtenemos:

38 x 95 0
que es el lugar geométrico buscado.

Podíamos haber hecho el problema viendo que esta definición de lugar geométrico se ajusta
a la mediatriz del segmento AB.

2. La solución queda:

Sea P ( x, y ) un punto genérico del lugar geométrico buscado. Dicho punto debe cumplir:

d (P , A ) 3
d (P , B ) 2

Expresando las distancias en coordenadas cartesianas y operando se obtiene la siguiente


ecuación del lugar geométrico:
5 x 2 5 y 2 34 x 28 y 25 0

17 14 72
Este lugar geométrico es la circunferencia de centro , y radio .
5 5 5

3. Sea P ( x, y ) un punto genérico del lugar geométrico buscado. El desarrollo de la relación


d (P, A) 2d (P, r ) , nos conduce a la ecuación:

x2 y2 4 xy 6 x 12 y 3 0

La ecuación anterior es una hipérbola.

137
4. Sea P ( x, y ) un punto genérico del lugar geométrico buscado. Debe verificar: d (P, r ) d (P, s ) ,
es decir:
x 3y 3 3x y 2
2 2
1 3 32 12

El lugar geométrico buscado son las rectas de ecuaciones:


2 x 2y 5 0
4x 4y 1 0
Estas rectas son las bisectrices de los ángulos que forman las rectas dadas.

5. Queda del siguiente modo:

a) La circunferencia tiene de ecuación: ( x 2)2 ( y 0)2 3 2 o bien desarrollando obtenemos la


forma siguiente : x 2 y 2 4 x 5 0 .

2
b) La circunferencia tiene por centro el punto C 1,2 y por radio r 42 3 5 . Por tanto
su ecuación es: ( x 1)2 ( y 2)2 25 .

c) La circunferencia tiene por centro el punto medio de los extremos del diámetro C 0,0 y por
radio r 5 . Su ecuación es: x 2 y 2 25 .

7
d) La circunferencia tiene por centro el punto C 1,4 y por radio r d (C,recta tangente) .
5
2
2 2 7
Por tanto su ecuación es: ( x 1) ( y 4) .
5
La circunferencia buscada tendrá por centro el circuncentro del triángulo de vértices ABC.
Para hallar el circuncentro hallamos dos mediatrices de este triángulo y el punto en que se
cortan:
mediatriz AB 2x 8y 9 0
mediatriz BC 2x y 4 0
23 13
Centro , Radio 2,76
18 9
2 2
23 13
La ecuación de la circunferencia es: x y (2,76)2
18 9

e) Su centro estará en el punto de intersección de la recta dada con la mediatriz del segmento
PQ:
mediatriz PQ 2 x y 1 0 Centro (1,1)
recta dada 4 x y 3 0 radio d (C, P ) 5
La ecuación de la circunferencia es: ( x 1)2 ( y 1)2 25

138
6. Las soluciones quedan:

a) La ecuación de esta circunferencia se puede escribir en la forma: ( x 2)2 ( y 1)2 5 . Por


tanto su centro es C 2, 1 y su radio es r 5.
b) La recta tangente en el punto P (4,0) pasará por el punto P y es perpendicular a la recta que
pasa por C y P, por tanto su ecuación es: 2x y 8 0 .

c) La circunferencia concéntrica con C tendrá el mismo centro que ésta, es decir, el punto
7
C (2, 1) y su radio es la distancia del centro a la recta tangente, r . Su ecuación será
5
49
de la forma: ( x 2)2 ( y 1)2 .
5

7. El simétrico de P (2,1) respecto a la recta dada es el punto Q ( 6,7) . La ecuación de la


circunferencia que pasa por P (2,1) , Q ( 6,7) y el origen O (0,0) tiene por centro el circuncentro
del triángulo de vértices PQO y por radio la distancia desde el centro a uno de los vértices, es
decir:
2 2
5 5 17 5 2 5 17
Centro ,5 Radio cuya ecuación es x y 5
4 4 4 4

8. La circunferencia tangente a los ejes coordenados tendrá por centro C (a, a ) y su radio valdrá
a unidades. Como ha de pasar por el punto P dado, podemos escribir:

2 2
d (C, P ) radio a 4 a 2 a

Operando obtenemos dos circunferencias:

C1 centro(2, 2) y radio 2 cuya ecuación queda : ( x 2)2 ( y 2)2 4


C2 centro(10, 10) y radio 10 cuya ecuación queda : ( x 10)2 ( y 10)2 100

9. La solución es:

a) El eje radical es la recta x 0 .


b) La potencia pedida es 52 .

10. Esta circunferencia tiene por centro el punto C (1,2) y por radio 1 unidad.
3
Hallamos la distancia del centro a la recta dada y obtenemos: d (C,recta) 0,83 radio
13
Por tanto, como lo distancia del centro a la recta dada es menor que el radio, la recta dada es
secante a la circunferencia.

139
11. En cada uno de los casos queda:

7
a) Focos : F ( 7,0) F ´( 7,0) Semiejes : a 4 b 3 Excentricidad : e 0,6614
4

b) Focos : F (0;6,25 ) F ´(0; 6,25) Semiejes : a 8 b 5


6,25
Esta elipse tiene como eje mayor el OY Excentricidad : e 0,781
8

x2 y 2
c) Escribimos la ecuación reducida : 1
54 36
Focos : F (4,24;0) F´( 4,24;0) Semiejes : a 7,35 b 6 Excentricidad : e 0,58

d) Esta elipse tiene como eje mayor el OY


Focos : F (0,4) F ´(0, 4) Semiejes : a 5 b 3 Excentricidad : e 0,8

12. Hallamos las rectas pedidas en los puntos P (4;4,71) y Q(4; 4,71).

Recta tangente en P: y 4,71 0,147( x 4)


Las ecuaciones de las rectas tangentes son:
Recta tangente en Q: y 4,71 0,147 ( x 4)

1
Recta normal en P: y 4,71 ( x 4)
0,147
Las ecuaciones de las rectas normales son:
1
Recta normal en Q: y 4,71 ( x 4)
0,147

140
PÁGINA 195

141
SOLUCIONES

x2 y2
13. La ecuación de la elipse pedida se presenta de la misma forma: 1.
a2 b2
Imponiendo las condiciones dadas obtenemos:

3 73 x2 y2
a) c ; b 4 a2 donde la ecuación queda : 1
2 4 73 16
4
36 16
b) 1 a 2 100 x2 y2
a2 b2 donde la ecuación queda : 1
b 2 25 100 25
a 10

9 16
c) 1
a2 b2
a 2 34
x2 y2
c 3 544 donde la ecuación queda : 1
b2 34 544
a 5 25 25
a2 b2 c 2

14. El lugar geométrico buscado es una elipse de eje mayor OX y centrada en el origen de
coordenadas, de la que conocemos c 12; 2a 26. Por tanto:
x2 y2
a2 b2 c 2 132 b 2 122 b 2 25 La elipse queda : 1
169 25

x2 y2
15. Suponiendo que la elipse tiene como eje mayor OX, su ecuación es de la forma: 1.
a2 b2
Obligándola a pasar por los puntos dados obtenemos:
4
1 a2 4
a2 x2 y2
4 La elipse queda : 1
1 1 b2 16 4
1 27
a2 9b 2 27

142
16. Queda del siguiente modo:

a) a 6; b 8; c 10
Focos : (10,0) ( 10,0) Semiejes : a 6; b 8
10 4 4
Excentricidad : e 1,7 Asíntotas : y x; y x
6 3 3

b) a 5; b 2; c 29
Focos : ( 29,0) ( 29,0) Semiejes : a 5; b 2
2 2
Excentricidad : e 1,08 Asíntotas : y x; y x
5 5

x2 y 2
c) 1 a 1; b 2; c 5
1 4
Focos : ( 5,0) ( 5,0) Semiejes : a 1; b 2
Excentricidad : e 5 2,24 Asíntotas : y 2x ; y 2x

x2 y2 3 45
d) 1 a 3; b ; c 3,35
9 9 2 4
4
3
Focos : (3,35;0) ( 3,35;0) Semiejes : a 3; b
2
1 1
Excentricidad : e 1,12 Asíntotas : y x; y x
2 2

17. La ecuación de esta hipérbola corresponde a una hipérbola equilátera referida a sus asíntotas.
Sus asíntotas son: y 0; x 0. Sus ejes y x ; y x.
Todo queda:

a2
32 a 2 64 c 2 128 Sus focos son : ( 128,0) ( 128,0)
2

18. Para que represente una hipérbola equilátera, a 9. El semieje vale 2 , puesto que la
3
2 2
x y
ecuación es: 1.
4 4
9 9

143
x2 y2
19. La ecuación reducida de esta hipérbola es: 1
9 9
2
3 3
Vértices: ,0 ,0 Focos: 3,67;0 3,67;0
2 2

Asíntotas son las rectas de ecuaciones: y 2x ; y 2x


Las ecuaciones de la tangente y normal en el punto 3,3 son, respectivamente:
1 9
y 2x 3 y x
2 2

x2 y2
20. La ecuación de la elipse pedida se presenta de la misma forma: 1.
a2 b2
Imponiendo las condiciones dadas obtenemos:

x2 y 2
a) a 6; b 8 donde la ecuación queda : 1
36 64
9
b) 1 a2 9
a2 x2 16 y 2
81 donde la ecuación queda : 1
25 9 b2 9 81
1 16
a2 b2

288 25
c) 1 a 2 144 x2 y 2
a2 b2 donde la ecuación queda : 1
b 2 25 144 25
c 13 a 2 b 2 169

100 16
d) 1
a2 b2
a 2 36 x2 y2
c 5 donde la ecuación queda : 1
b2 9 36 9
a 2
a2 b2 c 2

21. Las parábolas quedan del siguiente modo:

a) Esta parábola tiene por eje el eje OY y por vértice el punto V (0, 1) . Además, la
distancia d (V , F ) p p 6. La ecuación de la parábola puede representarse del
2
2
siguiente modo: x 0 12 y 1 x 2 12y 12 0 .

144
b) Esta parábola tiene por eje la recta y 4 y el parámetro p vale 6, pues ocurre que:
p p
d (V ,directriz) 3 p 6.
2 2
2
La ecuación de la parábola queda: y 4 12 x 3 y 2 8y 12 x 52 0 .

c) La ecuación de todas las parábolas que tienen por vértice el punto V (2, 4) y por eje la
2
recta x 2 es de la forma: x 2 2p y 4 .
Obligándola a que pase por el punto A (8, 7) , dado que obtenemos p 6 , la ecuación
2
final quedará: x 2 12 y 4 x 2 4 x 12y 52 0.

22. Todas las parábolas de eje paralelo a OX tienen por ecuación ( y b )2 2 p ( x a ). Obligándola a
que pase por los puntos dados obtenemos la ecuación:

(1 b )2 2 p (6 a ) a 2
b 3 1
(3 b )2 2 p ( 2 a ) La parábola queda es :( y 3)2 ( x 2) x 2y 2 12y 16
1 2
(6 b )2 2 p (16 a ) p
4

23. La ecuación de esta parábola se puede escribir de la forma: ( x 1)2 y 16 . De esta expresión
se pueden deducir los demás elementos:

65
Eje : x 1 Directriz : y
4
63
Vértice : V ( 1, 16) Foco : F 1,
4

24. Los puntos con igual abscisa que ordenada los hallamos resolviendo el sistema:

y x 2 3x 3 P (1,1)
y x Q (3,3)

La recta tangente en P (1,1) tiene por ecuación: y 1 1( x 1) y x 2


La recta tangente en Q (3,3) tiene por ecuación: y 3 3( x 3) y 3x 6

145
25. Transformando cada una de estas ecuaciones en su correspondiente obtenemos los
elementos:

a) y 2 6 y 12 0 y2 6 ( x 2)
7 1
Eje : y 0 Vértice : V ( 2,0) Foco : F ,0 Directriz : x
2 2

2
b) x 2 4 x 4 y 8 x 2 4 ( y 3)
Eje : x 2 Vértice : V (2, 3) Foco : F 2, 2 Directriz : y 4

2
c) y 2 6 y 2 x 9 0 y 3 2( x 0)
1 1
Eje : y 3 Vértice : V (0, 3) Foco : F , 3 Directriz : x
2 2

146
PÁGINA 196

147
SOLUCIONES

26. Hallamos los puntos dados resolviendo los respectivos sistemas:

x 2 y 2 40 26 18
a) P ( 6, 2) Q ,
x 2y 2 0 5 5

x 2 y 2 40 No tiene solución, luego


b)
9 x 2 16 y 2 25 no hay puntos de corte

x 2 y 2 40 P (6,28; 0,76) Q(6,28; – 0,76)


c)
x 2 25 y 2 25 R(–6,28; 0,76) Q(–6,28; – 0,76)

x 2 y 2 40 P (4,47; 4,47)
d)
xy 20 Q(–4,47; – 4,47)

27. Es una recta de ecuación y 3.

28. En cada uno de los casos quedará:


a) La circunferencia tendrá por centro C (a, a ) y su radio valdrá (a 2) unidades. Como ha de
pasar por el punto P dado, podemos escribir:

2
d (P,C ) radio a2 a 7 a 2

Operando obtenemos dos circunferencias:

C1 centro (3,3) y radio 5 cuya ecuación queda : ( x 3)2 ( y 3)2 25


C2 centro(15,15) y radio 17 cuya ecuación queda : ( x 15)2 ( y 15)2 289

b) Al ser tangente en el O (0,0) a la recta x y 0 , tendrá el centro C en la recta perpendicular


a la tangente en el punto O de tangencia, es decir, en la recta x y 0 .
Además, por pasar por A y por O el centro está en la mediatriz del segmento AO, es decir,
en la recta y 4 0 .
Luego el centro es el punto C 4,4 y el radio r 32.
La ecuación final de la circunferencia es: ( x 4)2 ( y 4)2 32

148
29. El lugar geométrico buscado corresponde a una elipse no centrada en el origen de
coordenadas. Obtenemos su ecuación imponiendo el enunciado a los puntos P ( x, y ) .

( x 2)2 ( y 1)2 ( x 2)2 ( y 1)2 8

Operando obtenemos: 3 x 2 4 y 2 8 y 44 0

30. Es una parábola de ecuación: ( y 4)2 4 ( x 3). Se pueden extraer los elementos a partir de
la ecuación.
Estos quedan: Eje : y 4 Vértice : V (3,4) Foco : F 2,4 Directriz : x 4

31. La circunferencia circunscrita al triángulo PQR tendrá el centro en el circuncentro del triángulo,
es decir, en el punto C (2,2) y su radio es la distancia del centro a uno de los puntos dados, es
decir, r 5.
Su ecuación es: ( x 2)2 ( y 2)2 5

32. Hallamos la ecuación de la secante que pasa por los puntos A (1,32) y B (2,16) , siendo ésta la
recta: 16 x y 48 0 .
Las rectas paralelas a esta secante serán de la forma: 16 x y K 0 .
Obligando a que esta recta corte en un solo punto a la hipérbola, obtenemos: K 32 2 .
Luego las ecuaciones de las rectas tangentes son: 16 x y 32 2 0; 16 x y 32 2 0

33. Las soluciones quedan:

Se puede trazar circunferencias de radio menor o igual que 10 m.


Se pueden trazar elipses, con el procedimiento del jardinero, utilizando cuerdas de 10 m.

En la circunferencia hay que dar el centro y el radio.


El semieje mayor de las elipses no puede superar los 10 m.

149
3 1
34. La circunferencia de diámetro AB tendrá por centro el punto medio de AB, C , y por
2 2
10
radio la mitad de la distancia entre A y B, r .
2
2 2
3 1 5
La ecuación quedaría : x y .
2 2 2
El punto T pertenece a esta circunferencia puesto que verifica su ecuación.
El punto diametralmente opuesto a T es el simétrico de T respecto al centro C, es decir, el
punto P (0,0) .
La recta tangente en el punto T (3, 1) tendrá por vector director el vector perpendicular a TC ,
es decir, el vector v (1,3) ; por tanto, su ecuación es: 3 x y 10 0 .
La recta tangente en el punto P (0,0) tendrá el mismo vector director por ser paralela a la
anterior y por ecuación: 3 x y 0 .

x2 y 2
35. La hipérbola es de la forma: 1
a2 b2
Obligándola a pasar por los puntos dados obtenemos:

25 4
1 a2 9
a2 b2 x2 y2
9 La hipérbola queda : 1
18 9 b2 9 9
1 4 4
a 4b 2
2

36. La ecuación de las parábolas de eje paralelo al eje de ordenadas es y ax 2 bx c .


Imponiendo las condiciones del enunciado obtenemos:

0 4a 2b c a 1
2
1 2
0 36a 6b c b 4 La parábola queda : y x 4x 6
2
6 c c 6

Los elementos de esta parabola son :


3 5
Eje : x 4 Vértice : V (4, 2) Foco : F 4, Directriz : y
2 2

37. Serán rectas de la forma y 0 m ( x 8) cuya distancia al centro de la circunferencia debe


coincidir con el radio.
8m 5
d (C,tangente) radio 5 m
m 2
1 39

Las ecuaciones de las tangentes son: 5 x 39y 40 0 5x 39y 40 0

150
Unidad 9 – Propiedades globales de las funciones
PÁGINA 199

SOLUCIONES

1. Las soluciones pueden quedar así:


a) b)

2. Los dominios quedan:

Dom f Dom g 3, 3

151
PÁGINA 213

SOLUCIONES

1. Designamos los colores por: rojo (R), verde (V), azul (Z) y amarillo (A); y las tres formas por:
cuadrada (C), circular (O), triangular (T) y pentagonal (P).
Por ensayo y error las colocamos en un tablero 4 x 4, cumpliendo las condiciones que marca el
enunciado.
Una solución es:

Podemos encontrar hasta 72 soluciones distintas.

2. El número total de amanitas ha de ser múltiplo de 9 menos 1, es decir, 8 amanitas. Haciendo el


problema mediante ecuaciones:

8 8
x x x 8 amanitas
9 9

152
3. El enunciado del problema nos muestra que el número de latas de zumo debe ser un número
impar. Por ensayo y error dirigido obtenemos:
Hay 7 latas de zumo.
7
El 1.er amigo se bebe 0,5 4 latas. Quedan 3 latas.
2
3
El 2.o amigo se bebe 0,5 2 latas. Queda 1 lata.
2
1
El dueño de la casa se bebe 0,5 1 lata.
2
Luego, efectivamente, había inicialmente 7 latas de zumo.
Este problema se puede resolver también por medio de ecuaciones.

4. Sea n un número real.

Veamos si n 2 n 2 1 12
n2 n2 1 n n n 1 n 1

n n 1 n 1 3, pues es producto de tres números consecutivos.

Si n 3 n 1 2 y n 1 2, luego n 1 n n 1 3 2 2 12,

Si n 1 3 n 2, por lo que n n 1 n n 1 2 3 2 12,

Si n 1 3 n 2, por lo que n n 1 n n 1 2 2 3 12,

En cualquier caso, efectivamente, n 2 n 2 1 12.

153
PÁGINA 216

154
SOLUCIONES

1. Los dominios quedan:

Dom f Dom g 3,0


Dom h 5, Dom i ,1 4,
Dom j Dom k 2,3
Dom l 1 Dom m 2,
Dom n 6, Dom o

Dom p Dom q x x 1 K ;K
2
Dom r ,1 3, Dom s , 2 2,
Dom t 1,1 2, Dom u , 1 3,

2. Las funciones se caracterizan por:

y f (x)
Dom f ; Im f 0,
Estrictamente creciente en todo su dominio.
No tiene extremos relativos.

y g( x )
Dom g 2,2 ; Im g , 1 0,

Estrictamente creciente en , 2 2,0


Estrictamente decreciente en 0,2 2,
Máximo relativo 0, 1

y j(x)
Dom j ; Im j
Estrictamente creciente en , 5 1,
Estrictamente decreciente en 5, 1
Máximo relativo 5, 4
Mínimo relativo 1, 3

155
3. Las representaciones quedan:

4. Las demostraciones quedan del siguiente modo:

Para demostrar que existe un máximo relativo en el punto de abscisa 1 hay que probar que
x E 1; f ( x ) f (1) , es decir, que x 3 6 x 2 9 x 4 x 3 6 x 2 9 x 4 0.(Con 0).
2
Para ello estudiamos el signo de la función: g ( x ) x 3 6 x 2 9 x 4 x 1 x 4

Luego se cumple que x E 1; f ( x ) f (1)

Para demostrar que existe un mínimo relativo en el punto de abscisa 3 hay que probar que
x E 3; f ( x ) f (3) , es decir, que x 3 6 x 2 9 x 0.
2
Para ello estudiamos el signo de la función: h( x ) x 3 6 x 2 9 x x x 3

Luego se cumple que x E 3; f ( x ) f (3)

156
5. Las demostraciones quedan:

2
a) Hemos de probar que 0 2
1 x2
2
0 , que se cumple x.
1 x2
2
2 2 2 2x 2 0 2x 2 ;esto es cierto para x.
1 x2
No alcanza el extremo inferior o ínfimo, luego no tiene mínimo absoluto. Para el valor
x 0 f (0) 2 , luego alcanza el extremo superior o supremo, es decir, tiene un
máximo absoluto en x 2 .

b) Hemos de probar que 2 3 sen(2 x ) 1 4


2 3 sen(2 x ) 1 4 3 3 sen(2 x ) 3 1 sen(2 x ) 1, esto es cierto x.
3 3
3 sen(2 x ) 1 2 sen(2 x ) 1 2x 2 K x K.
2 4
3
Tiene mínimo absoluto K ( K ).
4

3 sen(2 x ) 1 4 sen(2 x ) 1 2x 2 K x K.
2 4

Tiene máximo absoluto K ( K ).


4

157
PÁGINA 217

158
SOLUCIONES

6. El estudio de cada función nos ofrece la siguiente información:

a) Esta función y f ( x ) está acotada por x 0 y x 4 .


El supremo es x 4 y el ínfimo es x 0 .
Esta función tiene un mínimo absoluto en x 0 .
No tiene simetría.

b) Esta función y g ( x ) está acotada por x 3 y x 2.


El supremo es x 3 y el ínfimo es x 2.
Esta función no tiene extremos absolutos.
No tiene simetría.

c) Esta función y h( x ) está acotada por x 3 y x 5.


El supremo es x 5 y el ínfimo es x 3.
Esta función tiene un máximo absoluto en x 5 .
Es simétrica respecto al eje de ordenadas.

d) Esta función y i ( x ) no está acotada.


Es simétrica respecto al origen de coordenadas.

e) Esta función y j ( x ) está acotada inferiormente por x 1.


El ínfimo es x 1 y no tiene supremo.
Esta función tiene un mínimo absoluto en x 1.
No tiene simetría.

f) Esta función y k ( x ) está acotada superiormente por x 2 .


El supremo es x 2 y no tiene ínfimo.
Esta función no tiene extremos absolutos.
Es simétrica respecto al eje de ordenadas.

7. Las funciones se dividen en tres grupos:

Las funciones f; i; k; l; son simétricas respecto al eje de ordenadas.


Las funciones h; j; m; q; son simétricas respecto al origen de coordenadas.
Las demás funciones no tienen simetrías.

159
8. Las funciones quedan:

a) Simétrica respecto al eje de ordenadas y no periódica.


b) No tiene simetría y es periódica de período T 1 .
c) Simétrica respecto al origen de coordenadas y periódica de período T 2 .

9. Las gráficas quedan:

160
PÁGINA 218

161
SOLUCIONES

10. Los dominios quedan:

x x 3 2x 2 x 4
a) (f g )( x ) f ( x ) g ( x ) x2 2 Dominio 2
2 x 2 x

x x 3 2x
b) (f g )( x ) f ( x ) g ( x ) ( x 2 2) Dom f g 2
2 x 2 x

f f (x) x x f
c) (x) :( x 2 2) Dom 2
g g(x ) 2 x (2 x ) ( x 2 2) g

2
x x 3x 2 8x 8
d) ( g f )( x ) g [f ( x )] g 2 Dom g f 2
2 x 2 x 4 4x x 2

e) ( g g )( x ) g [g ( x )] g [ x 2 2] ( x 2 2)2 2 x 4 4 x 2 6 Dom g g

11. Queda en cada caso:

a) f ( x ) x 3 2 f 1 (x) 3
x 2
y se comprueba que : (f f 1)( x ) f [f 1( x )] f [ 3
x 2] x

1 x
b) g ( x ) 1 3 x g 1( x )
3
1 x 1 x
y se comprueba que :( g g 1)( x ) g [g 1( x )] g 1 3 x
3 3

c) h ( x ) 2 x 2
h 1( x ) log2 x 2
y se comprueba que :( h h 1)( x ) h [h 1( x )] h [log2 x 2] 2log2 x 2 2
x

12. Las soluciones son:

a) f g ( x ) 6 x 11 d) (f f )(1) 13
3
b) h g ( x ) e) (g f )( 1) 2
2x 3
x 4 2 x 2 28 3
c) f h( x ) f) (h h )(0)
x 4 2x 2 1 10

162
13. La solución queda:

(f g)( x ) f [g ( x )] f (3 x a ) 5 a 3 x
a 5
(g f )( x ) g [f ( x )] g (5 x ) 3 (5 x ) a 15 a 3 x

14. Queda:
x 1 1
Veamos que (f f f )( x ) i ( x ) x (f f f )( x ) f f [f ( x )] f f f x
x x 1

15. Llamando x a la base, el cateto altura será (20 x ) , por tanto el área viene dada por:

1 2
f (x) x 10 x
2

El dominio de esta función es , es decir, todos los valores reales positivos pueden tomar
como valor el cateto base, excluido el cero.
Es una función acotada superiormente por 50 y este valor es su máximo absoluto, y acotada
inferiormente por 0 que es su mínimo absoluto, aunque estos valores carecen de sentido como
catetos del triángulo.

16. Por ejemplo, las funciones pueden ser:

a) f ( x ) x 2 g( x ) x3 2
b) h( x ) 3 x l ( x ) 2x
x 1
c) t ( x ) p( x ) x 2
x 2

17. Queda:
(f g )( x ) f [g ( x )] f [2 x 4] x 3
(f g ) 1 ( x ) x 3
(f g ) 1 (4) 7

163
18. La representación quedaría del siguiente modo:

Es una función periódica de período 45 minutos.

19. La gráfica queda del siguiente modo:


a) Es la gráfica azul
b) Es la gráfica verde

20. La gráfica queda del siguiente modo:

164
Unidad 10 – Funciones elementales
PÁGINA 221

SOLUCIONES

1. Las ecuaciones para estas gráficas son:

x
2 3 1 1
a) y x 4x 5 c) y b) y x d) y
2x 2 2

2. Las funciones son:

a) f ( x ) 4

f ( x ) es una función constante.


Dom f
Im f 4
Acotada por 4.

165
b) g ( x ) 2x

g ( x ) es una función lineal.


Dom g
Im g
Estrictamente decreciente en todo su dominio.

c) h( x ) 3 x 1

h( x ) es una función afín.


Dom h
Im h
Estrictamente creciente en todo su dominio.

d) i ( x ) x 2 4 x

i ( x ) es una función cuadrática.


Dom i
Im i 4,
Acotada inferiormente ( 4)
Estrictamente decreciente ( ,2)
Estrictamente creciente (2, )
Mínimo relativo en (2, 4)

166
e) j ( x ) 3x

j ( x ) es una función exponencial.


Dom j
Im j 0,
Acotada inferiormente (0)
Estrictamente creciente en todo su dominio.

f) k ( x ) log3 x

k ( x ) es una función logarítmica.


Dom k 0,
Im k
Estrictamente creciente en todo su dominio.

1
g) l ( x )
x

l ( x ) es una función proporcional inversa.


Dom l ,0 0,
Im l ,0 0,
Estrictamente decreciente en todo su dominio.
Simétrica respecto del origen de coordenadas.

167
PÁGINA 237

SOLUCIONES

1. Como cada día asciende 30 m y resbala 20 m, en realidad asciende 10 m.


Luego al cabo de 27 días ha ascendido 270 m, y ya el día 28 asciende a la superficie, pues
asciende 30m 270 30 300m.
El caracol tarda 28 días en salir.

2. La solución queda:

Simplemente cambiando tres


monedas, las señaladas con los
números 1- 2- 3, el triángulo se
invierte.

3. La solución queda:

Llamamos x 1 1 1 y elevamos al cuadrado x2 1 1 1 1

x2 1 1 1 1 x2 1 x x2 x 1 0

1 5 1 5
x x nº áureo.
2 2

168
4. Comenzando el problema desde el final.

Ave 8ª le da 1 1 2.
Ave 7ª (tiene 6) —le da 3 1 4 — le quedan 2.
Ave 6ª (tiene 14) —le da 7 1 8 — le quedan 6.
Ave 5ª (tiene 30) —le da 15 1 16 — le quedan 14.
Ave 4ª (tiene 62) —le da 31 1 32 — le quedan 30.
Ave 3ª (tiene 126) —le da 63 1 64 — le quedan 62.
Ave 2ª (tiene 254) —le da 127 1 128 — le quedan 126.
Ave 1ª (tiene 510) —le da 255 1 256 — le quedan 254.

Al principio tenía 510 gramos de maíz.

5. Las pesas que necesitamos han de ser de: 1, 3, 9 y 27 kg.


Así:
1 kg 1
2 kg 3 1
3 kg 3
4 kg 3 1
5 kg 9 3 1
6 kg 9 3
7 kg 9 3 1
8 kg 9 1
9 kg 9
10 kg 9 1

Y así sucesivamente.

La suma de los números significa que las pesas se colocan en el mismo plato de la balanza, y la
diferencia, que se colocan en platos diferentes.

169
PÁGINA 240

170
SOLUCIONES

1. La recta que pasa por A (1, 3) y B ( 2,6) es : 3 x y 0

2
2. La pendiente de la recta es: m
7

3. Es la función lineal y 2x

4. Las demostraciones quedan:

Sea la relación x1 x2 2 x1 2 x2 2 x1 3 2 x2 3 f ( x1 ) f ( x2 ), luego la función f ( x ) es


estrictamente creciente en todo su dominio.

Sea la relación x1 x2 3 x1 3 x2 3 x1 5 3 x2 5 g ( x1 ) g ( x2 ), luego la función


g ( x ) es estrictamente decreciente en todo su dominio.

5. La ecuación de la recta que pasa por (0, 7) y (3, 6) es:

1
y x 7
3

El punto ( 3, 8) verifica la ecuación, por tanto este punto está en la misma recta que los
anteriores, es decir, los puntos están alineados.

3
6. Una de las posibles ecuaciones es: y x
5

7. Las representaciones quedan:

a) y f ( x )
Dom f
Im f 1,
Estrictamente decreciente 1,1
Estrictamente creciente 1,
No tiene extremos relativos.
Está acotada inferiormente por 1.
No es simétrica ni respecto al eje de ordenadas ni
respecto al origen.

171
b) y g ( x )
Dom g ,1 2,
Im g
Estrictamente creciente ,1 2,
No tiene extremos relativos.
No está acotada.
No es simétrica ni respecto al eje de ordenadas ni
respecto al origen.

8. La solución queda:

a) La función es: P 6 1,8 t , donde P es el precio a pagar en euros y t el tiempo en horas.


b) Queda:

116 116
c) La función será: f ( x ) P (6 1,8t ) .
100 100
Todas las ordenadas de esta función quedan multiplicadas por 1,16.

9. Queda del siguiente modo:

a) La función que nos da el precio a pagar por encuadernar un libro es:

7 si x 200
f (x)
7 0,02 x si x 200
b) La gráfica queda:

172
PÁGINA 241

173
SOLUCIONES

10. Las gráficas quedan:

a) f ( x ) x 2 8 x 12
Dom f
Im f 4,
Estrictamente decreciente ,4 .
Estrictamente creciente 4, .
Mínimo relativo en (4, 4).
Está acotada inferiormente por 4 .Mínimo absoluto 4.
Es simétrica respecto a su eje x 4.

b) g ( x ) x 2 6 x 9 Dom g
Im g 0,
Estrictamente decreciente ,3 .
Estrictamente creciente 3, .
Mínimo relativo en (3,0).
Está acotada inferiormente por 0 .Mínimo absoluto en 0.
Es simétrica respecto a su eje x 3.

c) h( x ) x 2 2 x 3

Dom h
Im h 2,
Estrictamente decreciente ,1 .
Estrictamente creciente 1, .
Mínimo relativo en (1,2).
Está acotada inferiormente por 2 .Mínimo absoluto en 2.
Es simétrica respecto a su eje x 1.

174
d) i ( x ) x2 4x 6

Dom i
Im i , 2
Estrictamente creciente ,2 .
Estrictamente decreciente 2, .
Máximo relativo en (2, 2).
Está acotada superiormente por 2 .
Máximo absoluto en 2.
Es simétrica respecto a su eje x 2.

e) j(x) x 2 6x 5

Dom j
Im j ,4
Estrictamente creciente ,3 .
Estrictamente decreciente 3, .
Máximo relativo en (3,4).
Está acotada superiormente por 4 .
Máximo absoluto en 4.
Es simétrica respecto a su eje x 3.

f) k(x) x 2 4x 4
Dom k
Im k ,0
Estrictamente creciente , 2 .
Estrictamente decreciente 2, .
Máximo relativo en ( 2,0).
Está acotada superiormente por 0 .
Máximo absoluto en 0.
Es simétrica respecto a su eje x 2.

175
11. Las soluciones son:

a) f ( x ) ax 2 bx c
Imponiendo que pase por el punto (0,6), por el punto (2,2) y tiene en este último su vértice,
obtenemos:
f (x ) x 2 4x 6

b) g ( x ) ax 2 bx c
Imponiendo que pase por el punto ( 1,0) , ( 3,0) y tiene su vértice en ( 2,1) , obtenemos:
g(x ) x 2 2x 1

12. Hay infinitas soluciones. Todas las funciones cuadráticas de la forma: y K x 2 1 con K .

13. La representación queda:

A la vista de la gráfica tenemos que :


f ( x ) 0 en ,1 5, .
f ( x ) 0 en 1,5 .
f ( x ) 0 en x 1 y x 5.

14. Buscamos f ( x ) x 2 5 x 6 2 x 2 5 x 4 0. Para calcular los intervalos observamos la


representación de dicha función:

Veamos los intervalos para los cuales la función


g( x ) x 2 5 x 4 0.
En la gráfica se observa que :
g ( x ) 0 en ,1 4, .
g ( x ) 0 en x 1 y x 4.
Luego f ( x ) 2 en ,1 4,

176
15. La función que da el beneficio P en función del tiempo t en días es:

P (10 000 500t ) (0,3 0,02t ) 3 000 50t 10 t 2

Es una función cuadrática, y el beneficio será máximo en el vértice de la parábola, es decir, en


t 2,5 días , y el beneficio será: P 3 062,5 euros.

16. Los números que suman 18 son x y (18 x ). Su producto es P x (18 x ) P x 2 18 x.


Este producto será máximo en el vértice de la función, es decir, para x 9; y 9 y P 81.

17. Se demuestra del siguiente modo:

f ( x z) f ( x ) f (z)
f ( x z ) a ( x z )2 a ( x 2 2 xz z 2 )
Luego f ( x z ) f ( x ) f ( z )
f ( x ) f ( z ) ax 2 az 2

f ( x · z ) f ( x )· f ( z )
f ( x · z ) a ( x · z )2 a·x 2 ·z 2
Luego f ( x · z ) f ( x )· f ( z )
f ( x )· f ( z ) a 2 ·x 2 ·z 2

f (tx ) t · f ( x )
f (tx ) a (tx )2 a·t 2 ·x 2
Luego f (tx ) t · f ( x )
t · f ( x ) t ax 2

18. Las representaciones quedan:

177
19. Las representaciones quedan:

20. La solución queda:

a) y x 26 x6 x2 0 x 0 Los puntos de corte son : (0,0), (1,1) y ( 1,1)


y x x 1

b) y x 57 x7 x5 0 x 0 Los puntos de corte son : (0,0), (1,1) y ( 1, 1)


y x x 1

c) y x 23 x3 x2 0 x 0 Los puntos de corte son : (0,0), (1,1)


y x x 1

178
PÁGINA 242

179
SOLUCIONES

21. La correspondencia queda:

y f (x) x4 1 (d )
7
y g(x ) x 1 (e )
1 3
y h( x ) x (a )
8
y i(x) x5 1 (f )
2
y j(x) 2 x 1 (c )
y m( x ) 2x 4 (b )

22. Las representaciones quedan:

23. La correspondencia queda:


1 1
f (x) n
;con n par g( x ) n
;con n impar
x x
n n
h( x ) x ;con n par k(x) x ;con n impar

24. La solución queda:

a) x 1 x 1 Dos soluciones
b) x 1 x 1 Dos soluciones
c) x 1 Una solución

180
PÁGINA 243

181
SOLUCIONES

25. La verificación queda:

a) x 4 1 Se verifica en 1,1
b) x 5 32 Se verifica en ,2
1 1 1
c) 4 Se verifica en , 0
x2 2 2

26. La representación queda:

27. En cada uno de los casos queda:

a) log4 2,5 log4 3 b) 23 2 2

2 2
1 1
c) log 1 6 log 1 4 d)
5 5
5 5
1 1
e) 0,53 0,5 3
f) log2 log2
4 8

182
28. La solución queda:

a) La gráfica queda:

b) La parte negativa de a gráfica no tiene sentido.

En cada caso:
t 2 p 64% siguen funcionando al cabo de 2 años.
t 5 p 32,768% siguen funcionando al cabo de 5 años.

c) El punto de corte con el eje de ordenadas significa el 100% de ordenadores que


funcionan en el momento de salir de la cadena de montaje.

29. La solución queda:

En el año 1 600 hay una unidad de madera.


En el año 1 800 había (1 50% de 1)2 1,52 unidades de madera.
En el año 1 900 había 1,53 unidades de madera.
En el año 2 005 había 1,5 4,05 unidades de madera.

a) La función es y 1,5t , con t siglos a partir de 1 600.

1
En el año 1 500 había 1,5 0,667 unidades de madera (u m).
2
En el año 1 400 había 1,5 0, 444 u m.
1.5
En el año 1 450 había 1,5 0,544 u m.
6
En el año 1 000 había 1,5 0,087 u m.

183
b) Para que haya el doble de madera que en 1 600 se ha de verificar:

log 2
2 1,5t t 1,710 siglos Es decir, en el año 1 600 171 1771.
log1,5

Para que haya la mitad de madera que en 1 600 se ha de verificar:

1 log0,5
1,5t t 1,710 siglos Es decir, en el año 1 600 171 1429.
2 log1,5

c) Si consideramos la madera en un tiempo t como 1, 5t y queremos saber cuánto tiempo t´


ha de pasar para que la madera se triplique, 3· 1, 5t , obtenemos:

1,5t ´ t
3 1,5t 1,5t ´ 3 t´ 2,710 siglos

Es decir, cada 2,710 siglos o 271 años, la madera se triplica.

30. La solución queda:

La función que da el número de tulipanes al cabo de t años es: N 5t .


Al cabo de 5 años habrá 3 125 tulipanes.
Para que haya 15 625 tulipanes han de pasar: 15 625 5t t 6 años.

31. La demostración queda:

Si (m, p) está en la gráfica de la función y a x entonces:


1 1 1 1
y ax p am a m
m, pertenece a la misma función y a x
p am p p

184
PÁGINA 244

185
SOLUCIONES

32. Queda en cada caso:

1 2 4
a) cos x en ,
2 3 3
5 3
b) tg x 1 en , ,
4 2 4 2
c) 2 sen x en todo el intervalo 0,2
3 11
d) cos x 0 en 0, ,2
2 6 6

33. Queda:
3 240º 360º K
a) arc sen
2 300º 360º K

b) arc tg 1 45º 180º K

1 60º 360º K
c) arc cos
2 300º 360º K

d) arc tg 3 60º 180º K

1 30º 360º K
e) arc sen
2 150º 360º K

f) arc cos 0 90º 180º K

34. Queda:
1 1 y 210º 360º K
a) y arc sen sen y
2 2 y 330º 360º K

3 3 y 30º 360º K
b) y arc cos cos y
2 2 y 330º 360º K

c) y arc tg 1 tg y 1 y 135º 180º K

186
35. En cada caso queda:

a) Se obtiene de trasladar la gráfica y x 4 , 2 unidades hacia abajo.

1
b) Se obtiene de trasladar la gráfica y , 3 unidades hacia arriba.
x3

c) Se obtiene de trasladar la gráfica y log2 x , 2 unidades hacia arriba.

d) Se obtiene de trasladar la gráfica y x 3 , 2 unidades hacia la derecha.

1
e) Se obtiene de trasladar la gráfica y , 1 unidad hacia la izquierda.
x4

f) Se obtiene de trasladar la gráfica y cos x , unidades hacia la derecha.

g) Se obtiene de trasladar la gráfica y sen x , 3 unidades hacia abajo.

h) Se obtiene de trasladar la gráfica y e x , 1 unidad hacia la derecha.

i) Se obtiene de trasladar la gráfica y cos x , unidades hacia la izquierda y ésta 2


2
unidades hacia abajo.

j) Se obtiene a partir de la gráfica y cos x , multiplicando por 2 sus ordenadas.

k) Se obtiene a partir de la gráfica y sen x , con período T 4 .

2
l) Se obtiene a partir de la gráfica y sen x , con período T .
3

36. Decimos:

a) Se obtiene de trasladar la gráfica de la función y x 2 , 1 unidad hacia la derecha y ésta 4


unidades hacia arriba.

b) Se obtiene de trasladar la gráfica de la función y sen x , unidades hacia la izquierda y


ésta 3 unidades hacia abajo.

c) Se obtiene de trasladar la gráfica de la función y e x , 2 unidades hacia la izquierda y


ésta 2 unidades hacia abajo.

187
37. Las resultantes quedan:

d)

188
Unidad 11 – Límites de funciones. Continuidad
PÁGINA 247

SOLUCIONES

1. Podemos decir lo siguiente:

f ( x ) tiende a (3) cuando x tiende a ( ) y tiende a cuando x tiende a .


g ( x ) tiende a ( ) cuando x tiende a ( ) y tiende a cuando x tiende a .
h( x ) tiende a ( 2 ) cuando x tiende a ( ) y tiende a (2) cuando x tiende a .
t ( x ) tiende a cuando x tiende a ( ) y tiende a ( ) cuando x tiende a .

189
PÁGINA 263

SOLUCIONES

1. La solución queda:

Basta con mover el cuadrado para


ver que el área de la región limitada
es la cuarta parte del cuadrado.

2. Basta conocer el lado del cuadrado que se forma dentro de la figura. La resolución nos recuerda
al problema de los perros guardianes.
El área de esta rosa de 4 pétalos es igual al área del cuadrado rayado más 4 veces el área de
un pétalo. El área de un pétalo lo puedes encontrar en el problema de los perros guardianes.

190
3. La representación geométrica del problema así como su resolución quedan:

Los cálculos quedan:

Área ( x ) Área cuadrado Área triángulo 2 Área sector


a 3
a
2 2 a2 3
a 2 a2 1
2 12 4 6

1
Área (rayada) Área círculo Área triángulo rectángulo
4
a2 a2 a2
1
4 2 2 2

Área(y) Área triángulo rectángulo Área(rayada) 2 Área (x)


a2 a2 a2 3 a2 3 a2
2 a2 1 a2
2 4 2 4 6 2 12

a2 a2 a2 3 a2
Área (z) 2 Área (rayada) 2 Área (y) 2 2 a2
4 2 2 12

a2 a2 a2
a 2 2a 2 a 2 3 a2 a2 3
2 6 3

191
4. Sean las figuras:

En la figura (1) el área pedida es 2 veces el área de una de las aspas rayada en el dibujo
adjunto.

Área aspa Área cuadrado 2 Área(a ) 2 Área (b )

Vamos a hallar el área de la zona (a).


El radio de esta zona es la mitad de la
diagonal del cuadrado.

1 1 2 50 25
D 102 102 10 2 r 5 2 y Área(a) r2 5 2 m2
4 4 4 2

Ahora hallamos el área de la zona (b). El radio de esta zona es el lado del cuadrado menos el
radio de la zona (a) r 10 5 2.
1 (75 50 2)
Área (b) (10 5 2)2 m2
4 4
El área del aspa queda:

Área aspa 102 25 (75 50 2) 100 100 50 2

El área pedida queda:

Área pedida 2 Área aspa 2 (100 100 50 2 ) 15,97 m2

Área pedida 15,97 m2

En la figura (2) el área pedida es igual al área del cuadrado de lado 10 m menos el área del
círculo de radio 5 m.

Área figura (2) 102 5 2 100 25 Área figura (2) 21, 46 m2

192
PÁGINA 266

193
SOLUCIONES

1. Las soluciones quedan:

• f (2) 1; f (5) 2; f (–5) no definida;


f (–6) 1; lím f ( x ) no existe;
x 5

lím f ( x ) 1; lím f ( x ) 1;
x 5 x 6

lím f ( x ) 2; lím f ( x ) 1;
x 2 x 2

lím f ( x ) 3; lím f ( x ) 3;
x 5 x 5

lím f ( x ) 1,5; lím f ( x ) 1,5


x 2,5 x 2,5

• g (1) 2; g (2) 0; g (2,5) no existe;


g (3) no existe; lím g ( x ) 2;
x 1

lím– g ( x ) 0; lím+ g ( x ) no existe;


x 2 x 2

lím+ g ( x ) 2; lím– g ( x ) no existe;


x 3 x 3

lím g ( x ) no existe
x 3

2. Las correspondencias quedan:

a) lím g ( x ) 3 b) g (1) 3 c) lím g ( x ) 3


x 1 x 1

3. Las gráficas quedan:

194
195
PÁGINA 267

196
SOLUCIONES

4. Las respuestas quedan:

a) lím f ( x ) h) lím g ( x )
x 1 x 2

b) lím f ( x ) i) lím g ( x )
x 1 x 2

c) lím f ( x ) 1 j) lím g ( x )
x x

d) Asíntota horizontal : y 1. k) Asíntota horizontal : y 0.


Asíntotas verticales : x 1; x 1. Asíntota vertical : x 2.
e) lím f ( x ) l) lím g ( x )
x 1 x 2

f) lím f ( x ) m) lím g ( x ) 0
x 1 x

g) lím f ( x ) 1 n) lím g ( x ) 4
x x 1

5. Las representaciones quedan:

197
6. Los resultados son:

a) lím 2 2 b) lím x –5 0
x 0 x
1 1
c) lím d) lím x 5
x 3 x2 9 x

1
e) lím ( 7) 7 f) lím
x x x 10
0
1 1
g) lím 0 h) lím 13
x – x 10 x 0 x
1
i) lím 13 0 j) lím x 6 1
x x x 1

1
k) lím x 3 0 l) lím
x 0 x x6 0

1 1
m) lím 7
n) lím 7
x 0 x x 0 x
6
o) lím x p) lím x 1
x x 1

198
PÁGINA 268

199
SOLUCIONES

7. Los límites y la gráfica quedan:

lím f (x) lím (2 x 1) 3


x 1 x 1
lím f (x) 3
lím f (x) lím ( 3) 3 x 1
x 1 x 1

lím f (x) lím ( 3) 3


x 2 x 2
lím f (x) no existe
lím f (x) lím ( x 1) 3 x 2
x 2 x 2

lím f (x) ; lím f (x)


x x

8. Los límites quedan:

2
a) lím [2 x 3 7 x 2] b) lím 2
0
x x 3x 5x 2
4
c) lím [4 x 7 x 5] d) lím [ 3 x 5 2 x 4]
x x

2x 2 7x 5
e) lím [ x 2 3 x 2] f) lím 1
x x 2x 2 4 x 3
x2 7 1
3
x 4 3x 2 2
g) lím h) lím
x 2x 2 x 3
4x 2 5
x 3 1 3 x 5 2x 1
i) lím j) lím
x
2x 5 2 x 7x 4 2x 2
x 2 5x 1 2
k) lím 0 l) lím 0
x 2x 3 3 x 1 x3

9. Los límites quedan:

x3 1 0 ( x 1) ( x 2 x 1) 3
a) lím lím
x 1 x3 2x 2 3x 0 x 1 x ( x 1) ( x 3) 4

x2 x 0 x ( x 1)
b) lím lím 1
x 0 x 3 2x 2 x 0 x 0 x ( x 2 2 x 1)

x2 9 0 ( x 3) ( x 3) 6
c) lím lím
x 3 5 x 2 13 x 6 0 x 3 ( x 3) (5 x 2) 17

x4 1 0 ( x 1) ( x 1) ( x 2 1) 4
d) lím lím
x 1 x3 1 0 x 1 ( x 1) ( x 2 x 1) 3

200
x 4 3x 2 0 x 2 ( x 2 3) x ( x 2 3)
e) lím lím lím 0
x 0 x2 x 0 x 0 x ( x 1) x 0 x 1

x2 5x 6 0 ( x 2) ( x 3)
f) lím lím
x 2 x2 4x 4 0 x 2 ( x 2)2

1 x 1 0 ( 1 x 1) ( 1 x 1) 1 x 1 1 1
g) lím lím lím lím
x 0 2x 0 x 0
2 x ( 1 x 1) x 0
2 x ( 1 x 1) x 0
2 ( 1 x 1) 4

x2 1 0 ( x 2 1) ( x 1) ( x 2 1) ( x 1) 0 ( x 1) ( x 1) ( x 1)
h) lím lím lím lím 4
x 1
x 1 0 x 1
( x 1) ( x 1) x 1 x 1 0 x 1 x 1

x 3 0 ( x 3) ( x 3) x 3 0
i) lím 2
lím lím
x 3 x 9 0 x 3
(x 2
9) ( x 3) x 3
(x 2
9) ( x 3) 0

x 3 1
lím
x 3
( x 3) ( x 3) ( x 3) 12 3

3 5 x 0 (3 5 x ) (3 5 x ) (2 8 x) (4 x ) (2 8 x) 4 2
j) lím lím lím
x 4
2 8 x 0 x 4
(2 8 x ) (2 8 x ) (3 5 x) x 4
( x 4) (3 5 x) 6 3

2 x 2 x 0 ( 2 x 2 x)( 2 x 2 x)
k) lím lím
x 0 x2 x 0 x 0
(x 2
x) ( 2 x 2 x)
2x 0 2x 2
lím lím
x 0
(x 2
x) ( 2 x 2 x) 0 x 0
x ( x 1) ( 2 x 2 x) 2

x 2 2 0 ( x 2 2) ( x 2 2) ( 2 x 2)
l) lím lím
x 2
2x 2 0 x 2
( 2x 2) ( 2 x 2) ( x 2 2)
( x 2) ( 2 x 2) 0 ( x 2) ( 2 x 2)
4 1
lím lím
x 2
(2 x 4) ( x 2 2) 0 x 2
2 ( x 2) ( x 2 2) 8 2

10. El límite queda:


f ( x ) f (2) (2 x 2 1) 9
Si f ( x ) 2 x 2 1 f (2) 2 22 1 9 lím lím
x 2 x 2 x 2 x 2
2
2x 8 0 2 ( x 2) ( x 2)
lím lím 8
x 2 x 2 0 x 2 x 2

201
11. Los límites quedan:

x2 1 x2 1 x2 1
a) lím ; lím lím
x 1 x 1 x 1 x 1 x 1 x 1

2 x 2 x 2 x
b) lím ; lím lím
x 0 x2 x 0 x2 x 0 x2

x 5
c) lím
x 3 x 3

2 x 2 2x 2 ( x 2 2x ) 0 2 x ( x 2)
d) lím ·0 lím lím
x 0 x3 3 x 0 3x3 0 x 0 3x3

2 2 x2 1
e) lím x2 1 ·0
lím 2
x x 1 x x 1

1 1 1 x2 x x ( x 1)
f) lím 2 2
lím 0
lím no existe
x 0 x x 2 x 2 ·0 x 0 x 2 ( x 2) ( x 2 2) 0
x 0 x ( x 2) ( x 2 2)
2

202
PÁGINA 269

203
SOLUCIONES

12. Los límites quedan:

a) lím 9 x 2 2x 3 3 x lím
( 9 x 2 2x 3 3 x)( 9 x 2 2 x 3 3 x)
x x
( 9 x 2 2 x 3 3 x)
2x 3 2
1
lím
x
9 x 2 2x 3 3 x 9 3 3

2 ( x 2 3x x ) ( x 2 3x x)
b) lím x x 3 x lím x 3x x lím
x x x 2
x 3x x
3x 3
lím
x
x 2
3x x 2

x2 2 x2 1 x ( x 2 2) ( x 1) ( x 2 1) x2 x 1
c) lím lím lím 1
x x 1 x x x ( x 1) x x2 x

13. Los límites quedan:

3x 5x 2 15 x
5x 2 lím 3 x
5x 3
1 lím
5x 3 3
a) lím e e e
x x
1
x 5x 3

3 x2 1
3 x2 1 2x 7 21
2x 7 2x lím
2x 2x
1
b) lím 1 e x
e 4
x 2x

4x
3x
c) lím
x x 5

x2 5
5 lím x 2 1
2x
1
d) lím 1 1 ex e
x 2x

x2
2 x2 2x2 6x 5 x3 5 x2
2x 6 x 2
x
lím
2 2x2 x 5
1
x
lím
4 x2 2x 10
e) lím 1 e e e 0
x 2x 2 x 5

x 3 4 3x x 3 1
4 3x lím ( x 3)
5 3x
1 lím
5 3x 3
f) lím e e e3 e
x x
1
x 5 3x

204
2 2 6x
x lím (1 3x 1) lím
g) lím 1 3 x 1 e x 0 x
e x 0 x
e6
x 0

x2 3
2 x2 3 x2 1 ( x 2 3) ( x 2 x ) ( x 2 3) x ( x 1)
x 1 x 1 lím
x 1 x 1
1 lím lím
e2
x 1 ( x 1) ( x 1) ( x 1) ( x 1)
h) lím 1 e e x 1
e x 1

x 1 x 1

x 3
4x 2 2
i) lím 2 0
x 2x 2 1

14. Los límites quedan:

2
4 x 2 0 x 2 4x 0 x 4
a) lím 3
lím lím
x 0 x 0 x 0 x3 0 x 0 x2

1 3 x2 x 1 3 x2 x 2 0 x 2
b) lím 3
lím lím lím 1
x 1 x 1 x 1 x 1 x3 1 x 1 x3 1 0 x 1 2
x x 1

x 2
2x 2 5
c) lím 0
x 3x 2 5

x x
x 2 x
x 2 x 2 2 lím
x 5

2 7
d) lím lím e e
x 2
1
x x 5 x x 5

2
32 5 x 1 5 x 4 x 1 5 x
3 5 x 1
e) lím 0
lím lím
x 4
1 5 x 0
x 4
12 5 x
2
3 5 x
x 4
x 4 3 5 x 3

x 2
lím 1
x 2 x 2 x 2 x 2
f) lím lím
x 2 x 2 x 2 x 2 x 2
lím 1
x 2 x 2
5
5 lím x 2 1· 5
x x2 9
g) lím x 2 x2 9
1
e e 6
x 3

x x x4 x3 x4 1
h) lím lím 2
x
x4 x3 x4 1 x 1 x3

205
15. Queda:

x2 2 (ax b ) ( x 2) ( x 2 2) (a 1) x 2 (2a b ) x 2b 2
lím ax b lím lím 0
x x 2 x x 2 x x 2
a 1 0 a 1
Debe ocurrir
2a b 0 b 2

16. Queda:
f (x) 2x 3 f (x h) 2 ( x h) 3
f (x h) f (x ) 2 ( x h) 3 2x 3 0
lím lím
h 0 h h 0 h 0

( 2 x 2h 3 2 x 3) ( 2 x 2h 3 2 x 3)
lím
h 0
h ( 2 x 2h 3 2 x 3)
2h 2 1
lím
h 0
h ( 2 x 2h 3 2 x 3) 2 2 x 3 2x 3

17. Queda:
ax 3 5 x2 3a
3 5x 2 x
lím ax
3 x 5 x2
1
lím 1 e e 5
a 10
x 3x 5x 2

18. Se calcula del siguiente modo:

lím f ( x ) lím x 2 1 1
x 0 x 0

lím f ( x ) lím 3 3 x 3
x 0 x 0

lím f ( x ) lím 3 3 x 0
x 1 x 1

lím f ( x ) 6
x 3
lím f ( x ) lím f ( x ) 6
x 3 lím f ( x ) 6 x 3
x 3

f (0) 3; f (1) 0; f (3) 6


La función f ( x ) es continua en 0

206
PÁGINA 270

207
SOLUCIONES

19. El estudio queda:


x
a) f ( x ) x no es continua en x 0 , pues no está definida en dicho punto.
x
2
b) g ( x ) x E[ x ] es discontinua en todos los puntos de la abscisa entera.
c) h( x ) no es continua en x 0 , pues no está definida en dicho punto.
d) t ( x ) es continua en toda la recta real.

20. La solución queda:


a) f ( x ) es continua en 3,3
b) f ( x ) es continua en 2,2
c) f ( x ) es continua en
d) f ( x ) es continua en 3,
e) f ( x ) es continua en

f) f ( x ) es continua en K
4 2

21. En cada caso queda:


Veamos la continuidad de f ( x ) en x 2 y x 4.
f (2) 0
lím f ( x ) lím x 2 4 0
x 2 x 2
lím f ( x ) 0 f (2) 0 Luego f ( x ) es continua en x 2.
lím f ( x ) lím x 2 0 x 2
x 2 x 2

f (4) 2
lím f ( x ) lím x 2 2
x 4 x 4
lím f ( x ) Luego f ( x ) no es continua en x 4.
lím f ( x ) lím 5 5 x 4
x 4 x 4

Veamos la continuidad de g ( x ) en x 0 y x 3.
g (0) 1
5
lím g ( x ) lím 1
x 0 x 0 x 5 lím g ( x ) Luego g ( x ) no es continua en x 0.
x 0
lím g ( x ) lím x 1 1
x 0 x 0

g (3) 2
lím g ( x ) lím x 1 2
x 3 x 3

10 lím g ( x ) g (3) 2 Luego g ( x ) es continua en x 3.


x 3
lím g ( x ) lím 2
x 3 x 3 x 2

208
22. La solución es:
x2 9 ( x 3) ( x 3) 6
lím lím 3. En x 3 tiene una discontinuidad evitable.
x 3 2x 6 x 3 2 ( x 3) 2
x2 9
si x 3
La redefinimos: f (x) 2x 6
3 si x 3

23. La solución es:


Estudiamos la continuidad en x 1.
f (1) a 2
lím ax 2 2 a 2
x 1
a 2 4 a 6
4
lím 4
x 1 x

Estudiamos la continuidad en x 2.
f (2) a
lím a ln( x 1) a
x 2
a 1
lím e x 2
1
x 2

209
Unidad 12 – Introducción a las derivadas.
PÁGINA 273

SOLUCIONES

1. Los límites quedan:

f (2 h ) f (2) 3 (2 h )2 5 17 12h 3h 2 h (12 3h )


a) lím lím lím lím 12
h 0 h h 0 h h 0 h h 0 h

2 2

g ( x h) g ( x ) 3 ( x h) 1 3x 1 3 x 3h 1 3x 1
b) lím lím lím
h 0 h h 0 h h 0
h 3 x 3h 1 3x 1
3h 3
lím
h 0
h 3x 3 h 1 3x 1 2 3x 1

2. Las ecuaciones de cada una de las rectan quedan:

1
La primera ecuación es: y 3 ( x 2) x 5 y 13 0
5
La recta perpendicular tendrá pendiente 5. Su ecuación es: y 3 5( x 2) 5 x y 13 0

3. La ecuación pedida es: y 6 4 x .

210
4. En cada uno de los intervalos queda:

f1(2) f1(0) 6 f1(4) f1(2) 12 6


a) tvm [0,2] 3 tvm [2,4] 3
2 0 2 4 2 2

f2 (2) f2 (0) 8 2 f2 (4) f2 (2) 14 8


b) tvm [0,2] 3 tvm [2,4] 3
2 0 2 4 2 2

f3 (2) f3 (0) 8 f3 (4) f3 (2) 64 8


c) tvm [0,2] 4 tvm [2, 4] 28
2 0 2 4 2 2

f4 (2) f4 (0) 9 1 f4 (4) f4 (2) 81 9


d) tvm [0,2] 4 tvm [2,4] 36
2 0 2 4 2 2

211
PÁGINA 289

SOLUCIONES

1. El problema se representa del siguiente modo:

En el gráfico está muy clara la situación


del problema y la solución del mismo.
Efectivamente, hay un punto por el que
pasa a la misma hora, y es el punto (*)
en el que se encuentran los dos
trayectos: de ida y de vuelta.

2. La figura queda:

Cuando Luis está a la mitad del camino,


comienza a andar, luego la otra mitad va
a velocidad más lenta.
En cambio, Ana, al correr la mitad del
tiempo, corre más de la mitad del
camino, por lo que menos de la mitad lo
hace andando, así que llega antes Ana.

212
3. El primer cirujano se pone el guante (A) dentro del otro (B), es decir, se pone (A) y encima se
pone al (B).

El segundo cirujano se pone el guante (B) por la cara que no ha tocado al herido.

El tercer cirujano se pone el guante (A) dándole la vuelta y encima de éste (B), operando al
herido por el lado del guante (B) con que ya han operado los otros dos cirujanos.

213
PÁGINA 292

214
SOLUCIONES

1. La tabla queda del siguiente modo:

tvm f1( x ) 3 x f2 ( x ) 3 x 2 f3 ( x ) 3 x 2 f4 ( x ) x 2 f5 ( x ) x 3 f6 ( x ) 3 x

2,0 3 3 3 2 4 4
9

1,1 3 3 3 0 1 5
3
0,2 3 3 3 2 4 4

1,2 3 3 3 3 7 6

a, a 1 3 3 3 2a 1 3a2 3a 1 2·3a

2. En cada apartado queda:

a) Llamando x a la altura del líquido obtenemos:

La ecuación es:

V · x con 0 x 3

b) Queda:

0,5 2 1,5 2,5 2


tvm [0,5; 1] tvm [1,5; 2] tvm [2; 2,5]
0,5 0,5 0,5

c) Queda:

b a
tvm [a; b]
b a

215
3. En cada uno de los casos queda del siguiente modo:

f (4) f (1) 14 5 g (4) g (1) 12 3


tvm [1, 4] 3 tvm [1, 4] 5
4 1 3 4 1 3

8 8
h(4) h(1) 9 6 3 6 t (4) t (1) 20 5 2
tvm [1, 4] tvm [1, 4]
4 1 3 3 4 1 3 5

4. Queda:

e (5) e (2) 4,9 52 4,9 22


Vm tvm [2, 5] 34,3 m/s
5 2 5 2

e (2 h ) e (2) 4,9 (2 h )2 4,9 22 4,9 (4h h 2 ) 0


vi tvi [2] lím lím lím
h 0 h h 0 h h 0 h 0

h (19,6 4,9h )
lím 19,6 m/s
h 0 h

e (5 h ) e (5) 4,9 (5 h )2 4,9 52 4,9 (10h h 2 ) 0


vi tvi [5] lím lím lím
h 0 h h 0 h h 0 h 0

h (49 4,9h )
lím 49 m/s
h 0 h

5. Queda:

e (3) e (0) 3 32 8 3
Vm [0,3] tvm [0,3] 17 km/s
3 0 3

e (5) e (2) 115 28


Vm [2,5] tvm [2,5] 29 km/s
5 2 3

e (8) e (1) 256 11


Vm [1,8] tvm [1,8] 35 km/s
8 1 7

e (12) e (8) 528 256


Vm [8,12] tvm [8,12] 68 km/s
12 8 4

Al alejarse de la Tierra aumenta la velocidad del cohete.

216
6. Las derivadas quedan:

f (1 h ) f (1) 2(1 h )2 3 5 4h 2h 2
a) f' (1) lím lím lím 4
h 0 h h 0 h h 0 h

2 2
g (1 h ) g (1) 1 h 1 2 h 1
b) D [g (1)] lím lím lím 2
h 0 h h 0 h h 0 h 2h 2

h (2 h ) h(2) 3(2 h )2 4 4 3h 2 12h 16 4 3h 2 12h 16 4


c) h' (2) lím lím lím
h 0 h h 0 h h 0
h 3h 2 12h 16 4

3h 2 12h h (3h 12) 12 3


lím lím
h 0
h 3h 2 12h 16 4 h 0
h 3h 2 12h 16 4 8 2

k (0 h ) k (0) 6 6
d) D [k (0)] lím lím 0
h 0 h h 0 h

3 1
l (3 h ) l (3) (3 h )2 3 h 2 6h 6 2
e) l' (3) lím lím lím
h 0 h h 0 h h 0 (3h 2 18h 27) 27 9

t (6 h ) t (6) 6 h 3 3 3 h 3 3 h 3
f) D [t (6)] lím lím lím
h 0 h h 0 h h 0
h 3 h 3
h 1
lím
h 0
h 3 h 3 2 3

7. La derivada queda: f'(x) 3 x 2 6 x


La pendiente de la recta tangente en P ( 1, 2) es: f ' ( 1) 9
La ecuación de la recta tangente es: y 2 9( x 1) 9x y 7 0
1
La pendiente de la recta normal es: 1 y su ecuación es: y 2 ( x 1) x 9y 19 0
9 9

217
8. La solución queda:

Los puntos de corte:


y 2 x 2 12 x 10
P (1,0) Q (5,0)
y 0

La derivada queda: y' 4 x 12

La pendiente de la recta tangente en P es: y' (1) 4·1 12 8


La ecuación de la recta tangente queda: y 0 8( x 1) 8x y 8 0
La pendiente de la recta normal en P es: 1
8
1
La ecuación de la recta normal queda: y 0 ( x 1) x 8y 1 0
8

De igual forma queda en el punto Q.


La ecuación de la recta tangente queda: y 0 8( x 5) 8x y 40 0
1
La ecuación de la recta normal queda: y 0 ( x 5) x 8y 5 0
8

218
PÁGINA 293

219
SOLUCIONES

9. La bisectriz del primer cuadrante es la recta de pendiente m 1.


Luego hay que hallar el punto en el cual:

1 1
f'(x) 1 ·2 x 1 x 1 el punto es P 1,
2 2

10. Realizamos los cálculos a partir de siguiente gráfico:

D f (2) valor de la pendiente de la tangente


y 2 x 1 a la curva y f ( x ) en el punto de
abscisa 2 D f (2) 2.

D f (6) valor de la pendiente de la tangente


y 1 a la curva y f ( x ) en el punto de
abscisa 6 D f (6) 0.

11. Queda:
2 2 1
Calculamos la pendiente mediante la derivada y' m .
x2 16 8
1
El ángulo queda: tg x x 172º 52' 30''
8

12. Queda el siguiente sistema:


f (1) 1 a b 1 a 2
f' (1) 2 a 2 b 1

13. Queda:

a) f '1( x ) 0 b) f' 2 ( x ) 3
c) f' 3 ( x ) 2 x 2 d) f' 4 ( x ) 3 x 2 12
1
e) f' 5 ( x ) f) f' 6 ( x ) 2
x2

220
14. Queda en cada uno de los casos:

f ( x h ) f (x) 3( x h ) 5 ( 3 x 5) 3h
a) f'(x) lím lím lím 3
h 0 h h 0 h h 0 h

2
g ( x h ) g (x) 2( x h ) 3 (2 x 3)2 4h 2 8 xh 12h
b) g'(x) lím lím lím 8 x 12
h 0 h h 0 h h 0 h

h ( x h ) h (x) [( x h )3 2( x h )2 5] ( x 3 2 x 2 5)
c) h'(x) lím lím 3x 2 4x
h 0 h h 0 h

k ( x h ) k (x) 2 ( x h )2 3 2 x 2 3
d) k'(x) lím lím
h 0 h h 0 h
2
4h 8 xh 8x 2x
lím
h 0
h (2 ( x h ) 3 2 x 3 ) 4 x 3
2 2 2
x2 3

15. Queda:

2
f' ( x ) tiene por ecuación : f' ( x ) x
3
1 2
Quedaría f ( x ) x 3 Gráfica C
3

221
PÁGINA 294

222
SOLUCIONES

16. Quedan:

a) D x 6 6x 5

7 5 6 35
b) D D 7x 7·5 x
x5 x6

1 2
c) D 8 4 x D 8x 4
4
x3

d) D 3 x 2 x 4 6x 1

4 3
e) D x2 x 8x 4 x 2 x

5x 20
f) D 2
4 5x 4 5x

1 5 25 x 4 10 x
g) D 5
D x5 x2 3 6
x5 x2 3 x5 x2 3

x2 1 x
h) D
4 2

3 12 x
i) D
2 3
4x 5 4x 2 5

17. Las derivadas quedan:

3
3
x 3
a) D 4 x 4 ln 4 b) D 3 2 x 3 2 x ln 2
x2
2 2 2 2 2 2
c) D e 2 x ex 2 e2 x 4x e x d) D 2 x 3 x D 6x 6 x 2 x ln 6
2x
e e 2x 3 2
e) D f) D e 2 x 1 6 e2 x e2 x 1
4 2

223
18. Las derivadas quedan:

2x
a) D ln x 2 7 2
x 7

ex
b) D ln e x 2
ex 2

12 x 2 60 x 2
c) D ln 3 4 x 3 5
D 5 ln 3 4 x 3 5
3 4x3 3 4x3

1 1 6x 2x
d) D ln 3 3 x 2 1 D ln 3 x 2 1 · 2
3 3 3x 1 3x 2 1

2x
e) D log2 x 2 1 2
x 1 ln 2

1 1
1 x 2 x 2 x x
f) D ln D ln 1 x ln 1 x
1 x 1 x 1 x x x2

1
x 1
g) D ln ln x
ln x x ln x

1 1 1 2x 4 2x 2
h) D ln x 4 x2 D ln x ln 4 x 2
2 x 2 4 x2 x 4 x2

1 x 1 1 2
i) D ln D ln 1 x ln 1 x
1 x 1 x 1 x 1 x2

19. Las derivadas quedan:

a) D sen 4 x 4 cos 4 x

b) D 4 sen x 4 cos x

x 1 x
c) D sen cos
4 4 4

224
4 4 4
d) D sen cos
x x2 x

e) D sen x 4 4 x 3 cos x 4

4
f) D sen4 x D sen x 4 sen3 x cos x

1
g) D arc sen x 1
2 3x 2 x 2

4
4
4 cos x
h) D sen x
x5

cos x
i) D 4
sen x
4 4 sen3 x

j) D cos x 1 sen (x 1)

k) D cos3 x 3 1 9 x 2 cos2 x 3 1 sen x 3 1

4x 2
l) D arc tg (2 x 1)2 4
8x 16 x 3 12 x 2 4 x 1

2x
m) D tg x 2 2 2
cos x2 2

1
n) D tg x
2 x cos2 x

3 tg2 ( x 1)
ñ) D tg3 ( x 1)
cos2 ( x 1)

1
o) D arc cos (ln x )
x 1 ln2 x

3 x ln 3
p) D tg (3 x )
cos2 3 x

1
q) D tg x 2
2 cos x tg x

225
20. Las derivadas quedan:

2x 4 1 1 1 1
a) D f (x) D f (1) b) D g (x) D g (0)
x 2 1 2x 2 3 4 x2 2
2 x ln 2 2 ln 2
c) D h (x) 12 sen 3 x cos 3 x D h( ) 0 d) D j (x) 2
D j ( 1)
2 x
1 9

21. El estudio en cada caso queda:

f ( x ) 2x 4 3 x 3 x 2 ax 5 D [f ( x )] 8 x 3 9 x 2 2x a D [f (1)] 19 a 3 a 22
x2 x a x 2 2x a 1 2 a
g(x ) D [g ( x )] 2
D [g (1)] 0 a 2
x 1 x 1 4

226
PÁGINA 295

227
SOLUCIONES

22. Las derivadas quedan:

1 x 1 3x
a) D (1 x ) 1 x 1 1 x
2 1 x 2 1 x

b) D ( x 2 1) 52 x 2 x 52 x 52 x ln 5 2 ( x 2 1)

c) D 2 x ln 2 2 x (ln 2)2

1 sen x cos x cos x 2


d) D ln
1 sen x 1 sen x 1 sen x cos x

x2
e) D x 2 ln x x ln x 2 2 x ln x 2ln x 2 2 x ln x x 2 ln x 2
x

f) D sen x 32 x cos x 32 x 32 x 2 ln 3 sen x

23 x 23 x 3 ln 2 x 2 2 x 23 x 23 x (3 x ln 2 2)
g) D
x2 x4 x3

1 cos x sen x (1 cos x ) sen x (1 cos x ) 1


h) D
1 cos x 1 cos x 1 cos x
2 (1 cos x )2
1 cos x

x 1 1 2x 9
i) D ln
x 2
9 x 2 x2 9 x ( x 2 9)

j) D x 2a a 2 x e x
2ax 2a 1
a2 x e x
a 2 x ln a 2 x 2a e x
e x
x 2a a 2 x

2a
x 2a a 2 x e x
2 ln a 1
x

1 1 1 x
k) D ln x (2 x ) D ln x ln (2 x )
2 2 x (2 x )

228
x 1 1 x 1 x 1 1
l) D arc tg 2 2 2
x 1 x 1 ( x 1) x 1
1
x 1

1 1 tg x
m) D tg2 x 2 (tg x ) 2
cos x 2 x x cos2 x

e x 20 x 3
n) D ln (e x 5 x 4 )
ex 5x 4

ñ) D (sen 2 x )cos x 2cos x (sen 2 x )cos x 1


cos 2 x (sen x )2 ln(sen 2 x )

1
o) D ln tg x
sen x cos x

p) D ( x 2 1)7 x 7( x 2 1)7 x 1
2 x 2 ( x 2 1)ln( x 2 1)

2x 2 1 4 x (2 x 2 1) 4 x (2 x 2 1) 8x
q) D
2x 2 1 2
(2 x 1)2
(2 x 1)2
2

4 4x 3 x3
r) D x4 2
4 4 ( x 4 2)3 4
( x 4 2)3

1 arc tg x
s) D arc tg2 x 2 arc tg x
2 x (1 x ) x (1 x )

x
x 10 ln 10 1
t) D 10 ln x (2 x )8 ln x (2 x )8 10 x
(2 x )8 16 (2 x )7 ·10 x
ln x
2 x 2x

x2
1 x2
x 1 1 x2 1
u) D arc sen 2
1 x2 2 1 x (1 x 2 ) 1 2 x 2
x
1
1 x2

229
sen x
v) D ln (ln cos x )
cos x ln (cos x )
w) D sen4 x 3 cos3 x 4 4 sen3 x 3 cos x 3 3 x 2 cos3 x 4 3 cos2 x 4 sen x 4 4 x 3 sen4 x 3

1 1 1 1 2 x
x) D ln
1 x 1 x (1 x )2 1 x (1 x )2
2 x ln 2 2 x 2 x ln 2 (8 3 2 x )
y) D 2x 4 2x 2 x ln 2 4 2x
2 4 2x 2 4 2x
2x 2x
1 1
2 2
x 1 x 2 x 1 2 x2 1 2
z) D ln
x2 1 x x2 1 x x2 1 x x2 1

23. Queda:

La velocidad viene dada por la expresión: v (t ) e '(t ) 3t 2 18t


La velocidad instantánea en t 4 es v (4) 24 km
h
2
La velocidad se anula es: 0 3t 18t t 0 y t 6.
Al cabo de 6 horas se anula.

24. Queda:
( 1)n 2 n ! 2 10!
f (n ) ( x ) f (10) ( x )
( x 1)n 1 ( x 1)11

25. La derivada queda: D f ( x ) 3 x 2 12 x 0 y el intervalo resultante es: x ,0 4,

26. Queda:
f ( n ) ( x ) ( 2)n e 2x
f (2 003) ( x ) 22 003 e 2x

27. El punto buscado es el punto en el que la pendiente de la recta tangente valga 1.


f '( x ) 6x 7 1 x 1 El punto es P (1,2).

28. La pendiente de la recta tangente es el valor de la derivada en el punto dado.


4x
Derivamos la función en implícitas: 8 x 2 yy ' 0 y' m 5
y
Por tanto, la ecuación de la recta queda: y 4 5 x 5 y 5x 1

29. El punto es P (8,6).


La pendiente de la recta es: 2x 8 2y ' 0 y' x 4 y '(8) 8 4 m 4
Por tanto, la ecuación de la recta es: y 6 4( x 8) y 4 x 26

230
PÁGINA 296

231
SOLUCIONES

30. La tangente paralela a la bisectriz del segundo cuadrante tendrá pendiente 1.


1 1 1
Por tanto: y ' 2 x 1 x El punto pedido es : ,
2 2 4

31. La solución queda:

x2 y 2
La ecuación de la elipse es: 1.
100 36
Los puntos indicados son:

P 5, 3 3 Q 5, 3 3

R 5, 3 3 S 5, 3 3

Hallamos la pendiente de todas las rectas tangentes a la elipse, que viene dada por la siguiente
expresión:
36 x
y'
100 y

En cada uno de estos puntos la recta tangente queda:


3
• En P y 3 3 x 5
5
3
• En Q y 3 3 x 5
5
3
• En R y 3 3 x 5
5
3
• En S y 3 3 x 5
5

32. El punto es P (1,2) ó Q(1, 10).


Hallemos la pendiente de todas las rectas tangentes a esta circunferencia:

x 3
2 x 2yy ' 6 8 y ' 0 y'
y 4

2 2 8
La recta tangente en P: y 2 x 1 y x
3 3 3
2 2 32
La recta tangente en Q: y 10 x 1 y x
3 3 3

232
33. Mediante la derivada: f ( x ) a para todo valor de x pendiente a

34. La expresión queda:


800 200t
f (t ) 2
0
t 2 8t 18

Resolviendo la inecuación obtenemos la cantidad de agua recogida para t 0, 4 , es decir,


durante los primeros meses del año.
La velocidad fue nula para f (t ) 0 , es decir, para t 4.

35. Por pasar por el punto (1,4) obtenemos: 4 2 a b .


Por tener en ese punto una tangente de pendiente ( 6) obtenemos: 4 a 6.
Resolviendo el sistema obtenemos: a 10 b 12.

36. Los puntos de corte con los ejes son: P (3,0); Q(0,6); R (0, 2).
2
Hallemos la pendiente de todas las rectas tangentes a esta parábola: y
2 y
La recta tangente en P: y 0 1 x 3 y x 3
1 1
La recta tangente en Q: y 6 x y 6 x
2 2
1 1
La recta tangente en R: y 2 x y x 2
2 2

37. La solución queda:

Los puntos comunes son: P (1,2) Q( 1, 2).

Rectas tangentes en P, tienen por pendientes m1 2 m2 2 ; por tanto, el ángulo que


forman es de 0º.
Rectas tangentes en Q, tienen por pendientes m1 2 m2 2 ; por tanto, el ángulo que
forman es de 0º.

38. Queda:
( 1)n 1 (n 1)!
f (n ) ( x )
( x 2)n

233
39. La demostración queda:

Sea la función y eax sen bx, derivamos e introducimos en la expresión a demostrar.


y a eax sen bx b eax cos bx y '' a 2 e ax sen bx 2ab eax cos bx b 2 e ax sen bx
Introducimos las derivadas en la expresión a demostrar :
2ay y '' a 2 eax sen bx b 2 eax sen bx (a 2 b 2 ) eax sen bx (a 2 b 2 )· y

40. La solución queda:

Por pasar por el punto P (1,7) verifica: 7 a b c.


Por pasar por el punto O(0,0) verifica: 0 c.
Por ser tangente a la recta y x y (0) b 1.
Resolviendo el sistema:
a b c 7 a 8
c 0 b 1
b 1 c 0

234
Unidad 13 – Aplicaciones de las derivadas
PÁGINA 299

SOLUCIONES

1. La solución queda:

La función h(x) es estrictamente creciente en el intervalo


, 2 0,2 y estrictamente decreciente en el intervalo
2,0 2, .
Además tiene dos máximos relativos en los puntos 2,5 y
2,5 , y un mínimo relativo en el punto 0,2 .

2. Dos números cualquiera que sumen 20 son ( x ) y (20 x ).


Su producto es: P (20 x ) x 20 x x 2 es una función cuadrática cuya gráfica es una parábola
con un máximo relativo en su vértice (10, 100).
Es decir, los números pedidos son 10 y 10.

235
3. El estudio queda del siguiente modo:

a) Es una función de proporcionalidad inversa. Su gráfica es una hipérbola estrictamente


decreciente en todo su dominio, es decir, en 0 y carece de extremos relativos.
b) Es una función cuadrática. Su gráfica es una parábola con un máximo relativo en su vértice
(4, 32); estrictamente creciente en el intervalo ,4 y estrictamente decreciente en el
intervalo 4, .

4. La solución queda:

Sea la ecuación: 12,5 x 2 150 x 250 0 x 2 y x 10.


Por tanto obtiene ganancias a partir de 2 euros el kilo de caramelos.
El beneficio máximo lo obtiene en el vértice para x 6 euros el kilo, y este beneficio asciende a
200 euros diarios.

236
PÁGINA 313

SOLUCIONES

1. Vamos a organizar los datos en una tabla:

Recibe Marca
Los discos que recibe menos los que marca
Lunes X M son los 20 discos que le quedaron para el
sábado:
Martes X M 12
X X M X 14 4M 4
Miércoles X 14 2M (M 12 2M 10 X ) 20
Jueves 4M 10 3 X 3M 18 3M X 22 20
Viernes 4 X 14 14 2 X 24 X 12 discos recibió el lunes.
Sábado 20

2. Sea v la velocidad del camión y w la velocidad del tractor.


La expresión queda: v w 2(v w ) v 3w
Es decir, la velocidad del camión es el triple que la velocidad del tractor.

3. Llamamos R4 al reloj que mide 4 minutos y R9 al que mide 9 minutos.

Para medir 1 minuto: ponemos ambos relojes. Cuando pasan 4 minutos, damos la vuelta a R4
y al pasar otros 4 minutos, lo que queda de R9 es 1 minuto.

Para medir 2 minutos: conseguimos 1 minuto por el procedimiento anterior. A la vez que
logramos 1 minuto, el reloj R4 lo ponemos y quedan en él 3 minutos. En este momento ponemos
a funcionar R9 y al terminar, quedan en éste 6 minutos; ponemos a funcionar R4 y al terminar
éste último, quedan en el anterior 2 minutos.

Para medir 3 minutos: está explicado en el procedimiento anterior.

237
Para medir 4 minutos: con el reloj R4.

Para medir 5 minutos: ponemos R4 y R9; al terminar R4, quedan en R9 5 minutos.

Para medir 6 minutos: esta situación se explica en el procedimiento para medir 2 minutos.

Para medir 7 minutos: conseguimos 2 minutos por el procedimiento dado anteriormente. Los 2
minutos los tenemos en R9. Ponemos a funcionar R4 y al pasar 2 minutos en R9 quedan otros 2
minutos en R4. Ponemos a funcionar R9 y, al pasar los dos minutos de R4 quedarán 7 minutos
en R9.

Para medir 8 minutos: ponemos dos veces R4.

Para medir 9 minutos: ponemos a funcionar R9.

Para medir 10 minutos: conseguimos que queden 6 minutos en R9 por los procedimientos ya
vistos anteriormente y, cuando pasan esos 6 minutos, ponemos a funcionar R4 obteniendo así
los 10 minutos.

4. en esta figura podemos encontrar los siguientes tipos de triángulos:

En cada figura podemos encontrar


5 triángulos iguales al rayado en la
misma; por tanto, en total hay
5 x 6 30 triángulos.

238
PÁGINA 316

239
SOLUCIONES

1. La solución queda:

a) D f ( x ) 3 x 2 12 x 3 x 2 12 x 0 ,0 4, .
6 6
b) D g ( x ) 0 0,
x3 x3
c) D h( x ) ex xe x e x xe x 0 1,

2. Los intervalos quedan:

5 5
a) f '( x ) 2 x 5 es creciente en , y decreciente en , .
2 2

b) g '( x ) 3 x 2 12 x 9 es creciente en 1,3 y decreciente en ,1 3, .

2
c) h '( x ) es decreciente en 0 .
x2

1 x2
d) i '( x ) 2
es creciente en 1,1 y decreciente en , 1 1, .
1 x2

1
e) j '( x ) es creciente en su dominio 0, .
x

4x
f) k '( x ) 2
es creciente en , 1 1,0 y decreciente en 0,1 1, .
2
x 1

3. Los ingresos aumentan para los valores de p que verifiquen l’(p)>0.


Es decir: l '( p ) 18 6 p 0 p 0,3 .

4. El rendimiento aumenta para los valores de t que hacen f '(t ) 0 y disminuye para los valores de
t que hacen f '(t ) 0 .
f '(t ) 0 t 0, 4
La derivada queda: f '(t ) 3 200 800t
f '(t ) 0 t 4,8

5. La producción aumenta para todos los valores de x que verifiquen P '( x ) 0 .


Queda: P '( x ) 3 x 2 96 x 720 P '( x ) 0 x 12,20 .

240
6. Queda en cada caso:

a) Mínimo en el punto ( 2, 11) .


b) Carece de máximos y mínimos relativos.
c) Máximo en (0,2) y mínimo en (2, 10) .
d) Mínimo relativo en (1, 12) y máximos relativos en los puntos ( 1,20) y (2, 7) .
e) Máximo relativo en (1,5) y mínimo relativo en (3,1) .
f) Máximo relativo en (0,2) .
g) Mínimo relativo en (0,0) .
4
h) Mínimo relativo en (0,0) y máximo relativo en 2, .
e2
i) Mínimo relativo en (0,0) y máximo relativo en 2, 4 .

7. La función debe verificar que f '(3) 0 y f ''(3) 0.


Por tanto:
f '( x ) 3 x 2 2Kx 3 f '(3) 24 6K 0 K 4
f ''( x ) 6 x 2K 6 x 8 f ''(3) 0

Por tanto, el valor buscado es K 4.

8. La función tiene un máximo relativo en (2,7) si verifica:

f (2) 7 2a 4 b 7 b 3
f '(2) 0 a 4 0 a 4

9. Los números son ( x ) y (6 x ) .


El producto queda: P (6 x )· x 2 6 x 2 x 3 .
Este producto es máximo para x 4 . Los números buscados son 4 y 2.

241
PÁGINA 317

242
SOLUCIONES

10. La solución queda:

a) Función Beneficio B( x ) I ( x ) G( x )
B( x ) (2 x 2 360 x ) (4 x 2 120 x 70) 2 x 2 240 x 70

b) B '( x ) 4 x 240 0 x 60
B ''(60) 0 El beneficio es máximo al vender 60 unidades del producto.

c) Para x 60 B(60) 7130 euros.


El beneficio máximo es de 7130 euros.

11. Llamamos x a la anchura de la puerta e y a la altura.


2,5
Obtenemos de las condiciones del enunciado que: x · y 2,5 y .
x
15 4 x 2
Hemos de minimizar el coste del marco, que viene dado por: C 6 y 4 x .
x
4 x 2 15
Derivando e igualando a cero: C '( x ) 0 x 1,94
x2
Como: C ''(1,94) 0 El coste es mínimo para : x 1,94 m; y 1,29 m.

12. Llamando x e y a los lados de la finca rectangular obtenemos:

1600 1600
x · y 1600 y Perímetro 2 x 2y 2 x 2·
x x

Y este perímetro es de longitud mínima para x 40 m e y 40m.


Por tanto, la más barata de cercar es un cuadrado de 40 m de lado.

13. La solución queda:


Observando el dibujo tenemos:

720 2 x
4 x 6 y 1400 y
3
720 2 x
Área 3· x · y 3· x · 720 x 2 x 2
3

El área es máxima para x 180 m e y 120 m.

14. La empresa alcanza el valor máximo en f '(t ) 0 y f ''(t ) 0, es decir, para t 2 años.

243
15. La solución queda:
1
Llamamos r al radio de la base y h a la altura del cilindro. Ocurre: r 2 · h 1 h
r2
2 r 2
Cantidad de material 2 r h 2 r2 2 r2 2 r2
r2 r
Esta cantidad es mínima para:

1 4
r 3 dm; h 3 dm, es decir, las dimensiones son r 0,54 dm y h 1,08 dm.
2

16. La solución queda:

Llamamos r al radio de la base y h a la altura del cilindro. Según las condiciones del enunciado
ocurre que:
6 400 h2
h2 (2r )2 (80)2 r2
4
Operando:
6 400 h 2
Vol r2 h h 6 400 h h 3 V '(h ) 6 400 3h 2 0 h 46,2 cm
4 4 4
V''(46, 42) 0 El volumen es máximo para : h 46,2 cm; r 32,7 cm.

17. Llamando x a la arista básica e y a la arista lateral se debe verificar:

8 x 4y 48 2 x y 12 y 12 2 x

Volumen x 2 y x 2 (12 2 x ) 12 x 2 2 x 3 ; es máximo para x 4 cm e y 4 cm. Es decir, el


prisma de volumen máximo es un cubo de 4 cm de lado.

18. Llamando x e y a los lados del rectángulo se debe verificar: 2 x 2y 24 x y 12 .


Área x · y x ·(12 x ) 12 x x 2 debe ser máximo.
El área es máxima para x 6 cm e y 6 cm.
Por tanto, el rectángulo es un cuadrado de 6 cm de lado.

19. La cantidad de material aumenta cuando la función es creciente y disminuye cuando es


decreciente.
Esta función es creciente ;3,3 10, , y decreciente en 3,3;10 ; por tanto, la cantidad
de material aumenta para t 10,20 y disminuye para t 8,10 .
El máximo de esta función está en (3,3;2,48) , luego la cantidad máxima de material está en
t 3,3 y arroja 2,48 kg, pero esta hora se sale del intervalo fijado.

244
PÁGINA 318

245
SOLUCIONES

20. En cada uno de los casos queda:

2 2
Cóncava hacia las y negativas en , .
3 3
2 2
Cóncava hacia las y positivas en , , .
3 3

a)Cóncava hacia las y negativas en todo .

Cóncava hacia las y negativas en ( 1, ).


Cóncava hacia las y positivas en ( , 1).

Cóncava hacia las y negativas en ( ,0).


Cóncava hacia las y positivas en (0, ).

b)Cóncava hacia las y negativas en 1.

Cóncava hacia las y negativas en ( ,1).


Cóncava hacia las y positivas en (1, ).

21. Quedan del siguiente modo:

2
a) y ' 16 x 4 x 3 y '' 16 12 x 2 x .
3
2 62 2 62
Esta función tiene los dos siguientes puntos de inflexión: , y , .
3 9 3 9
b) Esta función tiene un punto de inflexión en: ( 2,0).
c) Esta función tiene dos puntos de inflexión en: (1, 15) y ( 1, 15).

246
22. La función debe verificar:

3 c c 3
1 3
b 0 b 0 La función queda : y x x 2 3.
3
6a 2 0 1
a
3
13
Esta función presenta un máximo relativo en 2, , un mínimo relativo en 0,3 y un punto de
3
11
inflexión en 1, .
3
Por tanto, no tiene un máximo en 0,3 , sino un mínimo.
No existe ningún valor de a, b, c que verifique las condiciones del enunciado.

23. La función debe verificar:

3 8 2p q p 12
La función queda : f ( x ) x 3 12 x 19.
12 p 0 q 19
Esta función presenta un máximo en 2,35 , un mínimo en 2,3 . Además tiene un punto de
inflexión en 0,19 .

24. La función debe cumplir y ''( x ) 0 e y '''( x ) 0; por tanto, 12 2m 0 m 6.

La función y x 4 6 x 2 2 tiene un punto de inflexión en (1, 3) y otro en ( 1, 3).

25. Las representaciones quedan:

a) b) c)

247
26. Cada una de las gráficas quedan:

a) f ( x ) x 3 3 x 2 3

Dominio .
No presenta cortes con OX enteros.
Corte con OY en (0, 3) .
No tiene simetrías. No tiene asíntotas.
Tiene máximo en (0, 3) y mínimo en (2, 7) .

b) g ( x ) 3 x 2 2 x 3

Dominio .
3
Corte en (0,0) ,0 .
2
Tiene máximo en (1,1) y mínimo en (0,0) .

c) h( x ) 3x 4 4 x 3 12 x 2 1

Dominio .
Corte con ejes en (0,1) .
Máximo en (2,33) y ( 1,6).
Mínimo en (0,1) .

248
9
d) k ( x ) 2
x 9

Dominio 3,3 .
Corte con ejes en (0, 1) .
Simétrica respecto OY.
Asíntotas verticales: x 3; x 3.
Asíntota horizontal: y 0.

x 2
e) l ( x )
x 2

Dominio 2 .
Cortes en (0, 1) (2,0) .
Asíntota vertical: x 2.
Asíntota horizontal: y 1.

x2
f) m( x )
x 1

Dominio 1.
Corte con ejes en (0,0) .
Asíntota vertical: x 1.
Asíntota oblicua: y x 1.

249
8
g) p( x ) 2
x 4

Dominio .
Corte con ejes en (0,2) .
Simétrica respecto a OY.
Asíntota horizontal: y 0.
Máximo relativo en (0,2) .

4 x
h) i ( x )
x2 1

Dominio 1,1 .
Cortes en (0, 4) (4,0) .
Asíntotas verticales: x 1; x 1.
Asíntota horizontal: y 0.
Máximo en (0,2; 3,96) .
Mínimo en (7,8; 0,06).

x2 4
i) v ( x )
x 1

Dominio 1.
Cortes en (0,4) .
Asíntota vertical: x 1.
Asíntota oblicua: y x 1.
13 5
Máximo en 3, y mínimo en 1,2; .
2 2

250
x3
j) w ( x )
x2 4

Dominio 2,2 .
Cortes en (0,0) .
Simétrica respecto al origen.
Asíntota vertical: x 2; x 2 .
Asíntota oblicua: y x.
Máximo en 3,5; 5,2 .
Mínimo en 3,5;5,2 .

27. La función dada es :

En el dibujo está representada


gráficamente la función dada.
Sólo consideramos la parte positiva
de la gráfica (desde t 0 ), pues el
resto no tiene sentido en el contexto.

251
Unidad 14 – Introducción a las integrales y sus
aplicaciones
PÁGINA 321

SOLUCIONES

1. Las primitivas quedan:


a) Primitivas de f ( x ) 4 x son: F ( x ) 2 x 2 3; G( x ) 2 x 2
b) Primitivas de f ( x ) 3 x 2 son: F ( x ) x 3 5; G( x ) x 3 2
c) Primitivas de f ( x ) cos x son: F ( x ) sen x ; G ( x ) sen x 1
d) Primitivas de f ( x ) e x son: F ( x ) e x 3; G( x ) e x 7
5
e) Primitivas de f ( x ) son: F ( x ) 5ln x 2; G( x ) 5ln x
x

2. La solución queda:

En el diagrama cartesiano está representado el recinto


cuya área queremos hallar.
El área vale:
10 2
A ·4 24u2
2

3. Queda en cada caso:


1 2 3
Área ( A1 ) 22 1 3 u2 Área ( A2 ) ·2·2 2 u2 Área ( A3 ) ·2 5 u2
2 2

252
PÁGINA 335

SOLUCIONES

1. La solución queda:

2 7 6 La estrategia consiste en establecer una analogía con el


cuadro mágico 3 x 3 que contiene los nueve primeros
9 5 1 números naturales 1 9 y la constante mágica 15.
Hay que utilizarlo como si se jugase a las tres en raya.
4 3 8

2. En total el nabab tenía 36 gemas y 6 hijos.


35
Al mayor le da: 1 6 gemas. Quedan 30.
7
28
Al 2.º le da: 2 6 gemas. Quedan 24.
7
21
Al 3.º le da: 3 6 gemas. Quedan 18.
7
14
Al 4.º le da: 4 6 gemas. Quedan 12.
7
7
Al 5.º le da: 5 6 gemas. Quedan 6.
7
Al 6.º le da: 6 gemas.

253
3. La solución queda:

r2
Área triángulo ;
2
1
Área lúnula Área semicírculo Área ( x );
2
1 r2 r2
Área ( x ) Área círculo Área triángulo
4 4 2
2
1 r 2 r2 r2 r2 r2 r2 r2
Área lúnula
2 2 4 2 4 4 2 2
Ambas áreas son iguales.

4. Cortó la cadena en 4 trozos de 1, 2, 4 y 8 cm cada uno.

El primer día le dio 1 cm.


El segundo día le dio el trozo de 2 cm y le devolvió la patrona el de 1 cm.
El tercer día le dio el trozo de 1 cm, luego la patrona tiene 1 cm y 2 cm.
El cuarto día le dio el trozo de 4 cm y la patrona le devolvió los dos trozos que tenía.
Así sucesivamente.

254
PÁGINA 338

255
SOLUCIONES

1. Las primitivas quedan:

a) Primitivas de f1( x ) 2 x son: F ( x ) x 2 ; G( x ) x 2 3; H ( x ) x 2 5


b) Primitivas de f2 ( x ) 4 x 3 5 son: F ( x ) x 4 5 x ; G( x ) x 4 5 x 7; H ( x ) x 4 5 x 2
c) Primitivas de f3 ( x ) 2 sen 2 x son: F ( x ) cos 2 x ; G ( x ) cos 2 x 3 ; H ( x ) 5 cos 2 x
1 2x 1 2x 1 2x
d) Primitivas de f4 ( x ) e 2 x son: F ( x ) e ; G( x ) e 7; H ( x ) e 5
2 2 2

2. La solución queda:
1 1
Las primitivas de f ( x ) son: F ( x ) dx ln x 1 C
x 1 x 1

La primitiva que vale 3 para x 0 es: F (0) 3 ln1 C 3 C 3, es decir, F ( x ) ln x 1 3

3. Las funciones cuya derivada es 3x 2 son de la forma: f ( x ) x 3 C


La que pasa por (2, 10) verifica: f (2) 8 C C 2
Por tanto, la función buscada es: f ( x ) x 3 2

4. Basta con demostrar que F '( x ) f ( x ) :


2x 2
F '( x ) 4 x ln x 2 x 4 x ln x f ( x )
x

5. Las integrales quedan:

5x 8
a) 5 x 7 dx C
8
2 2
b) dx 2 x 6 dx C
x6 5x 5
7
4
3 x7
3
3 4 x3
c) x dx x dx
3
C C
7 7
3
x 2 1
d) dx x 5 dx C
x3 4x 4
3
12 x 7
4
3x
e) dx 3 x 4 dx C
4
x 7
x8 x6
f) x 5 ( x 2 1) dx (x7 x 5 ) dx C
8 6

256
(5 x 3)13
g) (5 x 3)12 dx C
65
(6 x 5)4
h) (6 x 5)3 dx C
24
1 1
i) 2
dx (2 x 1) 2 dx C
(2 x 1) 2 (2 x 1)
(sen x )2
j) sen x cos x dx C
2
tg x (tg x )2
k) 2
dx C
cos x 2
ln x (ln x )2
l) dx C
x 2

6. Las integrales quedan:

1 2x 1 32 x
a) 32 x dx 3 2 dx C
2 2 ln 3
b) e sen x cos x dx e sen x C
2

x2 1 1 x2 1 1 4x 1
c) 4 x dx 4 2 x dx C
2 2 ln 4
x
x x 2
1 2
d) 2x dx 2 dx 2 2
2 2
dx 2 C
2 ln 2
eln x
e) dx eln x C
x
x
2 x 1 2 x
f) dx 2 2 dx 2 C
x 2 x ln 2

7. Las integrales quedan:

2 2 3 2
a) dx dx ln 3 x 5 C
3x 5 3 3x 5 3
b) tg x dx ln cos x C
3x 6 3 2x 4 3
c) 2
dx 2
dx ln x 2 4 x 5 C
x 4x 5 2 x 4x 5 2
2x 2 2 9x 2 2
d) 3
dx 3
dx ln 3 x 3 7 C
3x 7 9 3x 7 9
ex 1
e) x
dx ln 2 e x 3 C
2e 3 2
1 1 1
f) 2
dx (ln x ) 2 dx C
x (ln x ) x ln x

257
8. Las integrales quedan:

1 1
a) sen (3 x 1) dx sen (3 x 1) 3 dx cos (3 x 1) C
3 3
1 sen (3 x 2 )
b) 2 x cos (3 x 2 ) dx cos (3 x 2 ) 6 x dx C
3 3
c) (1 tg2 e x ) e x dx tg e x C
3x 3 2x 3
d) 2 2
dx 2 2
dx tg ( x 2 ) C
cos x 2 cos x 2
e) sec 2 x dx tg x C
sen x 1
f) dx 2 sen x dx 2 cos x C
x 2 x

258
PÁGINA 339

259
SOLUCIONES

9. Las integrales quedan:

5
5 9 5 x
a) dx dx arc tg C
9 x2 x
2
3 3
1
3
7 7 5 7
b) dx dx arc sen 5 x C
1 25 x 2 5 1 5x
2 5
6x
3x 3 x 3 4 4 1 3x 2
c) 4
dx 2
dx 2
dx arc tg C
16 9 x 16 3x 2 16 6 3x2 8 4
1 1
4 4
3
8 2 2 8 3x
d) dx 4 dx arc sen C
4 9x 2 3 3x
2 3 2
1
2
ex
e) dx arc tg e x C
1 e2 x
3
3 3 7 7 7 3x
f) 2
dx 2
dx arc tg C
9x 7 7 3 3x 7 7
1
7

10. Las integrales quedan:

2x 1
a) 2
dx ln 7 x 2 3 C
7x 3 7
1 (cos 4 x)4
b) cos3 4 x sen 4 x dx (cos 4 x)3 sen 4 x dx (cos 4 x)3 ( 4 sen 4 x ) dx C
4 16
1 3
3 3 5 2 5 3 3 x4
c) 2 x 7 dx 3x 2x3 7 dx 5 ln x 7x C
x2 x x x 2
1
dx x dx
d) ln ln x C
x ln x ln x

5 1
5 ( x 2 1)3
e) x 2 1 5 x dx ( x 2 1) 2 2 x dx C
2 2
3x 1 1
2 1 3x
f) dx (1 3 x ) 2
( 3 x ln 3) dx C
1 3x ln 3 ln 3

260
4x 2 8 4 3x 2 6 4
g) 3
dx 3
dx ln x 3 6 x C
x 6x 3 x 6x 3
x2 x 1
x2
h) dx x x 2 dx 2 x C
x 2
i) ecos x sen x dx ecos x ( sen x ) dx e cos x C
5x 5 2x 5
j) dx dx arc sen ( x 2 ) C
1 x 4 2 1 (x ) 2 2 2
x x
k) 3 e dx 3e C
ex 1
l) x
dx ln e x x C
e x
3x 1 3 2x 1 3 1 x
m) dx dx dx ln x 2 16 arc tg C
x 2 16 2
2 x 16 x 2
16 2 4 4
3 ( x 2 5)3 ( x 2 5)3
n) 3 x ( x 2 5)2 dx C C
2 3 2
2 5 2
1
5 ( x 2 2 x 3)3
ñ) (5 x 5) x 2 x 3 dx ( x 2 x 3) (2 x 2) dx
2
C
2 3
sen x 1
o) dx ln 3 2 cos x C
3 2 cos x 2
x 2 1
2 1 4x 2
p) dx x (1 4 x 2 ) 2
dx dx arc sen (2 x ) C
1 4x 2 1 (2 x )2 4
1
sen x
sen x 1 2 1 cos x
q) dx dx arc tg C
4 cos2 x 2 cos x
2
2 2
1
2

11. Las integrales quedan:

1
1 x3 4 2 1 2 3
a) (x2 2 x ) dx x2 b) dx ln x 1 ln
0 3 0
3 1 x 1 1 2
3 4
3 1 1 1 4 x4 x3 3x 2 479
c) dx d) (x3 x2 3x 2) dx 2x
2 x2 x 2 6 3 4 3 2 3
12
2
2 2 1 x2 e4 e 1 1
e) x e x dx e f) e x dx ex e 1
1 2 1 2 0 0

6
4
4 x 1 17 6 2 ( x 2)3 16
g) 2
dx ln x 2 1 ln h) x 2 dx
2 x 1 2 2 5 2 3 3
2

261
3 1
3
5 ( x 1)6 32 1
3x e3 x e3 1
i) (x 1) dx j) e dx
1 6 1
3 0 3 0
3
4 1 4 cos 2 x
k) dx 2 x 2 l) sen 2 x dx 0
1
x 1 0 2 0
4 4 ln 2
m) cos x dx sen x 0 n) tg x dx ln cos x
0 0 0 0 2
2
2
2 3 2 (1 x 3 )3 2 52
ñ) x 1 x dx 6
0 9 9 9
0

12. En cada uno de la solución es:

2 32 2
a) Área 2 ( x2 4) dx u
0 3
e 1 e
b) Área dx ln x 1u2
1 x 1

2
2
2x e2 x e4 1
c) Área e dx 26,80 u2
0 2 0
2

262
PÁGINA 340

263
SOLUCIONES

13. Quedan del siguiente modo:

b c b
a) A f (x) dx b) A f (x) dx f (x) dx
a a c
c d b b
c) A f (x) dx f (x) dx f (x) dx d) A f (x) g(x) dx
a c d a
c b
e) A f (x) g(x) dx g(x) f (x) dx
a c
c d b
f) A f (x) g(x) dx g(x) f (x) dx f (x) g(x) dx
a c d

14. En cada caso queda:

3
3
2 x3 28 2
a) (x 9) dx 9x u
1 3 1
3

2
2
2 x3 x2 9 2
b) ( x x 2) dx 2x u
1 3 2 1
2

3 3
2 4 2 4 2 2 4 2
c) cos x dx cos x dx sen x sen x 1 u2
0
2
0
2
2 2

264
15. En cada caso:

a) y x 2 ; y 2
y x2
Puntos de corte: ( 2,2) ( 2,2)
y 2
2
2
2 x3 8 2 2
Área (2 x ) dx 2x u
2 3 2
3
2
b) y x ; y 2 x
y x2 P (0,0)
Puntos de corte:
y 2x Q(2,2)
2
2 x3 4 2
Área (2 x x 2 ) dx x2 u
0 3 0
3
2
c) y x x; y x
y x2 x P (0,0)
Puntos de corte:
y x Q (2, 2)
2
Área
0
[ x2 x ( x )] dx
2
2
2 2 x3 4 2
( x 2 x ) dx x u
0 3 0
3
d) y 2 x 1; y 0; x 0; x 3
3 3
Área (2 x 1) dx x2 x 12 u2
0 0

e) y x 2 4 x 5; y 5
0 4
Área1 ( x 2 4 x 5) dx 5 dx
1 0
5 76 2
( x 2 4 x 5) dx u
4 3

4 32 2
Área2 ( x 2 4 x 5 5) dx u
0 3
f) y x 2 2x ; y x 2
1
Área [ x 2 ( x 2 2x )] dx
0
1
1
2 2x 3 2 1 2
( 2x 2 x ) dx x u
0 3 0
3

265
g) y x 3 3 x 2 ; y x 3
P (1, 2)
y x 3 3x 2
Puntos de corte: Q ( 1, 4)
y x 3
R (3,0)
1
Área [( x 3 3 x 2 ) x 3 ] dx
1
3
[ x 3 ( x 3 3 x 2 )] dx 8 u2
1

h) y 4 x 2 ; y 2 x 2 8
y 4 x2 P (2,0)
Puntos de corte:
y 2x 2 8 Q ( 2,0)
2
Área [(4 x 2 ) 2 x 2 8 ] dx 32 u2
0

1
i) y sen x ; y
2
Puntos de corte en un período [0,2 ]:
1
y sen x P ,
6 2
1
y 5 1
2 Q ,
6 2

5
6 1
Área sen x dx
6
2
5
x
cos x 6
3 u2
2 6 3
j) y sen x ; y cos x ; x 0; x
Puntos de corte en un período [0,2 ]:
2
P ,
y sen x 4 2
y cos x 5 2
Q ,
4 2

5
4
Área cos x sen x dx
4
5

sen x cos x 4
2 2 u2
4

266
16. El cálculo y la gráfica quedan:

1 2
Área 2
sen 2 x dx cos 2 x 1u2
0 2 0

17. El cálculo y el área quedan:

2 2
2 x 2 x 2 7 2
A dx u
1 2 4 4
1

18. Las funciones que verifican f ''( x ) 6 x 6 son: f '( x ) 3 x 2 6 x C.


Como f '(1) 0 f '( x ) 3 x 2 6 x 3 f ( x ) x 3 3x 2 3x K.
Como f (1) 0 1 3 3 K 0 K 1 f ( x ) x 3 3 x 2 3 x 1.

267
Unidad 15 – Distribuciones bidimensionales.
Correlación y regresión
PÁGINA 343

SOLUCIONES

1. Las soluciones quedan:

324,5
La media, x 3,245 ; La moda, Mo 3,25 ; La mediana, Me 3,25
100
1085,25
La varianza, 2 (3,245)2 0,3225 ; La desviación típica, 0,3225 0,5679 ;
100
Los intervalos pedidos son:

(x ,x ) (2,677;3,813) ( x 2 , x 2 ) (2,109;4,381) ( x 3 , x 3 ) (1,5413;4,949)


En el intervalo ( x ,x ) En el intervalo ( x 2 , x 2 ) En el intervalo ( x 3 , x 3 )
hay 86 niñas. hay 96 niñas. hay 99 niñas.

La distribución es casi simétrica, ligeramente desplazada hacia la derecha.

268
2. Es un día raro ya que 15 se sitúa en el intervalo ( x 2 , x 2 ) . La puntuación típica
15 10
z 1,6667 se aleja bastante de la media estándar que es cero.
3

3. La solución queda:

La nube de puntos aparece en la gráfica


de la izquierda.
La recta ajustada a ojo puede ser
bisectriz del cuadrante, y x .
La correlación será positiva y fuerte,
próxima a 1.

269
PÁGINA 357

SOLUCIONES

1. Veamos los dos casos límite:

1.er r 0 V · h · R 2 volumen del cilindro.


2.do r R V · h ·(R 2 R 2 ) 2· · R 2 ·h, pero si r R el volumen es cero.

Luego la fórmula es falsa.

2. La solución queda:

Números felices de 2 cifras:

10 12 02 1 13 12 32 10 12 02 1
23 22 32 13 12 32 10 12 02 1 31 32 12 10 12 02 1
32 32 22 13 12 32 10 12 02 1
44 42 42 32 32 22 13 12 32 10 12 02 1

270
Números felices de 3 cifras:

100 12 02 02 1 130 12 32 02 10 12 02 1
103 12 02 32 10 12 02 1

Igualmente: 310; 301; 230; 203; 320; 302; 440; 404.

Números felices de 4 cifras:

Igual que los que hemos estado formando hasta ahora más otros como 1 339.

Así sucesivamente podemos seguir con los demás.

3. Después de varios intentos vemos que la situación final, para lograr el objetivo buscado, que
debe quedar en la vía muerta superior es: W1 W2 L.
Llamamos A al lugar en que inicialmente está el vagón W1 y B al lugar donde está inicialmente
el vagón W2.
Los pasos a seguir son:

1.º L coge W1 y lo lleva a la vía muerta de abajo.


2.º L da la vuelta al circuito pasando por el túnel y empuja a W2 hasta el punto A.
3.º L coge W1 y lo lleva junto a W2.
4.º L da la vuelta al circuito y empuja a ambos vagones a la vía muerta de arriba,
quedando la situación que buscábamos, W1 W2 L.
5.º L remolca a W2 hasta el punto A.
6.º L da la vuelta al circuito y engancha a W1 llevándolo a la posición B.
7.º L vuelve a la vía muerta de arriba y los vagones han cambiado de posición.

4. La solución queda:
Este problema es una doble simetría.

Construimos P’, simétrico de P respecto a la


banda (1), y Q’ simétrico de Q respecto a la
banda (2).

Unimos P’ y Q’ y llamamos A y B a los puntos


en que la recta P’Q’ corta a las bandas.

La trayectoria pedida es PABQ.

271
PÁGINA 360

272
SOLUCIONES

1. La solución queda:

34 425
La media, x 172,125 ; La moda, Mo 172,25 ; La mediana, Me 172,25
200
59 942 750,1
La varianza, 2 (172,125)2 86,735 ; La desviación típica, 86,735 9,313 ;
200

El número de países en:


(x ,x ) (162,8;181, 4) es 138.
( x 2 , x 2 ) (153,5;190,7) es 188.
( x 3 , x 3 ) (144,2;200,1) es 200.

2. La solución es:
267 285
Las medias aritméticas son: x A 26,7 y x B 28,5.
10 10
7 243 8 209
Las desviaciones típicas son: A (26,7)2 3,38 y B (28,5)2 2,94.
10 10

a) Será aconsejable optar por la marca B, ya que tiene mayor media y, a su vez, menor
desviación típica.

3. En cada caso queda:

a) No existe correlación.
b) Existe correlación negativa y fuerte.
c) Existe correlación positiva y fuerte.
d) No existe correlación.
e) Existe correlación positiva y fuerte.

273
4. La solución queda:

a) La tabla de doble entrada es:

y x
viajes / viajes 1 2 3 4
hijos padres
1 — — — 3
2 — — 1 3
3 — — 1 —
4 — 1 1 —
5 — 2 — —
6 3 3 — —
7 2 — — —

b) El diagrama de dispersión es:

274
PÁGINA 361

275
SOLUCIONES

5. La solución queda:

6. La solución queda:

b) Para ambas variables queda:

390
x 7,8 horas dormidas y x 0,89
50
141
y 2,82 horas dormidas y y 0,71
50

20 16 3
c) El porcentaje de individuos por encima de la media es: 0,78, es decir, el 78%.
50

xy 1078
d) Para el cálculo de r , calculamos la covarianza: xy 7,8·2,82 0,436.
x · y 50

0, 436
Así r 0,69 . La correlación no es muy fuerte y es negativa.
0,89·0,71

7. La solución queda:
1819
La covarianza es AB 12,5·14,3 3,15.
10
xy 3,15
El coeficiente de correlación es: r 0,424.
x · y 2,73· 2,72
La dependencia estadística es moderada.

8. La correspondencia de cada gráfico con su coeficiente de correlación es:

a) r 0,05 c) r 0,98 e) r 0,62


b) r 0,71 d) r 0,93

276
9. La solución queda:

Los parámetros estadísticos son: x 2,68; y 15,4; x 1,98; y 7,96; xy 8,47.

8, 47
a) La correlación es r 0,54.
1,98· 7,96
8, 47
b) La recta de regresión es: y 15, 4 ( x 2,68).
3,92

277
PÁGINA 362

278
SOLUCIONES

10. Llamamos x al número de CDs vendidos e y al número de conciertos. Los datos en una tabla
simple son:

Los parámetros estadísticos son: x 9,8; y 41; x 4,62; y 16,55; xy 62,53.

a) El número medio de CDs vendidos es x 9,8 .

62,53
b) El coeficiente de correlación es r 0,82. La dependencia lineal es moderada.
4,62· 16,55

62,53
c) La recta de regresión es: y 41 ( x 9,8).
21,34

d) Si x 18 y 65,03 conciertos.

11. La solución queda:

Los parámetros estadísticos son: x 5,54; y 25,2; x 2,13; y 7,49; xy 15,42.

15, 42
a) El coeficiente de correlación es r 0,97.
2,13· 7, 49

b) La rectas de regresión son:


15,42
De Y en X: y 25,2 ( x 5,54)
4,54
15,42
De X en Y: y 5,54 ( x 25,2)
51,12

Un chico de x 5 pesa, aproximadamente, y 27,03kg.


La edad que corresponderá a un peso de 36 kg es aproximadamente x 8,72 años.

279
12. La solución queda:

El coeficiente de correlación lineal es nulo si la covarianza es nula.


Por tanto:
3 2a 5 a
( 0,4)· 0 La solución es : a 2,083.
5 5

Los parámetros de las variables son: x 1,8; y 0,4; x 1,72; y 1,85; xy 2,92.
2,92
La recta de regresión de Y sobre X es: y 0, 4 2
( x 1,8) y 0,99 x 2,18
1,72
Si x 2 , el valor estimado de y es: y 0,99( 2) 2,18 4,16

13. La solución queda:

a) La recta de regresión 2x 3y 6 pasa por el punto ( x, y ) , por tanto:


3
2 x 3·1 6 2x 3 x
2
xy
b) El coeficiente de regresión 2
vale para la recta 2x 3y 6 , por tanto:
x

xy 2
2
; al ser x 3, obtenemos xy 2
x 3

xy 2
c) El coeficiente de correlación es: r 0,58
x · y 3·2
2
d) La recta de regresión de X sobre Y es: x 1,5 ( y 1) x 0,5 y 2
22

14. Al ser el coeficiente de correlación r 0,7 ; obtenemos:

xy xy
r 0,7 xy 26,25
x · y 5· 7,5

La recta de regresión de Y (estatura de los hijos) sobre X (estatura de los padres) es:

26,25
y 170 ( x 168) y 1,05 x 6,4
52
Si un padre mide 180 cm, se estima que su hijo tendrá: y 1,05·180 6,4 182,6 cm.

NOTA: Todos los datos se han convertido a centímetros.

280
Unidad 16 – Probabilidad
PÁGINA 365

SOLUCIONES

1. El azar no tiene memoria. Por cualquiera de las dos, cara o cruz.

2 2
4 1 1 3
2. La probabilidad queda: P(2 hembras) · ·
2 2 2 8

3. El resultado más probable es 2 caras y 2 cruces, se presentará 6 veces de cada 16 por término
medio.

4. Recopilamos la información en la siguiente tabla:

Inglés Francés Total


Chicos 27 4 31 69
P(Chica) 0,69
Chicas 63 6 69 100

Total 90 10 100

281
PÁGINA 381

SOLUCIONES

1 (2n 1)
1. La suma queda: 1 3 5 7 ... 2n 1 · n n2
2

2. La solución queda:

1.er piso: se necesitan 2 naipes.


2.º piso: se necesitan 5 naipes.
3.er piso: se necesitan 8 naipes.
4.º piso: se necesitan 11 naipes.

Luego en el enésimo piso habrá (3n 1) naipes.


(3n 1)· n
Una torre con n pisos tendrá: naipes.
2
(3·15 1)·15
Una torre con 15 pisos tendrá: 345 naipes.
2
Veamos cuántos pisos tendrá un castillo de 3 775 naipes:

(3 n 1)· n
3 775 3n 2 n 7 750 0 n 50 pisos
2

282
3. Imaginamos que la rueda del padre tarda 6 segundos en dar una vuelta y la del hijo 6 segundos
en dar vuelta y media.
En la situación de partida vuelven a estar al cabo de 12’’, pero en ningún momento coincidirán
las marcas azules sobre el suelo.

4. El cuadrado de cualquier número entero termina en 0, 1, 4, 5, 6, 9.


Si el número entero es par, su cuadrado es múltiplo de 4.

Así, 142 196 4.

Si el número entero es impar, su cuadrado es múltiplo de 4 1 . Así, 132 169 4 1.


Ahora bien, si el número al cuadrado termina en 111, 555, 666 ó 999, éstos no son ni múltiplos
de 4 ni múltiplos de 4 1 , luego no pueden ser.
Veamos, pues, los que terminan en 000 ó 444.
Efectivamente: 1444 382 , luego también se verifica si no son cero las cifras.

5. La demostración queda:

(2n )! 2n ·(2n 1)·(2n 2)·...·3·2·1


Veamos si es cierta la igualdad anterior transformada en otra :
2
2n ·(2n 1)·(2n 2)·...·3·2·1 1·3· 5·...· (2n 1) · 2n 1
2n 2n ·(2n 2)·...·6·4·2
2n 1 2n 1
1·3· 5·...· (2n 1) (2n 1)·...5· 3·1
Esto es lo que vamos a demostrar por el método de inducción :
Para n 1 2 2·1 1 2 3
2n ·(2n 2)·...·6·4·2
Supongamos que es cierto para n : 2n 1
(2n 1)·...5·3·1
(2n 2)·(2n )·(2n 2)·...·6· 4·2
Veamos que es cierto para n 1: ¿ 2n 3 ? (I)
(2n 1)·(2n 1)·...5·3·1
(2n 2)·(2n )·(2n 2)·...·6·4·2 (2n 2) (2n )·(2n 2)·...·6·4·2 (2n 2)
· 2n 1 2n 3
(2n 1)·(2n 1)·...5·3·1 (2n 1) (2n 1)·...5·3·1 (2n 1)
Elevando al cuadrado : (2n 2)2 (2n 1) (2n 1) (2n 1)2 (2n 3)
16n 4 14n 3 2n 1 0
Esto siempre es cierto, pues n es cierta la desigualdad (I) es cierto el enunciado.

283
PÁGINA 384

284
SOLUCIONES

1. El espacio muestral tiene 23 8 elementos: E (ccc )(ccx )(cxc )( xcc )(cxx )( xxc )( xcx )( xxx ) .

2. Quedan:
a) A C sacar oros
b) A B C
c) A B sacar oros o rey
d) B A rey de oros

3. Quedan:
77
a) No es una probabilidad pues P( A) P(B ) P(C ) P(D ) 1
60
b) Es una probabilidad pues P( A) P(B ) P(C ) P(D ) 1
1
c) No es una probabilidad pues P(B ) 0
3

4. Quedan:
1
P( A) 1 P(B ) P(C )
2
b) Llamando P(C ) x P( A) 6 x ; P(B ) 2 x ; P( A) P(B ) P(C ) 1
1 2
9x 1 x P( A)
9 3

1
5. La probabilidad es P(cruz)
4

6. Queda:
11
P(Al menos un seis)
36
3 1
P(Suma 10)
36 12

7. El espacio muestral tiene 16 elementos.


6 15
a) P(2 caras y 2 cruces) c) P(alguna cara)
16 16
5 5
b) P(como máximo una cruz) d) P(como mínimo 3 caras)
16 16

8. Queda:
1 12 3 16 4
a) P(copas) b) P(figura) c) P(oros o sota)
4 52 13 52 13

285
9. Queda:
5 4 10
a) P(2 negras) ·
14 13 91
9 5 45
b) P(1 roja y 1 negra) · ·2
14 13 91
5 4 81
c) P(al menos 1 roja) 1 ·
14 13 91

10. Queda:
6 7 5 35
Con devolución : · · ·6
18 18 18 162
6 7 5 35
Sin devolución : · · ·6
18 17 16 136

11. Queda:
6 5 3 6 8 4 5 34
a) P(2 de aluminio) · ; P(materiales distintos) · ·
10 13 13 10 13 10 13 65
1 4 3 1 8 7 16
b) P(2 monedas de cobre) · · · ·
2 10 9 2 13 12 65

286
PÁGINA 385

287
SOLUCIONES

12. Introducimos los datos en una tabla:

Mujeres Hombres Total 587,5


a) P(persona sin gafas) 0,47
Gafas 62,5 600 662,5 1250
600
No gafas 187,5 400 587,5 b) P(mujer con gafas) 0,48
1250
Total 250 1000 1250

13. Queda:

Alumnas Alumnos Total


600 3
Ciencias 300 300 600 P(ciencias)
1000 5
Letras 250 150 400
Total 550 450 1000

14. Queda:
1 2
a) P 3 b) P par
impar 3 mayor que 3 3

15. Queda:
1
a) P(mujer) 0,5
Hombres Mujeres Total 2
7
40 años 60 70 130 b) P( 40 años) 0,35
20
< 40 años 40 30 70 70 7
c) P mujer
Total 100 100 200 40 años 130 13

16. Queda:
200
a) P(hombre) 0,67
Hombres Mujeres Total 300
23
Enfermo 12 11 23 b) P(enfermo) 0,77
300
No enfermo 188 89 277 12
c) P hombre 0,52
Total 200 100 300 enfermo 23

288
17. Sean A y B respectivamente la primera y la segunda prueba.

a) P( A B ) P( A) P(B ) P( A B ) 0,6 0,8 0,5 0,9

b) P(no pase ninguna) 1 P(pase al menos una) 1 0,9 0,1

c) No son independientes puesP( A)·P(B ) P( A B)

P( A B ) P(B ) P( A B ) 0,3
d) P B 0,75
A P( A) 1 P( A) 0,4

18. Queda:
12
a) P(defectuosa) 0,04
Diurno Nocturno Total 300
8 2
Defectuosa 4 8 12 b) P nocturno
defectuosa 12 3
No defectuosa 4 1
c) P diurno
defectuosa 12 3
Total 200 100 300

19. Queda:
B2 2 4
P(B1 )·P ·
B1 B1 4 6 4
P
B2 B2 B2 2 4 2 3 7
P(B1 )·P P(R1 )·P · ·
B1 R1 4 6 4 6

289
PÁGINA 386

290
SOLUCIONES

20. Queda:

50 1
a) P(defectuoso y por A) · 0,25
100 2

50 1 25 1 25 1 17
b) P(defectuoso) · · ·
100 2 100 4 100 6 48

25 5
·
100 6 10
c) P C
no defectuoso 25 5 50 1 25 3 31
· · ·
100 6 100 2 100 4

21. Las configuraciones de las urnas son:

1a urna: Dos bolas blancas con probabilidad de salir dos caras.


2a urna: Una bola blanca y otra negra con probabilidad de salir cara y cruz.
3a urna: Dos bolas negras con probabilidad de salir dos cruces.

1
·1
a 1
P 1 urna 4 0,5
blanca 1 1 1 1 2
·1 · ·0
4 2 2 4

22. Queda:

Ingeniero Economista Otros Total


Directivo 15 10 12 37
No directivo 5 10 48 63
Total 20 20 60 100

15
P ingeniero 0,41
directivo 37

23. Queda:
N1 0,20·0,5 5
P
pérdidas 0,20·0,5 0,04·0,5 6

291
24. Queda:
5 4
·
5 4 3 7 41 a 20
a) P(2a roja) · · b) P 1 roja a
8 9
8 9 8 9 72 2 roja 5 4 3 7 41
· ·
8 9 8 9

25. Queda:

1 1 1 1 1 1
a) P(acierte llave) · · · 0,16
3 5 3 7 3 8
1 7 7
b) P(C y no abra) · 0,29
3 8 24
1 1
·
3 5 56
c) P A 0,43
llave abre 1 1 1 1 1 1 131
· · ·
3 5 3 7 3 8

26. Queda:
45 5
·
100 100 225
P 1º 0,34
fallo 45 5 55 8 665
· ·
100 100 100 100

27. Queda:
P( A B ) 0,2 2
a) P( A B ) 0, 4 0,3 0,2 0,5 c) P A
B P(B ) 0,3 3
b) P( A B ) P( A B ) 1 0,2 0,8 d) P( A B ) P(B ) P( A B ) 0,3 0,2 0,1

28. Queda:
1 3 3 1 7
a) P( A B ) P( A) P(B ) P( A B) c) P( A B ) P( A B)
2 8 4 8 8
1 7
b) P( A B ) P( A B) d) P( A B)
4 8

29. Queda:
1 1 5 1 A y B son compatibles pues P( A B ) 0
a) P( A B)
4 2 8 8 A y B son independientes pues P( A B ) P( A)·P(B )
1 1 1 A y B son incompatibles pues P( A B ) 0
b) P( A B) 0
6 3 2 A y B son dependientes pues P( A B ) P( A)·P(B )

292
Unidad 17 – Distribuciones de probabilidad.
Distribuciones binomial y normal
PÁGINA 389

SOLUCIONES

2 2
4 1 1 3
1. La probabilidad es: P (2V y 2M )
2 2 2 8

2. Sabemos que P(Defectuoso) 0,05 .


El número de chips que cabe esperar defectuosos es: 80 000 · 0,05 4 000 chips.

3. Sabemos que: 1,125; 1,452.

En ( , ) ( 0,327;2,577) hay 65 cajas defectuosas, es decir, el 81,25%.


En ( 2 , 2 ) ( 1,779; 4,029) hay 77 cajas defectuosas, es decir, el 96,25%.
En ( 3 , 3 ) ( 3,231;5,481) hay 79 cajas defectuosas, es decir, el 98,75%.

Esta distribución no tiene un comportamiento ¨normal¨.

293
PÁGINA 407

SOLUCIONES

1. La demostración queda:

Veamos que para n 1 se cumple : 2 12 1


Supongamos que se cumple para n p 2 4 6 ... 2 p p 2 p
Veamos qué pasa para n p 1:
2 4 6 ... 2 p 2( p 1) ( p 2 p ) 2( p 1) p 2 3 p 2 ( p 1)2 ( p 1)
Luego queda probado que la igualdad es cierta para todo número natural n.

2. La demostración queda:

Siguiendo el método de la inducción del mismo modo que en el problema anterior obtenemos :
32 1
Para n 1 30 31 4 por lo que se cumple la igualdad.
2
3p 1 1
Supongamos que se cumple para n p : 30 31 ... 3 p
2
p 1
3 1 3p 1 1
Veamos que para n p 1: 30 31 ... 3 p 3 p 1 3p 1
2 2
Luego la igualdad es cierta n .

294
3. La demostración queda:

Utilizando el proceso de inducción obtenemos :

Para n 1 (2 1)2 1 0 8 por lo que se cumple la igualdad.

Para n 2 (4 1)2 1 8 8 por lo que se cumple la igualdad.

Supongamos que se cumple para n p : (2 p 1)2 1 8


2
Veamos que para n p 1: (2 p 1)2 1 2
1 (2 p 1)2 1 (2 p 1) 2 1

(2 p 1)2 4 (2 p 1) 4 1 (2 p 1) 1 8p 8 8 8
Luego la igualdad es cierta n .

4. La demostración queda:

Utilizando el proceso de inducción obtenemos :


Para n 1 S1 a1 (1 1)! 1! 2! 1! 2·1! 1! 1!(2 1) 1!·1 a1
Por lo que se cumple la igualdad.
Supongamos que se cumple para n p : Sp a1 a2 ... ap ( p 1)! 1!
Veamos que para n p 1: Sp+1 a1 a2 ... ap ap 1 ( p 1)! 1! ( p 1)·( p 1)!
(1 p 1)·( p 1)! 1! ( p 2)! 1!
Luego la igualdad es cierta n .

295
PÁGINA 410

296
SOLUCIONES

1. La solución queda:

a) La función de probabilidad es:

Mayor nº 1 2 3 4 5 6
1 3 5 7 9 11
Probabilidad
36 36 36 36 36 36

b) El gráfico queda:

c) Los valores son:

1 3 5 7 9 11
1 2 3 4 5 6 4,47
36 36 36 36 36 36

1 3 5 7 9 11
12 22 32 42 52 62 4,472 1,41
36 36 36 36 36 36

2. La solución queda:

a) La función de probabilidad es:

Mayor nº 0 1 2 3 4 5
6 10 8 6 4 2
Probabilidad
36 36 36 36 36 36

b) El gráfico queda de forma análoga al ejercicio anterior.

297
c) Los valores son:

6 10 8 6 4 2
0 1 2 3 4 5 1,94
36 36 36 36 36 36

6 10 8 6 4 2
02 12 22 32 42 52 1,942 1, 44
36 36 36 36 36 36

3. Queda:

La función de probabilidad es:


X 0 1 2 3 4
1 7 5 2 1
P( X )
16 16 16 16 16

Y los valores de media y desviación típica:

1 7 5 2 1
0 1 2 3 4 1,6875
16 16 16 16 16

1 7 5 2 1
02 12 22 32 42 1,68752 0,98
16 16 16 16 16

4. La función de probabilidad es:

Suma puntos( X ) 0 1 2 3 4 5 6
1 1 2 2 3 3 4
Probabilidad Pi
28 28 28 28 28 28 28
Suma puntos( X ) 7 8 9 10 11 12
3 3 2 2 1 1
Probabilidad Pi
28 28 28 28 28 28

Sus valores de media y desviación típica: 6; 3.

5. La solución en cada caso queda:

a) La función de probabilidad es:


Color Blanco 0 1 2 3
Probabilidad 0,343 0,441 0,189 0,027
b) P ( X 2) P ( X 0) P ( X 1) P ( X 2) 0,343 0, 441 0,189 0,973
c) P ( X 1) P ( X 2) P ( X 3) 0,189 0,027 0,216

298
6. La solución queda:

a) La función de probabilidad es:

X 0 1 2 3 4 5
Pi 0,1681 0,3602 0,3087 0,1323 0,0284 0,0024

b) La media y la desviación típica son: n p 5 0,3 1,5; n p q 5 0,3 0,7 1,025.


P ( X 2) 0,3087
P ( X 3) 0,1323
P (X 2) P ( X 0) P ( X 1) 0,1681 0,3602 0,5283
P (X 3) P ( X 3) P ( X 4) P ( X 5) 0,1631

4 1
7. Es una distribución binomial B 6; 4 , con: P (cara) y P (cruz) .
5 5 5
4 2
6 4 1
a) P ( X 4 caras) 0,2458
4 5 5

b) P ( X 4) P ( X 4) P ( X 5) P ( X 6)
4 2 5 6
6 4 1 6 4 1 6 4
0,2458
4 5 5 5 5 5 6 5

8. Es una distribución binomial B 7;0,3 .


7 7
a) P ( X 7) 0,3 0,000227
7
7 7 7 6 1
b) P ( X 2) 1 P ( X 2) 1 P ( X 0) P ( X 1) 1 0,7 0,7 0,3 0,6706
0 1

9. Es una distribución binomial B 10;0,4 .


10 3 7
La probabilidad pedida es: P ( X 3) 0, 4 · 0,6 0,2150
3

299
PÁGINA 411

300
SOLUCIONES

10. Es una distribución binomial B 7;0, 45 .


7 3 4
a) P ( X 3) 0,45 0,55 0,2918
3

b) P ( X 3) P ( X 3) P ( X 4) P ( X 5) P ( X 6) 1 P ( X 0) P ( X 1) P ( X 2)
7 7 7 1 6 7 2 5
1 0,55 0, 45 0,55 0, 45 0,55 0,6836
0 1 2

c) P ( X 3) P ( X 0) P ( X 1) P ( X 2) P ( X 3)
7 7 7 1 6 7 2 5 7 3 4
0,55 0, 45 0,55 0, 45 0,55 0, 45 0,55 0,6083
0 1 2 3

11. La solución queda:

1 1
a) Es una binomial B 10; . Acertará, por término medio, 10· 2,5 preguntas.
4 4
1 3
b) La desviación típica es: 10 1,37
4 4

c) La probabilidad pedida es:

P( X 5) P ( X 5) P ( X 6) P ( X 7) P ( X 8) P ( X 9) P ( X 10) 0,076

1
12. Es una distribución binomial B 5; .
2
La probabilidad de que una familia formada por 5 hijos sean 2 mujeres y 3 hombres es:
2 3
5 1 1
0,3125
2 2 2

Entre 100 familias cabe esperar que haya: 100·0,3125 31 familias con 2 hijas y 3 hijos.

13. Es una distribución binomial B 5;0,517 .

5 5
a) La probabilidad es: P (al menos 1 niña) 1 P ( X 5 varones) 1 0,517 0,9631.
5
5 5
b) La probabilidad es: P ( X 1) 1 P ( X 0) 1 0,483 0,9737
0

301
14. La solución queda:

I) La representación es:
f1( x ) 0 x
2·1 Luego es una función de densidad.
Árearecinto 1
2

3 1 1
1
a) P (0,5 x 1,5) 4 4 1 0,5 b) P ( x 1) 2 0,25
2 2
0,2 1
c) P ( x 0, 4) 1,6 0,96 d) P ( x 0,65) 0
2

II) La representación es:

f2 ( x ) 0 x
3·1 Luego es una función de densidad.
Árearecinto 1
3

1 1 1 1
a) P (0,5 x 1,5) 1 b) P ( x 1) 1
3 3 3 3
1
c) P ( x 0, 4) 2,6 0,87 d) P ( x 0,65) 0
3

15. La solución es:

La gráfica 1 se corresponde con la distribución N (7;1,5) .


La gráfica 2 se corresponde con la distribución N (5;1,5) .
La gráfica 3 se corresponde con la distribución N (5;3,5) .

a) Las plantas más altas corresponden a la distribución N (7;1,5) . En las otras distribuciones, la
media de las alturas coincide, y en N (5;1,5) están más agrupadas, respecto a la media, que
en N (5;3,5) .

302
16. Manejando la tabla de la distribución normal, hallamos cada caso:

a) P (Z 1, 45) 0,9265
b) P (Z 1,45) 1 P (Z 0,25) 1 0,5987 0,4013
c) P (Z 1,45) 1 P (Z 1,45) 1 0,9265 0,0735
d) P (0,35 Z 1,5) P (Z 1,5) P ( Z 0,35) 0,9332 0,6368 0,2964
e) P ( 1,35 Z 0,25) P (Z 0,25) P (Z 1,35) P (Z 0,25) 1 P (Z 1,35) 0,5102
f) P (Z 0,84) P (Z 0,84) 0,7995
g) P ( 1,45 Z 0,15) P (0,15 Z 1,45) P (Z 1,45) P ( Z 0,15) 0,3669
h) P ( 2,25 Z 2) P (Z 2) P (Z 2,25) P ( Z 2) 1 P (Z 2,25) 0,965

17. En las tablas vemos que:

a) P (Z K ) 0,7967 K 0,83.
b) P (Z K ) 1 P (Z K ) 1 0,1075 0,8925 K 1,24.
c) P (Z K ) 1 P (Z K ) 1 0,1075 0, 4236 K 1,195.

18. Tipificamos la variable X, convirtiéndola en N(0,1) y, posteriormente, consultamos la tabla:

x 5 6 5
a) P ( X 6) P Z P (Z 0,5) 0,6915
2 2
x 5 4,5 5
b) P ( X 4,5) P Z P (Z 0,25) P (Z 0,25) 0,5987
2 2
x 5 7,2 5
c) P ( X 7,2) P Z P (Z 1,1) 0,8643
2 2
3 5 x 5 6 5
d) P (3 X 6) P P ( 1 Z 0,5) 0,5328
2 2 2
e) P (4 X 6).Imposible.

303
PÁGINA 412

304
SOLUCIONES

19. Tipificamos la variable y consultamos la tabla.

a) k 6,76 b ) k 5,1 c) k 1,66

20. La solución queda:

x 10 11 10 1 1
a) P ( X 11) P P Z 1 P Z 1 0,6293 0,3707
3 3 3 3
x 10 8 5 2 2
b) P ( X 8) P P Z 1 P Z 0,2546
3 3 3 3

21. La solución queda:

x 9 11 9 1 1
a) P ( X 8) P P Z P Z 0,6293
3 3 3 3
x 9 5 9 4 4
b) P ( X 5) P P Z P Z 0,9082
3 3 3 3
11 9 13 9 2 4 4 2
c) P (11 X 13) P Z P Z P Z P Z 0,1628
3 3 3 3 3 3

22. La solución es:

13 17 21 17
a) P (11 t 21) P Z P 1,33 Z 1,33 2P Z 1,33 1 0,8164
3 3
t 17 t 17
b) P ( X t ) 0,95 P Z 0,95 1,645 t 21,935 22minutos.
3 3

23. La solución queda:

a) P ( X 8,5) P Z 1 1 P Z 1 0,1587
b) P ( X 7,5) P Z 1 1 P Z 1 0,1587
c) P (7 X 9) P 2 Z 2 2P Z 2 1 0,9544

305
24. La solución queda:

a) P (162 X 178) P 1 Z 1 2P Z 1 1 0,6826


Por tanto el número de alumnos de estatura entre 162 y 178 cm es de 500 · 0,6826 341,3.

b) P ( X 186) P Z 2 1 P Z 2 0,0228
Por tanto el número de alumnos de estatura mayor 186 cm es de 500 · 0,0228 11,14.

25. Llamamos k a la nota mínima a partir de la cual se conseguirá el sobresaliente.


Debe cumplirse:
x 5,5 k 5,5 k 5,5
P 0,9 1,282 k 7, 423
1,5 1,5 1,5

Para el notable ocurre de igual modo:

x 5,5 k 5,5 k 5,5


P 0,7 0,525 k 6,2875
1,5 1,5 1,5

26. Es una distribución binomial B 360, 1 y la aproximaremos con una distribución normal.
6
1 1 5
Quedaría: 360· 60 y 360· · 7,07 .
6 6 6
La probabilidad es:
X ' 60 55,5 60
P ( X 55) P ( X ' 55,5) P P Z 0,64 1 P Z 0,64 0,2611
7,07 7,07

27. Es una distribución binomial B 50;0,9 y la aproximaremos con una distribución normal.

Quedaría: 50·0,9 45 y 50·0,9·0,1 1,12 .

La probabilidad pedida con la corrección de Yates es:

39,5 45 40,5 45
P ( X 40) P (39,5 X ' 40,5) P Z P 2,59 Z 2,12
2,12 2,12
1 P 2,12 Z 2,59 P Z 2,59 P Z 2,12 0,122

306
28. Es una distribución binomial B 100;0,5 y la aproximaremos con una distribución normal.

Quedaría: 100·0,5 50 y 100·0,5·0,5 5 .

La probabilidad pedida con la corrección de Yates es:

45,5 50 X ' 50 55,5 50


P (45 X 55) P (44,5 X ' 55,5) P P 1,1 Z 1,1
5 5 5
P Z 1,1 1 P Z 1,1 0,7286

29. Es una distribución binomial B 100;0,25 y la aproximaremos con una distribución normal
N (25;4,33) .

a) La probabilidad pedida es:


19,5 25
P ( X 20) P ( X ' 19,5) P Z P Z 1,27 P Z 1,27 0,8980
4,33

b) Es una binomial B 20;0,25 que aproximamos a una normal N (5;1,94) .


La probabilidad pedida con la corrección de Yates, como en el apartado anterior, es:

4,5 5
P ( X 4) P ( X ' 4,5) P Z P Z 0,26 1 P Z 0,26 0,3974
1,94

307

Vous aimerez peut-être aussi